First Aid OB GYN Clerkship HY Shelf Review

Pataasin ang iyong marka sa homework at exams ngayon gamit ang Quizwiz!

What age do you initiate pap smears? What age do you stop? How frequent do you test?

Start: 21yo Stop: 65yo 21-29yo: cytology q3yrs *>30*: co-testing (HPV + cytology) q5y, if cytology only then q3y

___________ is the most common benign solid ovarian tumor.

a fibroma

ASC-H How do you manage ASC-H? In pregnant women?

Colposcopy (colpo) with biopsies as indicated by areas with acetowhite change Pregos: · Higher false positive rate for ASCUS-H cyto · Do not defer colpo until postpartum

What is fetal hydrops?

Collection of fluid in two or more body cavities: - Scalp edema - pleural effusion - Pericardial effusion - Ascites

How do you manage *HSIL, AGC or AIS*?

Colpo +/- biopsies + Endocervical currettage (ECC) +/- Endometrial sampling

T/F? LNG-IUD changes serum progestin levels

False

T/F Overall health benefits of HRT appear to exceed the risks

False - risk appear to be > benefits

True Labor vs. False Labor - Compare & Contrast

False Labor = Braxton Hicks Contractions - Occur at irregular intervals - Intensity remains the same - Discomfort in lower abdomen - no cervical change - Pain relieved by medications True Labor - Occur at regular intervals that shorten - Increase in intensity - Discomfort in back and lower abdomen - *cervical change* - NOT relieved by medications

False negatives and positives for urine hCG tests occur when

False negatives: - Performed too early - Urine too dilute False positives: - Proteinuria (confirm with plasma hCG) - UTI

True or False: Vaginal exams should be performed in all patients with PROM. Why?

False! Avoid vaginal exams if possible to decrease risk of chorioamnionitis.

T/F Climateric menstrual irregularities are uncommon.

False, common - Oligomenorrhea --> --> --> Meno-metrorrhagia

T/F: A woman who gives birth to twins has had TWO parous experiences.

False. Giving birth to multiples is considered a single parous experience.

In a pregnant patient with overt signs of diabetes, perform Fingerstick. If fasting fingerstick is > ___ or random > ___, diabetes can be diagnosed.

Fasting > 110 Random > 150

In pregnancy, if fasting FS is > ___ or random FS > ____, consider hospitalization for glycemic control.

Fasting > 140 Random > 200

Gestational Diabetes: - Goal for fasting glucose < ____ - Goal for 2-hr postprandial glucose < ___

Fasting glucose < 95 2-hr postprandial glucose < 120

Gestational vs developmental/embryonic age?

GA is weeks from LMP DA is from fertilization (subtract 2 weeks from GA)

What are the classifications of gestational diabetes?

GDM Classification A1 - controlled with diet GDM Classification A2 - requires insulin

Major SE of Biophosphonates

GI (take with full glass of water and remain standing for 30-60 minutes)

Adverse effects of MTX

GI problems Abdominal pain Vaginal spotting Incr LFTs Alopecia is rare Pneumonitis -> return if fever/respiratory sx

signs of trich

Grey/green, thin discharge Malodorous Vaginal or vulvar irritation 'strawberry cervix' Asymptomatic *Not generally an ascending infection* (PID, endometritis)

What is the most common cause of chorioamnionitis?

Group B Strep (GBS) infection Asx carrier state in vagina and rectum is common

What is HELLP syndrome?

Hemolytic anemia Elevated Liver enzymes Low Platelets - can occur with or without HTN

How do you manage fluid loss in postpartum hemorrhage?

Start two large-bore IVs and infuse isotonic crystalloids. Type and cross blood. Monitor vitals. Strict inputs and outputs.

What are the main types of ovarian cancer?

epithelial (90%) germ cell sex cord stromal

Indications for POP surgery?

childbearing is complete (repeat surgery is not as successful) sx interfering w/ functioning and non-responsive to non-surgical mgmt

<40 post-coital bleeding - #1 cause?

chlamydia Also COCP - risk of cervical ectropion

What component of OCs suppresses FSH and thus prevents follicle maturation?

estrogen

Follicular atresia results in declining _____ and _____ production.

estrogen and inhibin Deep Dive: - estrogen falls dramatically 6months prior to menopause In summary, meno hormonal status: low estrogen (estradiol) high FSH and LH - high FSH is due to lack of inhibition by inhibin and resitance of remaining oocytes to FSH - androstenedione is converted to estrone (less potent than estradiol)

HRT for menopausal women must contain...to prevent endometrial hyperplasia/carcinoma.

estrogen and progestin

in the rare case an IUD user becomes pregnant, there is a greater chance it will be an __________ pregnancy.

ectopic pregnancy, since it is effective in preventing an intrauterine pregnancy Another way of saying this is that it does NOT increase the absolute risk of an ectopic, but if pregnancy does occur with an IUD in place, the pregnancy is more likely to be ectopic.

Embryo vs fetus?

embryo is fertilization - 8 weeks fetus is 9 weeks to birth

Most common gynecologic malignancy in the US is ________________.

endometrial carcinoma 3% lifetime risk

What must be ruled out in a patient with climateric menstrual irregularities?

endometrial lesion histologically (EMB)

Benign conditions that cause increased CA-125?

endometriosis hemorrhagic ovarian cyst (endometrioid) fibroids aka leiomyomas PID pregnancy liver dz

A women with painful periods, pain with sex, and pain with bearing down think...

endometriosis Note: severity of sx may correlate with the depth of penetration of the ectopic tissue not the quantity of it.

A women who complains of hemoptysis during her period think...

endometriosis of the nasopharynx or lung

Bartholin's Cyst/Abscess Cause Si/Sx Tx

forms when the main duct draining bartholin's gland becomes occluded causing a unilateral cyst --> stasis encourages infection, dev rapidly in 2-4 days abscess is fluid-filled, fluctuant mass that is hot and tender - dyspareunia and pain with walking (abscess>cyst as cyst is often asx) - 1-8 cm, tense - may rupture on their own, pain improves -may see pus draining from bartholin's duct if infected Tx If asx monitor if over 40 though, bx to exclude cx if abscess, I&D plus either: - marsupialization: is where you suture the edges of incision to prevent recurrence of cyst) OR Word catheter: has an inflatable tip that you leave in tbe gland for 10-14 days to aid in healing by making an epithealized tract for draining

Premature Ovarian Failure: Defined Presentation Workup Tx

menopause prior to 40 yo Amenorrhea, hot flushes, FSH >30 mIU/ml Autoimmune evaluation, karyotype Tx Estrogen replacement (HRT, OCPs) Oocyte donor for infertility pts

Tx of endometriosis?

lap with lysis of adhesions

Neonatal HPV infection can result in _________________ due to HPV 6 or HPV 11 strains.

laryngeal papillomatosis - hoarseness and resp distress

If rapidly enlarging myoma think...

leiomyosarcoma

Where does fetal erythropoietin originate

liver

Effects of cigarette smoking on the fetus

lowers birth weight by 200g, infants shorter, at greater risk for stillbirth or complicated labor

The Quad screen is performed ideally between _____ and _____ weeks. What does it entail?

maternal blood testing ideally between 16-18 weeks (15-22 ish weeks) Looks at 4 hormones: unconjugated estriol, maternal serum alpha-fetoprotein(MSAFP), beta-hCG, and inhibin A. Essentially, AFP is LOW in Tri 21 and 18, HIGH is NTDs. Doubt you need to know much else here but refer to the table. *sensitivity of 81% overall

Remember that a dark pigmented lesion could be ...

melanoma

What is the most common cause of persistent pyelonephritis despite adequate therapy?

nephrolithiasis

What are the possible fetal effects of maternal Influenza infection?

neural tube defects 2/2 high fever in early pregnancy

Will emergency contraception terminate a pregnancy?

no, it acts to prevent ovulation and fertilization

Workup for suspected ovarian cx?

pelvic US CA-125 (abeit not sensitive or sp) Surgical staging procedure

Signs and symptoms of vulvar cancer?

pruritic ulceration mass often exophytic vaginal bleeding

Vulvar dystrophies are a group of d/o characterized by...

pruritic (itchy), white lesions of the vulva

What is the preferred antiretroviral therapy during pregnancy?

zidovudine-lamivudine (NTRI) + atazanivir -ritonavir (protease inhibitor)

What percentage of tubal ligations can be reversed i.e. result in pregnancy?

~1/3 Note: pregnancy after tubal ligation reversal are ectopic until proven otherwise

What percentage of women who get Depo injections become amenorrheic after 5 years?

80%

What are the two types of endometrial cancer?

80% are type 1 includes endometrioid subtype Type 2 histologic subtypes: clear cell, serous, mucinous, undifferentiated

Tx for all of the vulvar dystrophies?

- topical steroid cream of varying potency *if really bad give po steroids (particularly with severe planus) *If itchy, give diphenhydramine to stop itch in sleep

SE of HRT

-Abnormal uterine bleeding, spotting -Breast tenderness

After having unprotected sex, a woman desires emergency contraception. What is the failure rate of this method?

1%

Types of *HPV* associated w/ cancer? Which cause high grade vs low-grade CIN?

*High-grade CIN*: 16, 18, 31, 33, 35 term-5 *Low-grade CIN & genital warts*: 6, 11 Note: 16 and 18 cause 70% of cervical cx and 50% of cancer precursors

Main etiologic factor in the development of cervical dysplasia?

*Human Papilloma Virus* · Double stranded DNA virus with over 100 serotypes

tx of condylomata acuminata

*Imiquimod (Aldara)* Podophyllotoxin (Podofilox) Cryotherapy Electrosurgery TCA application (depends on location and # warts, pt preference)

Limitations of US Scanning

- Scan dating becomes progressively less accurate and should be utilized only up to 20 w GA - Measures the size of the fetus, not the GA -Biologic variation in size increases as gestation advances

what percentage of men post-vasectomy can still be fertile?

1% Note: this is because it takes ~ 20 ejaculations to clear the ducts of sperm, rec is to use alt contraception for 12 weeks and to have 2 consecutive negative sperm counts

If poor glycemic control: Consider _____ testing for fetal lung maturity and delivery at 37 weeks.

Amniocentesis

Signs and symptoms of menopause (in women not on HRT)

#1 hotflashes (due to declining E) on face, neck, and upper chest last a few minutes and are accompanied by diaphoresis

Risk factors for Ectopic?

#1 is h/o prior ectopic History of 1 ectopic - 10% 2 + ectopics - 25% Damage to tubes e.g. PID or prior pelvic/fallopian surgery (i.e. sterilization) Assisted Repro Technology: ovulation-inducing meds and IVF --> tubal factor infertility and multiple embryo transfer contribute to this risk In utero DES exposure Current IUD use (Note: use of an IUD = lower risk of ectopic pregnancy than women who are not using any form of contraception because IUDs are highly effective at preventing pregnancy. However, up to 53% of pregnancies that occur with an IUD in place are ectopic Septate uterus h/o smoking, >35 yo

Ovarian Germ Cell Tumors: Dysgerminoma - characteristics - biomarkers - tx

#1 malignant germ cell tumor (1/3) - 10% b/l - extremely sensitive to chemo and rad - LDH is the tumor marker

Fallopian Tube Components

(lateral to medial) - Fimbriae - Infundibulum - Ampulla: (Widest) - Isthmus: (Narrowest) - Intrastitial: pierces uterine wall

What medications are used to treat asthma in pregnancy? Complication associated with asthma in pregnancy?

(same as non-pregnant patients) - B-agonists (SABA, LABA) - Epinephrine - inhaled corticosteroids IUGR (increases with severity of asthma)

A woman presents to the emergency department with suspected placental abruption. Maternal blood smear shows *microangiopathic hemolytic anemia*. - What is occurring? - What is the next step in management?

*Disseminated intravascular coagulation (DIC)* Transfuse blood products (PRBCs, platelets, FFP). Expedite vaginal delivery (avoid major surgery in DIC).

What test can be used to predict preterm labor? (vaginal swab)

*Fetal fibronectin assay* If this is negative, 99% predictability of NO labor within one week.

When is *HPV HR DNA* indicated?

*Follow-up for ASCUS* to determine need for colpo Screen *ALL WOMEN >30y/o* in addition to pap smear

Are the *HPV vaccines* effective? What about in males?

*HPV naive*: 97-100% protective against 16, 18 · Which cause 70% of cancer · *40% effective in non-naive* · Long lasting immunity Recommended for *males* now but benefits are controversial · Reduced HPV burden for females · Reduced external genital lesions · Reduced anal/penile cancer risk

LSIL How do you manage LSIL? In pregnant women?

*25-29y/o*: colposcopy *regardless* of HPV result *>30y/o w/ negative HPV* · Preferred is repeat co-testing in 1y or colpo (*most will be HPV +*) For women with two consecutive negative results, return to routine screening. Pregos: Colpo preferred acceptable to defer to 6 weeks PP

What age females are the *HPV vaccines* recommended for?

*9-26y/o* · *Target*: 11-12y/o · *Effective*: <45y/o

Incidence of cervical cytology results

*96%*: negative · *2.8%*: ASC-US · *0.97%*: LSIL · *0.21%*: HSIL · *0.21%*: AGC · *4.5 per 100,000*: SCC

HPV is *reflex* ONLY for what?

*ASC-US* · *21-24y/o*: repeat pap in 1y · *>25y/o*: HPV reflex

________ is the most common cause for postpartum (PP) hemorrhage. How can you treat it?

*ATONY* You will be asked this! TX with Uterotonics: - pitocin - methergine (don't give if hypertensive) - hemabate (don't give if h/o asthma) - misoprostol

Describe the process of Squamous Metaplasia.

*At birth*: no estrogen · Columnar epithelium in endocervical canal · Squamous epithelium in cervix/vagina *During puberty / pregnancy*: estrogen levels increase leading to 2 simultaneous effects 1. Columnar epithelium to proliferate & evert onto ectocervix 2. Acidifies the vagina forcing *columnar* epithelium to *"transform"* back into *squamous* epithelium *Transformation zone is the area btwn the original & new squamocolumnar junctions*

What are the clinical signs of labor?

*Bloody Show* - Discharge of small amount of blood-tinged mucus from vagina (mucous plug) *Rupture of Membranes* (ROM) - Sudden gush of nearly colorless fluid - Diagnosed with pool, nitrazine, and fern tests - most often occurs during the course of active labor (4-6 cm to 10 cm)

Table: Classification of FHR Patterns.

*Category I* FHR tracing - All criteria must be met: - Baseline WNL - Mod variability (6-25 bpm) - No late or variable decels +/- early decels +/- accels *Category III* - Either 1 or 2: 1. Absent variability PLUS one of these: - recurrent late decels - recurrent variable decels - bradycardia OR... 2. Sinusoidal pattern Category II: does not meet criteria for I or III indeterminant significance

What is the *Preinvasive* phase of cervical cancer called?

*Cervical Intraepithelial Neoplasia (CIN)* It's on a spectrum headed twd cervical cx

Pharmacotherapy Urge Urinary Incontinence: Anticholinergics

*Considered equally effective* (start with a lower dose for older adults) Difference in selectivity at receptors---more non-selective = more side effects 1) Darifenacin 7.5 to 15 mg/day 2) Festoterodine 4 to 8 mg/day 3) Solifenacin 5 to 10 mg/day 4) Trospium 20 mg twice daily---ER = 60 mg once daily 5) Oxybutynin immediate release 2.5 mg two to three times daily, XL = 5 to 10 mg/day, Oxtrol OTC patch/4 days, topical gel 6) Tolterodine IR 1 to 2 mg twice daily, LA = 2 to 4 mg/day * = most associated with anticholinergic side effects (can't see, can't pee, cant sh*t, can't spit) think drying

What is the gold standard for diagnosis of lower extremity DVT in pregnancy? And what test is most commonly done?

*Contrast venography* is gold standard (but complications, time consuming, not easy). *Compression ultrasonography* is used most often

What is *colposcopy*? How is it performed?

*Direct visualization with low-magnification microscope* of the cervix, vagina ,and vulva 1. *Cervix cleansed with 3% acetic acid* after 30 secs causes CIN areas to turn white = *"acetowhite"* - acetic acid dehydrates the cells ppt nucleic proteins; those with a higher nucleus to cytoplasmic ratio (neoplastic cells) appear whiter 2. *Green filter accentuates vascular changes* · CIN lesions often accompanied by abnormal vascular achitecture (punctuations, mosaicism) 3. *Biopsies taken of acetowhite + endocervical curettage (ECC) Note: the entire TZ and lesion must be visualized, then the colpo is deemed inadequate

Recommended Pap follow-up for *non-epithelial cell abnormalities*? How are these managed?

*Infections or limited endocervical cells*: Normal interval f/u · *Low risk*: 1yr · *High risk*: 6mo *Mod/severe* inflamm (50-75% cells obscured): *6mo f/u* *Unsatisfactory* (>75% cells obscured): *2-4mo f/u* · Colpo is also option if HPV+ & >30y/o *Atrophy*: Tx w/ topical estrogen & repeat pap in *2mo*

Describe the uterine, cervical, and vaginal changes that occur during the puerperium period.

*Involution of Uterine Corpus* - immediately after delivery fundus is at the umbilicus - contraction is painful esp in first 3 days - within 2 weeks PP uterus descends from umbilicus to cavity of the true pelvis *Endometrial Sloughing and Regeneration* - 2-3 days PP decidua superficial layer becomes necrotic and sloughs off as vaginal discharge (= lochia) and basal layer become new endometrium *Cervix* - within 1 week external os contracts slowly and becomes transverse - multip cervix looks like a fish mouth (horizontal slit *Vagina* - Gradually diminishes in size doesn't return to nulliparous dimensions, rugae reappear by 3rd week; rugae obliterated w/ multiparous and menopause

What medications can be used to manage HTN during pregnancy?

*Labetalol* (alpha and beta adrenergic blocker) alpha *Methyldopa* (centrally acting) *Hydralazine* (vasodilator) *Nifedipine* (CCB, used for tocolysis in labor)

Non-neoplastic results found w/ pap test?

*Negative for Intraepithelial lesion or Malignancy* aka normal *Infection*: Trich, Candida, BV, HSV *Other* non-neoplastic findings (optional) 1. *Reactive*: Inflammation, radiation, IUD 2. *Atrophic* 3. *Hyperkeratosis/parakeratosis*

What is the treatment for Pruritic urticarial papules and plaques of pregnancy (PUPP)?

*Oral antihistamines* and *topical steroids* are the mainstays of treatment. May require *systemic corticosteroids* for severe pruritis. Rash usually disappears shortly before or a few days after delivery.

What is the recommended treatment for hyperthyroidism in the first trimester (T1) of pregnancy?

*PTU in T1* only; due to risk of liver failure beyond T1 PTU inhbits conversion of T4-->T3 Note: small amounts cross placenta

How do you manage *ASC-US or LSIL* in women *21-24y/o*?

*Preference for both*: repeat cytology in 1y

What is the primary risk factor for uterine rupture?

*Prior uterine scar from c-section*: - Vertical scar: 10% risk - Low transverse (bikini) scar: 0.5% risk

List the different regional anesthesia options for labor and delivery

*Pudendal block* - lidocaine to pudendal n - affects lower vagina and posterior vulva; +/- with epidural *aracervical block* - relief of pain from uterine contractions (first stage of labor) - can result in transient fetal bradycardia - need additional anesthesia for delivery as pudendal nerve Spinal (subarachnoid)Block - uncomplicated C-section delivery and vaginal delivery of normal women of low parity

Types of *HPV vaccines*

*Quadravalent (Gardasil)*: 16,18 & 6,11 - this was the first one Gardasil 9 is now available: 6, 11, 16, 18, 31, 33, 45, 52, 58 Intervals: 0,2,6 months IM *Bivalent (Cervarix)*: 16, 18 only Intervals: 0,1,6 months IM *Quadravalent (ACIP)*: males · 11-12y/o but as early as 9y/o - Okay to give to BF moms - Preg class B = not recommended - Give even if prev exposed

*CIN* behavior (regression, persistence, progression)

*Regression* to normal · CIN *I*: 60% · CIN *II*: 40% · CIN *III*: 30% *Persistence* · CIN *I*: 30% · CIN *II*: 35% · CIN *III*: 48% Progression to *CIN III* · CIN *I*: 10% · CIN *II*: 20% Progression to *cancer* · CIN *I*: <1% · CIN *II*: 5% · CIN *III*: 22%

What are the two types of *Pap smears*?

*Sample both endocervical canal & ectocervix, transformation zone* *Conventional*: involves a spatula for collecting cells of the ectocervix and a endocervical brush, spread cells on glass slide and spray fixation on the slide then send to path - false-negative rate *20%*, *Liquid-based* thin layer cytology: cells from the cytobrush and spatula are placed in liquid fixation and swirled to get the cells off, machine prep eliminates confounders like blood, mucus, white cells- also allows for STI testing (gon,chlamydia, +/- HPV) · False negative rate *~8%* Decreased incidence of invasive cervical cancer by 50% in the last 30 years

What is the #1 cause of unsatisfactory cytology on pap testing? How are these managed?

*Scanty cellularity* · Obscuring inflammation or blood · Unlabeled or otherwise not processible Manage (see pic) - if no, unknown, or a negative HPV test result, repeat cytology testing in 2-4 months - if 30+ and HPV+, repeat cytology testing in 2-4 months or colposcopy - if two consecutive unsatisfactory cytology test results then colpo

What is shoulder dystocia?

*Shoulder dystocia* is diagnosed when the fetal shoulder is lodged behind the pubic symphysis after the fetal head has been delivered, and the delivery cannot be completed. *This is an obstetric emergency.* If the infant is not delivered quickly, it may suffer neurologic injury or death from hypoxia.

*Epithelial cell* abnormalities on *paps*

*Squamous cell* 1. Atypical squamous cells (*ASC*) · ASC-US (*unk sig*) · ASC-H (*cannot r/o HSIL*) 2. *LSIL*: HPV, mild dysplasia, CIN I 3. *HSIL*: mod/severe dysplasia, CIN II/III, CIS 4. *SCC* *Glandular cell* 1. Atypical glandular cells (*AGC*) · Favor neoplastic · Adenocarcinoma in-situ (*AIS*) · *Adenocarcinoma*

Prevalence of HPV infection: - in teenagers - 20 to 49 yo - 50+ yo

*Teenagers*: 20% *20-49y/o*: 40% *>50y/o*: 5%

How common is *cervical cancer* in the *U.S* vs *worldwide*?

*U.S*: relatively uncommon due to pap screening · 11th among cancers in women · Typically in those who don't get screened *WW*: 2nd MC cancer in women

Follow-up recommendations for *screening + HR HPV* testing in women *>30y/o*?

*Use genotype* 1. HPV 16/18 Genotype (-): Repeat cotesting in 1y · Repeated co-test (-): Routine screen 3yrs · Pap (-) HPV (+): Colpo/ECC · Pap (+) HPV (any): Colpo/ECC 2. HPV 16/18 Genotype (+): Colpo/ECC · *Long-term*: Based on biopsy results

Uterine activity is monitored by ___________ or _____________.

*external monitor (tocometer)* - measures freq NOT strength (must use IUPC) *intrauterine pressure catheters (IUPC)* - requires ROM to place catheters on fetal scalp - reports freq, duration, and strength - strength of contraction is reported in *Montevideo units* = increases in uterine pressure above baseline (8 to 12 mm Hg) multiplied by contraction frequency per 10 minutes - or if time, simply add them up

In pregnancy where might the DVT originate from besides the calf?

*iliac veins* ~90% on left in pregnancy vs 55% if not preg ~70% proximal vs 10% if not preg (64% were iliac +/- femoral vein)

Episiotomy: Types and associated risks?

- *Midline* (posterior fourchette at 6 oclock) - increases risk of 4th degree tear *Mediolateral* (5 or 7 oclock) - associated with more bleeding and pain than midline cut

What hematologic changes occur puerperium?

- *Leukocytosis* occurs during and after labor up to 30,000/μL - *Lymphopenia* - *Eosinopenia* - Fluctuating Hgb/Hct - plasma fibrinogen and ESR can remain elevated for 1+ weeks PP - *Cardiac output* is elevated for ~48h PP as systemic intravascular volume is increased with decreased blood flow to uterus --> hence, risk for cardiac overload is greatest in the immediate PP period - By 1 week PP, the blood volume has returned to the patient's non-pregnant range

Describe the 4 types of placenta previa: - complete placenta previa - partial placenta previa - marginal placenta previa - low-lying placenta

- *complete placenta previa* - covers entire internal cervical os - *partial placenta previa* - partially covers the internal cervical os - *marginal placenta previa* - one edge of the placenta extends to the edge of the internal cervical os - *low-lying placenta* - within 2 cm of the internal os

Face Presentation

- 0.3% of presentations at or near term - Fetal neck is sharply extended; occiput is in contact with the fetal back - Face is the presenting part - Dx: VE palpation of the fetal face delivery if the *mentum is anterior*, if chin is posterior, must do C/S

Breech Presentation

- 3.5% at or near term; 14% early pregnancy - Presenting part is the buttocks - Term Delivery is C-section - Early pregnancy will often spontaneously convert to vertex as term approaches - Dx: Leopold maneuvers or Ultrasound

In whom is COCPs (due to estrogen) contraindicated?

- 35 yo+ smoker - prior thromboembolism - coronary artery disease - CHF - cerebral vascular disease - migraine with visual aura - uncontrolled HTN - DM with vascular disease - liver tumor (benign or malignant) - liver dz - h/o breast cx - pregnant

Mixed GCTs: - characteristics - markers

- 5% of malignant GCTs - dysgerminoma and yolk sac tumor is the most common combo - LDH, AFP, and BhCG tumor markers

Effacement of cervix during labor

- AKA Length of the cervix with *Normal = 3 to 4 cm* - With labor, the cervix thins out and softens, and the length is reduced - 50% effaced = cervical length shrinks by 50% (2 cm) - 100% effaced = cervix length becomes as thin as the adjacent lower uterine segment - Determination: Palpate with finger and estimate the length from the *internal to external os*

Vertex Presentation

- AKA Occipital Presentation - 96% of at or near term presentations - Head is flexed so that the chin is in contact with the chest - *Posterior fontanel*= presenting part

Position of cervix during labor

- AKA location of cervix with respect to the fetal presenting part Classifications: - Posterior— difficult to palpate because it is *behind* the fetus, and usually *high* in the pelvis - Midposition - Anterior—easy to palpate, *low* down in pelvis - During labor, the cervical position usually progresses from posterior to anterior.

Oxytocin side effects

- Antidiuretic effects (similar structure to ADH) --> water intoxication --> hyponatremia --> convulsions, coma, and death - uterine tachysystole (hyperstimulation) --> abnl FHR

Late Decelerations in FHR: - nl or abnl? - cause - characteristic look - intervention? - when to consider delivery?

- Are abnormal - Due to *uteroplacental insufficiency* (not enough blood) during contractions; can follow epidural (hypotension) or uterine hyperstimulation (too much pit) - gradual decrease in HR with the onset, nadir, and recovery occuring after the contraction onset, peak and end, respectively. Interventions: *HOT PAP* (Hydration, Oxyygen, Tocolysis, Position, Amniotomy, and discontinue Pitocin) > Mainly: Change maternal position to the left *lateral recumbent position*, *O2* face mask, *stop oxytocin* (Pitocin) infusion > Additional: IV fluid bolus (esp if hypotensive), IV tocolytic (MgSO4), Monitor maternal BP for hypotension, check for cord prolapse - can use ephedrine or phenylephrine for epidural-induced hypotension If still > 50% of contractions have late decels, consider immediate delivery (operative or C/S)

Variable Decelerations in FHR: - nl or abnl? - cause - characteristic look - intervention? - when to consider delivery?

- Are abnormal and can be mild, moderate, or severe - Due to *cord compression* (*V*EAL *C*HOP) seen in *oligohydramnios* or with *nuchal cord* - sometimes head compression; if repetitive, suspicion is high for the cord to be wrapped around the neck or under the arm of the fetus. - abrupt decel that looks like a "V" and can occur at anytime (contracting or not) - Intervention: > Main: Amnioinfusion = Infuse normal saline into the uterus through the IUPC to alleviate cord compression > Additional: Change maternal position to side/Trendelenburg, if severe consider forceps delivery or C-section

Early Decelerations in FHR: - nl or abnl? - cause - characteristic look - intervention?

- Are normal - Due to *head compression* during contractions (V*E*AL C*H*OP) - Onset, peak, and end coincides with the timing of the contraction - Degree of deceleration proportional to contraction strength. - Effect is regulated by vagal nerve activation - NO Intervention is necessary

What are the risk factors for postpartum hemorrhage?

- Blood transfusion/hemorrhage in previous pregnancy - Coagulopathy - Trial of labor after cesarean - High parity - Large infant/twins or polyhydramnios - Midforceps delivery - Chorioamnionitis

What are the complications of shoulder dystocia?

- Brachial plexus nerve injuries (Erb's palsy) - Fetal humeral/ clavicular fracture - Hypoxia, death

Ligaments of the pelvic viscera, their course and what structures they house?

- Broad ligament: Extends from the lateral pelvic wall to the uterus and adnexa. Contains the fallopian (uterine) tube, round ligament, uterine and ovarian blood vessels, lymph, utererovaginal nerves, and ureter - Round ligament: The remains of the gubernaculum; extends from the corpus of the uterus down and laterally through the inguinal canal and terminates in the labia majora; contains Sampson artery - Cardinal ligament: Extends from the cervix and lateral vagina to the pelvic wall; functions to support the uterus - Uterosacral ligament -paired ligaments that secure uterus to sacrum; functions to support the uterus

Management of Breech Deliveries Which types of breech position(s) are not delivered vaginally and why?

- C-section - External cephalic version - maneuvering the infant to a vertex position - Trial of frank breech vaginal delivery in frank only No TOL for incomplete and complete breech due to high risk of umbilical cord prolapse

Types of Breech (%)

- Frank breech (65%): Thighs are flexed and the legs are extended; feet are in front of the head or face - Complete (25%) - thighs and legs are flexed on the abdomen - Incomplete aka footling (10%): One or both of the hips are not flexed and a foot lies below the buttocks

What instructions must you give the patient to ensure proper usage of the pessary?

- Ensure pt is using vaginal estrogen, and using it properly. - Instruct pt how to remove, clean, and replace the pessary —> encouraged to be done weekly to prevent infection.

DDx of Benign Breast Mass

- Fat necrosis - Fibrocystic breast changes - Fibroadenoma - Phylloides Tumor - Atypical hyperplasia

What is the appropriate management of pregnancy in a woman with chronic hypertension?

- Fetus should undergo testing to assess for adequate perfusion. - Fetus should receive ultrasounds to monitor growth. - Deliver at term (unless complications develop) - Vaginal delivery is preferred over C-section.

What are the maternal effects of gestational diabetes?

- Four times increased risk of preeclampsia - increased risk of bacterial infections - higher rate of C-section - increased risk of polyhydramnios - increased risk of birth injury

Components of the Uterus

- Fundus: Uppermost region of uterus - Corpus: Body of the uterus - Cornu: Part of uterus that joins the fallopian tubes - Cervix: Inferior part of cervix that connects to the vagina via the cervical canal - Internal cervical os: Opening of cervix on the uterine side - External cervical os: Opening of cervix on the vaginal side

External cephalic version requirements and risk?

- If breech is diagnosed before onset of labor and the GA is 35-37 weeks, 50% success rate - Risks: Placental abruption, cord compression, reversion

What test is done to asses fetal lung maturity?

Amniocentesis

List 3 causes of slowed (protraction disorder) active phase in 1st stage of labor.

- Inadequate uterine contractions - Fetal malposition - Cephalopelvic disproportion

What are the fetal effects of gestational diabetes?

- Increased risk of perinatal death (A2 > A1) - Fetal anomalies not increased in gestational diabetes - Two to three times increased risk of preterm delivery - Hyperinsulinemia --> fetal macrosomia --> birth injury (shoulder dystopia) - Hyperglycemia affects most fetal organs except brain - Excessive fat on shoulders and trunk - Metabolic derangements at birth (Hypoglycemia, hypocalcemia)

What are the complications associated with overt hypothyroidism in pregnancy?

- Infertility - Higher miscarriage rates - Preeclampsia - Placental abruption - cardiac dysfunction - low birth weight - still births

Breech risk factors?

- Low birth weight (20 to 30% of breeches) - Congenital anomalies such as hydrocephalus or anencephaly - Uterine anomalies - Multiple gestation - Placenta previa - Hydramnios or oligohydramnios - multiparity

SECONDARY AMENORRHEA Possible causes of hypothalamic dysfunction?

- May have low GnRH, low FSH/LH, low E - absent withdrawal bleed after progesterone challenge Critically low body fat (anorexia) Significant physical/psych stress (overexercise) Genetic (KAL1, FGFR1, GnRH1 etc. mutations) Infiltrative lesions (craniopharyngiomas) Systemic illnesses (encephalitis) Kallmann's syndrome Drugs (e.g. OCP can have amenorrhea for up to 6 mo after the pill is stopped)

What is the clinical presentation of uterine rupture?

- Non-reassuring fetal heart tones or bradycardia - Sudden cessation of uterine contractions - "Tearing" sensation in abdomen - Presenting fetal parts move higher in pelvis - Vaginal bleeding - Maternal hypovolemia from concealed hemorrhage

What criteria makes someone a candidate for TOLAC?

- One LTC - Clinically adequate pelvis - No other uterine scars or previous rupture - Physician monitoring TOL and equipped to perform emergent CD if necessary

Ovaries Blood Supply

- Ovarian artery - from aorta at L1 - Veins: R drains to vena cava; L drains to left renal vein

Complications of IUD

- PID related to insertion (1 percent in first 20 days, and 0.5 percent in first 3 to 6 months) - Uterine perforation - ectopic pregnancy (see later card for further explanation on this) - expulsion - menorrhagia and dysmenorrhea (with copper IUD) - Actinomyces infection (flourish on foreign bodies)

What are the si/symptoms of pelvic relaxation?

- Pelvic pressure/heaviness - protrusion of tissue from vagina (feels like sitting on a lump), feels a "bulge" - Low back pain - incontinence - post-coital vaginal bleeding - spotting - splinting used to defecate Often relieved when lying down, so less noticeable in the morning and worsens throughout the day.

Risk factors for Vulvar carcinoma

- Postmenopausal - Age - Smoking - Immunodeficiency - HPV (16. 18, 31, 33) - VIN (precancerous) - h/o vulvar disease (lichen sclerosis or planus)

Post-Partum Psychiatric Disorders

- Postpartum Blues - Postpartum Depression - Postpartum Psychosis

Three "Ps" that affect the duration of the Active Phase?

- Power(strength and frequency of contractions) - Passenger (size of the baby) - Pelvis(size and shape of mother's pelvis) assess these three things if prolonged active phase

RFs for PP Pelvic Infection?

- ROM > 18hr - prolonged labor - CS - GBS positive - preterm delivery - multiple cervical checks - manual placental extraction - DM - chorioamnionitis - internal monitoring

List indications for Cesarean delivery (CD).

- Prior CS (elective repeat where pt does not want a TOLAC or previous classical) - Dystocia or failure to progress in labor - Breech or transverse - Concern for fetal well-being (non-reassuring FHT) - Uterine malformation/scarring

What are potential complications of pregnancy in women with chronic hypertension?

- Superimposed preeclampsia - Abruptio placenta (smoking increases this risk) - Fetal growth restriction (correlates to severity of HTN) - Preterm delivery

Meconium aspiration syndrome (MAS) is prevented via...

Amnioinfusion intrapartum and DeLee suction post-partum

What are risk factors for abruptio placentae?

- Trauma (shearing, car accident) - Previous history of abruption - Preeclampsia (and chronic HTN) - Smoking - Cocaine - High parity

3 Signs of Placental Separation

- Typically w/in 5 min of delivery up to 30 mins - Uterus globular and firm 1. Sudden gush of blood 2. Uterus rises in the abdomen due to placental descent 3. Umbilical cord protrudes (laxity) farther out of the vagina

What is the clinical presentation of placental abruption?

- Vaginal bleeding (maternal and fetal blood present) - Constant, severe abdominal pain - Irritable, tender, hypertonic uterus - Evidence of fetal distress - Disseminated Intravascular coagulation DIC

What can cause false-positive nitrazine test?

- Vaginal infections with Trichomonas vaginalis - Blood - Semen (ask pt when last time they had intercourse)

cell-free fetal DNA (cfDNA) is performed after ______ weeks. What does it screen for?

- after 10 weeks - SCREENING test that requires invasive diagnostic testing if abnl - Take sample of maternal blood to retrieve fetal DNA, covered by insurance in AMA pregnancies (otherwise about $200-300) Tests for: Tri 21 (sensitivity 98-99%), 18, 13 and sex chromo aneuploides

Ovarian Germ Cell Tumors: Yolk Sac Tumor - characteristics - biomarkers - tx

- arises from extraembryonic tissue - 1/4 of malignant GCTs, most aggressive - median age is 23 - Schiller-Duval bodies - AFP is tumor marker

Postpartum Depression

- begins within 3 to 6 months after childbirth; 8-15% develop it in 2-3 months PP - up to 70% recurrence - Sx: same as major depression (SIGECAPS); symptomatic for months → years, typically with gradual improvement over 6 mo PP Tx SSRI and CBT

Common causes of nipple discharge?

- cancer - papillona - duct ectasia - infection With galactorrhea: - prolactinoma - pregnancy - hypothyroid - medications

Breast engorgement: Presentation Treatment

- common on days 2 to 4 postpartum, rarely persists for >24h - b/l painful, firm, globally swollen breasts - puerperal fever TX - sports bra, 24 on demand feeding, ice

Ovarian Germ Cell Tumors: Teratoma - characteristics - subtypes - biomarkers

- contains tissue form ecto, meso and endoderm: hair, teeth, skin, sebum, bone Immature - malignant teratoma Mature aka dermoid (95% of teratomas, most common germ cell tumor overall) - solid and or cystic, benign - can cause torsion of > 5cm Stroma ovarii - benign - mostly throid tissue --> may cause hyperthyroid Carcinoid (rare)

List the benefits of COCPs.

- decreases a woman's risk of ovarian and uterine cancer - decreases amount of bleeding --> lower risk of anemia - decreases pain (dysmenorrhea) - regulates cycles - reduces risk of PID (thickens mucus), fibrocystic breast changes, ovarian cysts, ectopic pregnancy, osteoporosis, acne, hirsutism

postpartum psychosis

- delusions (fixed false beliefs) - hallucinations (AVH) - extreme mental disorganization - can have pds of lucidity - 0.1-0.4% - peak onset 10-14 d PP but can start months later - often sx for 6 months RFs - Hx or FHx mental illness - young - primiparity Tx - hospitalization - pharm

PMB Risk factors for endometrial hyperplasia? How do you treat it?

- estrogen producing ovarian or adrenal tumors - exogenous hormones - obesity - conversion of androgens to estradiol in adipose tissue Tx Without atypia, manage with progestin and monitoring With atypia, 30% will have underlying cx so hysteroscopy and D&C to r/o malignancy prior to performing hysterectomy

List causes of polyhydramnios.

- fetal malformation: anencephaly, esophageal atresia (can't swallow fluid) - genetic d/o - maternal DM - multiple gestation - fetal anemia - viruses

Suspicious findings (breast cancer) of mass palpated on exam?

- fixed, hard, irregular mass - mass > 2cm

STI Testing

- h/o multiple partners, or partner has multiples - partner has STI - h/o STI annual screening for all sexually active woman under 25 - h/o dev disabilities - IVDU - in jail HIV - annual testing for those seeking treatment for STIs or with RFs: - h/o multiple sex partner HIV pos - IVDU/prostitute - partner is man that has sex with men transfused btwn 1978-1985 - with recurrent genital disease - have invasive cervical cx prego or planning to become - in jail

Laser Therapy: What is it? When is it appropriate?

- high E photon beam generates heat at impact and vaporizes tissue - causes less tissue destruction than the other methods - expensive but can use for excision or ablation of CIN

Risk assoc with cholecystitis in pregnancy? What is the mgmt of cholecystitis in pregnancy?

- high risk of preterm labor medical mgmt unless pancreatitis or bile duct obstruction then cholecystectomy

TOXOPLASMOSIS: How is it transmitted? Maternal sx? Classic Triad of Infected Newborn

- infected meat (cyst form) - cat feces (oocyst) - contaminated fruits or veggies Flu-like: fever, malaise, arthralgias but mostly Asx Infection confers immunity, thus infx prior to pregnancy makes vertical transmission practically 0 1. Hydrocephalus 2. Intracranial calcifications 3. Chorioretinitis Note: the later in gestation the mom is infected, the more likely transmission to the fetus

what is ICSI - intracytoplasmic sperm injection?

- inject spermatozoan into oocyte cytoplasm - not influenced by the cause of the abnl sperm - 25-30% cycle success rate

Adenocarcinoma of the Cervix

- mainly arise from endocervix - cervical cone bx with neg margins is reguired for microinvasive dz bc skip lesions are common - screen DES exposed women w/ annual paps Tx - similar to SCC - radical hys + lymphadenectomy for stage IB or IIA - vaginectomy if vagina involved Recurrence - most DES-related clear cell carcinomas recur at or < 3y - pulm and supraclavicular mets common - annual CXR

Is a backup method required for IUD or implants? If so, for how long?

- never need backup method after insertion of copper IUD -condom use for 7 days after insertion of the LNG-IUD or contraceptive implant... unless inserted immediately after surgical abortion, within 21 days of childbirth, upon transition from another reliable contraceptive method, within the first 7 days since menstrual bleeding started for the LNG-IUD, or within the first 5 days since menstrual bleeding started for the implant

Cryotherapy What is it? When is it appropriate? Complications

- outpt procedure with probe cooled with N2O to -70F to ablate lesions - appropriate for low-grade lesions only, must be completely visible on colpo Complications: - profuse watery discharge - cervical stenosis with small incr in PTL

Facts and Epidemiology of Cervical Cancer

- peak incidence ~50, more prevalent in African American and urban Hispanic women than Caucasian women - African American women mortality rates 2x higher than Caucasian women - 3rd most freq cx of female genital tract - 80% are squamous cell carcinomas - 80% of which are caused by HPV 16 &18 - 20% adenocarcinomas arising from endocervical canal and glands; fals neg rate with pap is 80%! assoc with maternal DES exposure and oncogenic strains of HPV - lifetime risk in US <1% - staged clinically not surgically - similar survival rates if dx in pregnancy -

List contraindications to an operative delivery.

- presenting part is not engaged - position of head is not known - membranes intact - cervix not fully dilated - CPD - prior scalp sampling is contraindicated for vacuum.

What are the major contraindications for performing a CST?

- preterm labor pts at high risk of delivery - premature rupture of membranes (PROM) - h/o extensive uterine surgery or prior C/S - known placenta previa

List contraindications to TOLAC.

- prior classical or T shaped incision or other transmyometrial uterine surgery - contracted pelvis - condition that precludes vaginal delivery - in a setting not equipped to perform emergent CD

Ovarian Sex Cord-Stromal Tumors: Characteristics 2 Main Types Tx

- rare (1% of malignant ovarian tumors) - arise from the sex cords and specialized stroma of the embryonic gonads (before they differentiate into ovaries or testes) - some secrete estrogen or testosterone - behave as low grade malignancies - older women 40-70 yo Granulosa Cell Tumor (most common type) - secretes estrogen (feminization, precocious puberty, menorrhagia, PMB) - Call-Exner bodies (eosinophilic bodies surrounded by granulosa cells) - Inhibin tumor marker Sertoli-Leydig Cell Tumor - secretes testosterone (virilization. hirsutism, menstrual disorders) - testosterone is tumor marker Tx TAH-BSO if done having children, otherwise unilateral salpingo-oopherectomy (young with low stage/grade) Chemo and rad if stage II-IV or recurrence

Phyllodes tumor What age group? Describe their growth pattern.

- rare, 1% - most phyllodes tumors are benign, some are malignant, some borderline so excise - develop in 40s (vs fibroadenoma in 30s) - tend to grow quickly, within a period of weeks or months, to a size of 2-3 cm or larger

Ovarian Germ Cell Tumors: Choriocarcinoma - characteristics - biomarkers

- rare, malignant, < 20 yo - less than 2% of malignant GCTs - more common to be of placental than ovarian origin - B-hCG is the tumor marker

Maternity/Postpartum Blues

- self-limited, mild mood disturbance - 50% of child bearing women - begins within 3 to 6 days PP and persists up to 2 weeks - Milder sx of depression - possibly related to progesterone withdrawl - Tx: Supportive care, reassurance and monitor

Poor prognostic indicators for ovarian cancer?

- short disease-free interval - mucinous or clear cell tumor - multiple disease sites - high/rising CA-125

Signs and symptoms of vaginal cancer?

- ulcerated, exophytic mass - abnl vaginal bleeding - foul smelling vag discharge - pain in advanced cases

Clinical Criteria for Dx of Menopause

-1 year of amenorrhea ± menopausal symptoms -Exclusion of other causes of amenorrhea: pregnancy, hypothyroid (Negative hCG, normal TSH) -FSH >30 mIU/ml

What does the American Cancer Society recommend about clinical breast exams? ACOG? And USPSTF? Describe technique for CBE.

-ACS: evidence unclear, no recommendations - ACOG: CBE every 1-3 yrs age 25-39, then annually -USPSTF: not enough evidence Most not recommending self breast exams anymore CBE Inspect for skin changes and symmetry noting any puckering with leaning fwd Rule of 3s: - 3 min exam - 3 middle fingers - palpating at 3 pressures (superficial, intermediate, deep) Apply gentle pressure to nipple to assess for discharge Examine axillary nodes (anterior, medial, posterior walls) and supraclavicular nodes

What is the main difference between a fine need aspiration and an excisional biopsy?

-FNA is a cellular sample -excisional biopsy is a core tissue sample

What is included in the work up for female infertility?

-FSH + estradiol -AMH (Anti-Mullerian hormone, prolactin + RPL panel) -TSH -Prenatal labs/Vit D/CBC/Prolactin -fasting blood sugars -free + total testosterone -chromosome analysis/genetic screening -US -SIS or HSG

What's included in the work up for male infertility?

-FSH and estradiol -TSH -Semen analysis -Chromosome analysis

List risk factors for ovarian cancer.

-Familial ovarian cancer syndrome -family history/personal history -Increasing age -early menarche -infertility -nulliparity - late childbearing -late onset menopause -obesity: BMI >30 -personal or family history of breast cancer - caucasian - maybe: talcum powder, high fat diet, fertility drugs

s/sx chlamydia

-Green/yellow discharge -fever, pelvic pain with PID -Cervical Motion Tenderness -Dysuria -Friable cervix (bleeding with intercourse)

Risk Factors for Fragility Fracture

-History of Prior Fracture -Family history of osteoporosis -Asian, Caucasian race -Low body weight/BMI -Smoking -Alcoholism -Estrogen deficiency: Early menopause, BSO -Immobility, sedentary

How can we treat infertility caused by sperm issues?

-IVF + ICSI (Intracytoplasmic Sperm Injection) -PICSI (Physiological Intracystoplasmic Sperm Injection) -sperm donor

Health Benefits of ERT

-Improves vasomotor and vaginal symptoms -Decreased risk of osteoporosis and related fractures (but bisphosphonates 1st line given risks) -Lower risk of colo-rectal cancer, low-impact fracture -Improved HDL and LDL levels

Criteria for Dx of Climactera/Peri-menopause

-Irregular menses +/- menopausal symptoms -Exclusion of other possible disorders (EMB or TSH) -FSH > 15 mIU/ml

Breast Cysts What is a breast cyst? Who gets them? What are the usually characteristics of a breast cyst? Does a breast cyst change with the menstrual cycle? Does having breast cysts increase your risk of cancer? How do we tx/prevent breast cysts?

-It's a pocket of fluid d/t hormonal stimulation -women age 35-50 yo -smooth, mobile, round or oval, distinct edges -usually unilateral, but can be bilateral -pain and tenderness will be present in the area of the lump -increase in size and tenderness just before period -decrease in size and resolution of sx after period No! But it may interfere with recognition of new lumps or abnormal changes. -Vit E + B6 can relieve pain associated with cysts -caffeine reduction can reduce prevalence -note that recurrence is very common

Cautions of using herbal/naturopathic remedies

-Not proven -Not FDA reg -Drug-botanical interactions do occur

What are common causes of female infertility?

-PCOS -Hypothalamic amenorrhea -Poor egg quality or decrease ovarian reserve -Endocrine: Elevated prolactin, thyroid issues -Stress or fatigue

What things can lead to pelvic/fallopian tube adhesions?

-PID -surg -endometriosis -old chlamydia infection

Alternatives to HRT

-SNRIs (serotonin & norepinephrine reuptake inhibitors) -SSRI's: Fluoxetine, paroxetine, sertraline -Herbal/Naturopathic remedies for hot flushes : Wild yam extract, Black cohosh, Dietary phytoestrogen (soy) supplements -Other therapies limited by side effects: Gabapentin, clonidine

What're correctable causes of low sperm count?

-Scrotal Heat: illness, hot tubs, laptop, cellphone -Cycling: bike or motorcycles -Change medications: gout, anti-hypertensives, H2 blockers (Cimetidine, Ranitidine) -Excessive alcohol or marijuana -Takes 3 month minimum to see changes

Climactera aka Peri-Menopause: Defined Typical duration

-The transition from the late reproductive into the post-menopausal stage of life -Period of waning ovarian function -Typically 1-2 year period before and after menopause -Symptoms last up to a decade!

Vasomotor Symptoms (VMS): Physiology Epidemiology Duration

-Vasomotor instability clinically described as "hot flushes" -Disruption of hypothalamic temperature regulation Experienced by 50% of postmenopausal women Prevalence and severity vary by patient Evidence of ethnic predisposition Symptoms typically last 2-4 yrs, but up to a decade for some! (impact QOL)

What're characteristics of benign nipple discharge?

-bilateral -multi-ductal -occurs with breast manipulation - milky think galactorrhea --> hypothyroidism, prolactinoma, meds **nipple discharge is usually benign in nature

Contraindications to depo shot?

-breast cx - AUB - prego - liver dz - low BMD

What's considered protective against breast cancer?

-breast feeding -multiple parity -exercise

Gynecomastia What causes gynecomastia? What drug classes commonly cause gynecomastia? How do we make the diagnosis of gynecomastia?

-drugs/medications -hyperthyroidism -liver or kidney disease -hypogonadism -testicular tumors -aging -antibiotics (ioniazid, ketoconazole, metronidazole) -anti-ulcer drugs -cancer chemotherapeutic drugs -cardiovascular drugs (ACEi, amiodarone, CCB, digitoxin) -hormones -psychoactive drugs A palpable mass on examination of at least 0.5 cm in diameter

Most common malignant germ cell tumor?

-dysgermiomas (50%) -immature teratoma -endodermal sinus (yolk sack) tumors

Identify causes of high MSAFP.

-incorrect dates (underestimation of GA as it rises at 13 weeks and peaks at 32 weeks) - NTDs - abd wall defects (gastroschisis and omphalocele) - fetal death - molar preg - high maternal weight

Fibrocystic Breast Changes What are fibrocystic breast changes? How do we tx fibrocystic breast changes?

-it is not a disease, it is a pathologic description - FC breast changes can cause pain, excessive nodularity, engorgement and cysts -common with hormone fluctuations, classically cyclic b/l breast pain - no incr risk of cx Tx good bra support, minimize caffeine & salt, daily exercise, low fat diet, Vit E 100 IUs daily or Vit B6 100 mg daily, evening primrose oil capsules 1000-3000 mg qd or other omega-3s, NSAIDs, moist heat

Describe noncyclic breast pain.

-most common in women 30 to 50 years of age -may occur in only one breast -often described as a sharp, burning pain that occurs in one area of a breast -may be caused by a fibroadenoma or a cyst -if the cause of noncyclic pain can be found, treating the cause may relieve the pain.

Describe cyclic breast pain.

-most common type of breast pain -caused by the normal monthly changes in hormones -pain usually occurs in both breasts -described as a heaviness or soreness that radiates to the armpit and arm -pain is usually most severe before a menstrual period in the luteal phase and is often relieved by menses onset -more often in younger women -usually doesn't require treatment Note: OCPs can cause mastalgia

Surgical Staging for Ovarian Cancer

-peritoneal washings (cytology) -Total abdominal hysterectomy (TAH) -Bilateral salpingo-oophorectomy (BS) -Omentomectomy - Appendectomy -Sampling of pelvic and para-aortic lymph nodes to achieve optimal cytoreduction debulking is done --> remove all sites of primary or mets > 1cm

What're the pros and cons of a breast MRI?

-pros: no radiation, very sensitive -cons: contrast, lower specificity, inferior for calcifications, costly, takes a long time, no metal, claustrophobia

Risk factors for developing breast cancer?

-radiation exposure -obesity (postmenopausal) -early menarche, late menopause - nulliparity -smoking -alcohol -HRT > 5 years postmenopausal -family history/genetic mutations (BRCA, CHEK2) -previous breast cancer or high-risk lesions (ADH, ALH, LCIS)

What imaging can be done in the work-up or nipple discharge?

-retroareolar US + mammography -ductography -breast MRI

What are characteristics of a cancerous mass?

-spiculated -soft tissue -architectural distortion -pleomorphic microcalcifications in a ductal distribution

SEs of ERT?

-spotting -breast tendernesss

How do we treat infertility caused by pelvic adhesions?

-surgery is an option and is most successful in pts < 34; success rate is only about 50% -surgery can actually just lead to more lesions and typically adhesions re-form -BEST OPTION IS IVF

Fibroadenomas What are fibroadenomas? Who gets them? What are the usual characteristics of a fibroadenoma?

-tumors made up of both glandular breast tissue and stromal (connective tissue) -firm, noncancerous tumor of a gland -age 15-35 y/o -often stimulated by estrogen and progesterone Firm, round, painless, rubbery and mobile

What is the classic presenting triad of fallopian tube cancer?

-vaginal bleeding -pelvic pain -watery discharge In any women with PMB and watery discharge think fallopian tube cancer Note: - usually asymptomatic, found incidentally with lap

What is the failure rate of emergency contraception using copper IUD?

0.1%

What are the benefits of steroids in preterm labor?

1) Accelerates fetal lung maturity - decreases risk of RDS 2) Reduces intraventricular hemorrhage

How can you evaluate the cause of postpartum hemorrhage? What should be inspected?

1) Carefully explore the uterine cavity. Make sure all placental parts have been delivered and the uterus is intact. 2) Inspect the cervix and vagina for trauma / lacerations

Pharmacotherapy Stress Urinary Incontinence: Estrogen Vaginal cream

1) Dose: 0.5 to 1 g two to three times weekly 2) Mechanism of action: alpha-adrenergic stimulation urethra estrogen dependent Modest role (okay choice if estrogen deficiency) 3) Usually need combo therapy

Pharmacotherapy Urge Urinary Incontinence: Beta adrenergic agonist: Mirabegron (Myrbetriq)

1) Dose: 25 to 50 mg/day 2) Mechanism of action: beta-3 adrenergic agonist = inhibition of detrusor muscle 3) ADRs: increased blood pressure, angioedema 4) Note: mirabegron a moderate 2D6 inhibitor

Contraindications to breastfeeding?

1) HIV 2) galactosemia in baby 3) HSV if lesions on breast 4) HBV until neonate has received HBV vaccine 5) Acute maternal disease if absent in infant (e.g., active tuberculosis , sepsis) 6) Breast cancer receiving tx 7) Substance abuse 8) Meds (e.g. antineoplastic: cyclophosphamide, doxorubicin) 9) Radiotherapy - CT w/ contrast temp stop then resume later timing dependent on dose

Functional Incontinence: Defined Causes

1) Inability to hold urine d/t reasons other than neurologic or urinary tract dysfunction (e.g. immobility limits timing) 2) Linked to primary disease process, Dementia, Delirium, Psychiatric disorders, Urinary tract infection, Mobility deficits Meds (e.g. Anticholinergics and alpha-adrenergics)

Nonpharmacologic Management: First line treatment

1) Lifestyle modifications - weight loss - dietary: decr caffeine and alcohol intake - treat constipation - quit smoking a) Dietary and fluid restrictions (elderly already don't drink as much fluids as they should---encourage hydration, but not right before bed), weight loss if obese 2) Scheduling/prompting voiding a) Every 2 hours, etc 3) Pelvic floor muscle rehabilitiation a) Kegel exercises 4) Anti-incontinence devices a) Catheters 5) Supportive interventions a) Absorbent pads, underwear

What are the risk factors for shoulder dystocia?

1) Maternal factors leading to *increased fetal birth weight*: - Obesity - Multiparity - Gestational diabetes 2) Fetal: post-term pregnancy > 42 weeks 3) Intrapartum: prolonged first and/or second stage of labor.

Stress Incontinence: Define - Exs Causes Quantity of loss? Nocturia? Enuresis? Risk Factors

1) Occurs when intra-abdominal pressure is greater than intra-urethral pressure a) Examples: exercise, running, coughing, sneezing 2) urethral hypermobility +/- intrinsic sphincter deficiency (maintains enough resting tone but not with certain manuevers) 3) Amount lost per episode = small 4) Nocturia (urination many times at night) and enuresis (involuntary urination) = rare 5) Risk factors: pregnancy, vaginal delivery, menopause, obesity, increased age, prostatectomy or TURP (trans urethral resection of prostate)

Pharmacotherapy Urge Incontinence: Anticholinergics: Mechanism of Action/Considerations

1) Oxybutynin: blocks M1, M2, M3 receptors M1 = impacts cognition, dry mouth, constipation M2 = CNS M3 = smooth muscle 2) Tolterodine: relatively selective for M2 and M3, dose adjusted for hepatic impairment, CYP2D6, 3A4 inhibitor 3) Fesoterodine: prodrug, metabolized (2D6, 3A4) to tolterodine 4) Tropsium: relatively selective for M2 & M3 5) Darfenacin: M3 antagonist = costs more, might not be on formulary 6) Solfenacin: M3 antagonist, note: QT prolongation 7) Major issue is tolerability (*none are bladder specific*) a) Adverse effects: dry mouth, constipation, blurred vision, flushing Hot as a Hare (increased temp, decreased sweating) Mad as a Hatter (confusion, delirium) Red as a Beet (Flushed face, tachycardia) Dry as a Bone (Decreased thirst, dry mouth) b) Oxybutynin immediate release = over 7.5 mg/day least tolerated - want to avoid 8) 4 full weeks for full effect (best with lifestyle modification)

Overflow Incontinence: Defined Cause Amount of urine loss? Nocturia? Enuresis? Risk Factors Tx

1) Small urine leaks d/t urethral overactivity and/or bladder underactivity 2) Bladder cannot empty fully 3) Bladder(detrusor) underactivity = neurogenic bladder (some causes?) Spinal cord injury, MS, Parkinson's disease 4) Urethral overactivity = long term obstruction d/t prostate cancer or benign prostate hypertrophy in males or cystocele/surgical overcorrection of stress urinary incontinence in females 5) What type of medications may cause this? Anticholinergics, alpha-adrenergics (Sudafed)

Types of Urinary Incontinence: Brief Overview

1) Stress urinary incontinence Urethral underactivity---not adequate closure of urethra 2) Urge urinary incontinence Bladder overactivity----bladder wants to contract more often than it should 3) Overflow urinary incontinence Urethral sphincter overactivity (urine not allowed to leave bladder) and/or bladder underactivity---bladder just fills up without emptying, no detrusor contraction 4) Functional urinary incontinence 5) Mixed (stress urinary incontinence + urge urinary incontinence) 6) Medication-induced urinary incontinence

What is the treatment for Hyperemesis gravidarum?

1) Vitamin B6 w/ Doxylamine 2) IV hydration, thiamine replacement, Antiemetics 3) Parenteral nutrition

What are the 5 indications for vaginal pessaries?

1. Most effective with mild to moderate prolapse 2. Alternative for surgery esp useful in elderly who often have contraindications to surgery 3. Symptomatic pelvic relaxation 4. Temporary use before surgery 5. Cervical prolapse occurring during pregnancy

FIGO staging for cervical cancer

1) cervix 2) upper 2/3 of the vagina 3) lower 1/3 of the vagina 4a) bladder or rectum(cystoscopy and sigmoidoscopy) 4b) beyond

What are the two components of a modified BPP (mBPP)?

1. NST 2. AFI

What are the clinical criteria for Antiphospholipid syndrome?

1. Arterial and venous thrombosis 2. Pregnancy morbidity - At least one fetal death at or > 10 weeks - At least one preterm birth < 34 weeks - At least 3 consecutive spontaneous abortions < 10 weeks 3. Puerperium - Fe deficiency - Acute blood loss

Possible Etiologies of Recurrent Abortions

1. Balanced translocation in parents that results in unbalanced translocation in the fetus 2. Anticardiolipin and antiphosphatidyl serine antibodies 3. Lupus Anticoagulant: antiphospholipid workup 4. Factor V Leiden

How To: Deliver the Placenta

1. MUST occur AFTER placental separation or can cause inversion of uterus 2. Pressure applied to uterus body while umbilical cord held taut 3. Uterus lifted cephalad with the abdominal hand repeatedly until placenta reaches the introitus 4. At introitus, uterus pressure stopped 5. Placenta is gently lifted away from the introitus 6. Maternal surface of the placenta should be examined to ensure that no placental fragments are left in the uterus

Urge Incontinence: Defined Cause Amount of urine loss? Nocturia? Enuresis? Risk Factors Tx

1) involuntary urine leakage with or preceded by urgency 2) Overactive bladder syndrome = might not involve loss of urine; unopposed detrusor contraction 3) D/t overactive bladder muscle a) Frequency = over 8 times/day b) Urgency = sudden need to urinate (difficult to delay) 4) Amount lost per episode: small or large (can empty the bladder) 5) Nocturia and enuresis = common 6) Risk factors: increased age, bladder outlet obstruction (prostate cancer or benign prostatic hypertrophy)----bladder becomes stretched & detrusor muscles become hyperactive, neurological diagnosis

What are the features of secondary syphilis ? How long after primary infection with syphilis can one get secondary infection?

1. Maculopapular rash on soles of feet and palms 2. Sore throat 3. Fever 4. Headache 5. Arthralgias shows up 10 weeks after primary

POP management

1. If pt is not bothered by this, then no treatment necessary 2. Mild prolapse (grade 1-2) = pelvic floor exercises, physical therapy, and behavioral modifications. 3. Moderate prolapse (grade 2-3) or poor surgical candidates = a vaginal pessary 4. Severe prolapse (grade 3-4) = surgery

What are four possible reasons for why a pregnant patient would have AUB?

1. Implantation bleeding 2. Exercised induced bleeding 3. Spontaneous abortion (miscarriage) 4. Ectopic pregnancy

Abortion is defined as...

1. Intentional or unintentional termination of pregnancy a. < 20 weeks or b. weight of < 500 g

What is the treatment for uterine atony causing PPH?

1. Massage uterus 2. Give additional dilute oxytocin (causes contraction) 3. Methergine / Prostaglandin F2a / Misoprostol 4. Decrease uterine artery pulse pressure. 5. Hysterectomy

List 4 ways to assess estrogen levels.

1. Lab values of estradiol: but these vary across the cycle so they may not always be informative 2. Presence of breasts/tanner stage: in evaluation of primary amenorrhea 3. US: can evaluate the thickness of the endometrial lining. 4. Progesterone withdrawal test: a - give pt progesterone and then stop. If they bleed, their estrogen status must have been normal. b - if they did not bleed you can give estrogen and progesterone. If they still do not bleed the problem is with outflow, probably not with estrogen.

The first stage of labor consists of what phases? What marks the start and end?

1. Latent phase: Onset of labor --> 4-6 cm cervical dilation (depends on the source referenced) 2. Active phase = Rapid dilationfromm dilation --> 10 cm dilation

What recommendations does the American Cancer Society make for breast MRIs?

1. Lifetime risk of breast cancer >20% 2. Hx radiation treatment to the chest prior to age 30 yo

What complications arise with syphilis infection in pregnancy?

1. Maternal tertiary infection 2. Still birth 3. Neonatal death 4. Congenital Syphilis

Bone mass decreases ≅ ___% each year following menopause.

1-2

What do *pap smear results* report?

1. *Description of specimen type*: conventional or liquid based 2. *Description of adequacy*: satisfactory, unsatisfactory, specimen rejected, endocervical cells not present 3. *General category (optional)*: negative, epithelial cell abnormality 4. *Interpretation/Result*: Negative for intraepithelial lesion/malignancy, organisms, reactive changes, atrophy 5. *Description of ancillary testing*: HPV, Autopap 6. *Educational notes (optional)*

What are the 7 cardinal movements of labor?

1. *Engagement* - descent of the biparietal diameter (BPD) to the pelvic inlet (zero station) 2. *Descent* - fetal head passing down into the pelvis 3. *Flexion* - fetus chin-to-chest position; presents smallest diameter of the fetal head to the birth canal 4. *Internal rotation* - fetal occiput gradually rotating toward the pubic symphysis 5. *Extension* - occiput moves to the fetal back after descent to maternal vulva allowing curving under and past the pubic symphysis 6. *External rotation* aka *Restitution* - occurs after delivery of the head, when the fetus resumes its normal "face-forward" position 7. *Expulsion* - delivery of the anterior shoulder --> posterior shoulder --> rest of body follows

List the 2 main types of Cesarean delivery. Under what circumstances would you perform the 2nd type?

1. *Low-Transverse (LTCS i.e. pfannenstiel) - horizontal incision in the lower uterine segment 2. Classical - vertical incision made in the contractile portion of uterine corpus - indicated when: lower uterine segment is not well-developed (extreme prematurity ) or transverse lie with back down or placenta previa

What are the laboratory criteria for antiphospholipid syndrome?

1. *Lupus anticoagulant* 2. *Medium to high titers of anticardiolipin antibody* 3. *Anti-B2 glycoprotein* (each of these must be present in plasma, *on at least two occasions > 12 weeks apart*)

Shoulder Dystocia Treatment

1. *McRoberts maneuver*: Maternal thighs are sharply flexed against maternal abdomen. This decreases the angle between the sacrum and spine and may dislodge fetal shoulder 2. *Suprapubic pressure* on maternal abdomen 3. *Woods corkscrew maneuver*: Pressure is applied against *scapula of posterior shoulder* to rotate the posterior shoulder and "unscrew" the anterrior shoulder 4. *Posterior shoulder delivery*: Hand is inserted into vagina and posterior arm is pulled across chest, delivering posterior shoulder and displacing anterior shoulder from behind pubic symphysis . 5. *Break clavicle or cut through symphysis* 6. *Zavanelli maneuver*: If the above measures do not work, the fetal head can be returned to the uterus. At this point, a C-section can be performed.

Recurrent Abortion

1. 2 or more successive abortions occuring prior to 20 weeks of Gestation 2. Women with 2 successive abortions have a 25-30% risk of recurrence.

Ddx for Trimester 3 bleeding

1. Abruptio placenta 2. Placenta previa 3. Rupture of vasa previa 4. Uterine Rupture

AUB-O 1. What is ovulatory dysfunction? 2. What causes bleeding in ovulatory dysfunction? 3. Possible findings on PE 4. Possible findings on imaging 5. Diagnosis 6. Management

1. Absence of or infrequent ovulation (hence no progesterone made bc no corpus luteum) - amenorrhea to irregular heavy menstrual periods 2. Endometrial hypertrophy which outgrows the blood supply and causes bleeding 3. Evidence of androgen excess may be present, pelvic exam will be normal 4. Endometrium thickened - Polycystic ovaries: often described as a "string of pearls" appearance along the periphery of the ovary 5. AUB-O is essentially a diagnosis of exclusion; Polycystic ovary syndrome (PCOS) may be present. 6. Medical Management: - Hormonal: Oral contraceptive pills, progestin IUD, medroxyprogesterone acetate. - Non-hormonal: metformin ,occasionally used for PCOS patients. Surgical Management: not considered initial Tx - Endometrial ablation -Hysterectomy

What are the two classes of DUB (dysfunctional uterine bleeding)?In terms of prevalence, what are the percentages that are associated with each? In terms of pathophysiology, what occurs during each that leads to DUB? What are the top six pathologies responsible for causing anovulatory DUB?

1. Anovulatory - 90% 2. Ovulatory - 10% Anovulatory Primary ovarian follicle develops, which leads to an increase in systemic estrogen, which causes the endometrial lining to thicken. However, the primary follicle does not release an egg and form into the corpus luteum, which means that there is no progesterone released into systemic circulation to maintain the lining. Essentially there is unopposed estrogen, which causes the lining to slough off at random intervals, and in heavy amounts. Ovulatory Prolonged progesterone release from the corpus luteum causes irregular shedding of the endometrium 1. PCOS 2. Obesity 3. Perimenarche (HPO dysfunction) 4. Perimenopause (HPO dysfunction) 5. Hypothyroidism 6. Hyperprolactinemia

What are the complications of in utero toxoplasmosis infection?

1. Miscarriage 2. Preterm labor 3. Death/ Still birth

PALM COEIN 1. What is adenomyosis? 2. Si/Sx 3/ Dx 4. Tx

1. Benign endometrial glands and stroma present WITHIN the myometrium. 30% of women usually parous in their 30-50s - Sometimes described as endometriosis of the myometrium, but is a distinct diagnosis. 2. Heavy , irregular menstrual bleeding and dysmenorrhea (painful periods); postcoital bleeding. Mildly globally/symmetrically enlarged globular, boggy uterus Uterus may be tender on exam . 3. . Only diagnosed after hysterectomy! Definitive diagnosis is histologic evaluation of the uterus (i.e. hysterectomy). Biopsy of the uterus is NOT helpful in establishing the diagnosis. And Although heterogeneous echogenicity of the myometrium is often seen on sonogram and has a high sensitivity, it has POOR positive and negative predictive values. MRI may aid in the diagnosis, but is rarely utilized. There are NO pelvic sonogram FINDINGS that are considered PATHOGNOMONIC for the diagnosis. 4. Hysterectomy is definitive treatment. Medications that will control bleeding by thinning the endometrium for those who desire fertility and control pain. Nevertheless, no proven medical tx. NSAIDs, OCPs, GnRH agonists Endometrial ablation and uterine artery embolization

PALM COEIN 1. What is a leiomyoma aka fibroid? 2. Demographics 3. Risk factors for leiomyomas 4. Si/Sx 5. Dx 6. Tx

1. Benign smooth muscle tumor arising from the myometrium that are hormonally responsive 2. AA women 3. Early menarche, hormonal medications, and family history 4. Asx in > 50% heavy irregular menses with possible anemia, pelvic pressure is common and pain is unusual Enlarged uterus on exam with irregular contour and non-tender (unless a fibroid is degenerating). Firm, mobile, midline. many are asymptomatic Note: Size, number, and location of myomas can correlate with symptoms. 5. hysterosonogram is considered diagnostic. - can be seen on XR, MRI, CT - can also do direct visualization via hysteroscopy or abdominal surgery and histologic confirmation 6. Tx None indicated if asx, likely shrink with menopause. If sx or bleeding not controlled (Hct falls), treat typically with meds first. Note: May grow in size during pregnancy. Surgical: 1. Hysterectomy is the definitive treatment. 2. Myomectomy an important options for patients desiring to preserve their fertility. 3. Endometrial Ablation if completed child-bearing and fibroids are small < 4 cm 4. Uterine artery ebolization if want to preserve uterus but not fertility Medical: 1. GnRH agonists (leuprolide acetate): will cause atrophy of the endometrium and shrinkage of the fibroids. 2. Progestins (medroxyprogesterone acetate, progestin IUD): causes atrophy of the endometrium. 3. Oral contraceptive pills: progestin effect on the endometrium. is the definitive treatment. , treat

Medications used for Induction of Labor and their function.

1. Cervical Ripening - if Bishop score (6 or under) indicates an unfavorable cervix can use: A) *Mechanical Cervical Dilators*: - hygroscopic dilators: expand when placed in cervix - osmotic dilators (Laminaria japonicum: seaweed component) - Foley catheters: rest with traction on internal os OR B) Synthetic *Prostaglandins* - E1 (PGE1) i.e. misoprostol (PV or PO recommended) - E2 (PGE2) i.e. dinoprostone (cervical gel or vaginal insert) - vaginal PGEs increase risk of uterine tachysystole (>5 contractions/10 mins averaged over 30 mins) 2. Uterine Contraction Stimulation *Pitocin* (synthetic oxytocin)- stimulates uterine contraction, given IV, no more than few hrs

What is the management of placental abruption?

1. Correct shock (IVF, PRBC, FFP, cryoprecipitate, platelets) 2. Maternal oxygen 3. Expectant management or induction of labor. 4. If fetal distress, perform C-section

What are 3 types of space-occupying pessaries?

1. Cube pessary 2. Donut pessary 3. Gellhorn pessary

Predisposing factors for LGA babies

1. DIABETES! 2. Obesity 3. Post-dates 4. Multiparity w/ same partner 5. Advanced age 6. Previous LGA 7. Genetic or congenital disorders 8. Male sex

Standard Method of Delivery

1. Delivery of the Head via Modified Ritgen Maneuver - hand on occiput, other hand protects perineum - head will then restitute L or R 2. Check for Nuchal Cord - if tight cut between two clamps 3. Delivery of Shoulders - gentle downward traction on head to deliver anterior shoulder - gentle upward traction delivers posterior shoulder - maternal pushing delivers the rest but support the neck with one hand and the buttocks with the other - consider episiotomy in shoulder dystocia or operative deliveries 4. Clear the nasopharynx - bulb suction mouth then nares 5. Clamp and cut the cord - between two clamps 6. Delivery of the Placenta - typically in the next 5 mins you see a gush of blood, uterus rises and gets firm, cord lengthens - apply gentle steady downward traction on cord (watch for uterine inversion!) - fundal massage helps contract uterus and thus decreases blood loss - check for two umbilical arteries and one vein - sample of cord blood taken from placental side 7. Inspection for laceration repair - circumfrential cervix, vaginal walls, labia, perineum 8. Postdelivery Hemostasis

What post-operative care is needed after repair of a pelvic organ prolapse?

1. Early ambulation 2. Good bladder drainage and bowel function (may have catheter in place and be on stool softeners while pelvic floor heals). 3. Pelvic rest x4-6 weeks (nothing in the vagina during this time). 4. Avoid sudden and repetitive increases of intra-abdominal pressure, i.e. heavy lifting and big coughing.

Four parameters of the cervix examined during labor? One additional parameter to report?

1. Effacement (thinning aka length of cervix) 2. Consistency (soft? firm?) 3. Dilation 4. Position (w/ respect to fetal presenting part) 5. Station (the degree of descent of the presenting part in relation to the ischial spines)

Morbidity in PCOS (6 problems).

1. Endometrial hyperplasia and cancer 2. Infertility 3. Obesity/metabolic syndrome 4. Increased CVD risk 5. Increased diabetis risk 6. Increased NAFLD

Methods of Continuous Electronic Fetal Monitoring (EFM)

1. External (indirect) electronic FHR monitoring (more common): US transducer through the maternal abdominal wall 2. Internal electronic FHR monitoring: Bipolar spiral electrode attached to fetal scalp; detects peak R-wave voltage of the fetal ECG

What are the 3 signs of placental separation? You will be asked this on the wards!

1. Gush of blood from vagina 2. Umbilical cord lengthening 3. Fundus of the uterus rises and becomes firm (aka fundal rebound)

Management of Preterm labor:

1. Hydration 2. Tocolytic therapy 3. Corticosteroids 4. Amniocentesis - Assess fetal lung maturity

Stages 3 and 4 : if the cancer is progesterone receptor positive - _____% have 5 year survival If the cancer is progesterone receptor negative - ______% have a 5 year survival

70% ; 15-20%

What are the five components of a BPP?

1. NST - nl has mod variability in FHR (6-25 bpm change over 1 min) and reactive aka 2+ accelerations (15 bpm above base for at 15+ secs) over 20 mins 2. Breathing - at least 1 episode for 30+ secs over 30 mins 3. Movement - 3+ body or limb movements within 30 mins 4. Muscle Tone: 1+ episode of extension with return to flexion or opening and closing of hand 5. Amniotic fluid: single vertical pocket 2+ cm or Amniotic Fluid Index of 5+ cm Each is given a score of 0 (normal) or 2(abnormal) Normal score = 8+ Equivocal = 6/7 Abnormal = 4 or lower

What are the features of tertiary syphilis?

1. Neurosyphilis - dementia, epilepsy, psychoses , abnormal gait 2. CVS Syphilis - aortitis , aortic regurgitation, HF 3. Gummatous Syphilis - skin , bone lesions

What are the four broad categories of differentials that must be considered in non-prego patients with AUB? What do the first three of these have in relation to the DUB? What broad category of etiology should be suspected if the patient characterizes AUB as cyclical bleeding? What broad category of etiology should be suspected if the patient characterizes AUB as breakthrough bleeding? If a patient presents appearing malnourished and complaints of AUB, what should be at the top of the differential for this patient?

1. Organic (structural) 2. Systemic 3. Iatrogenic 4. DUB The first three must be ruled out before DUB can be considered the cause of AUB Cyclical bleeding = organic (structural) cause Breakthrough bleeding = iatrogenic cause Anorexia nervosa

PALM COEIN 1. What is a uterine polyp? 2. Risk factors 3. Demographics 4. Most common symptom? 5. Dx 6. Tx

1. Overgrowth of endometrial glands and stroma (can also arise from endocervical tissue) 2. Exogenous estrogen, tamoxifen, and obesity 3. Women of reproductive age and post-menopausal women; rare in adolescents 4. Intermenstrual bleeding. Note: Uterine size and appearance of cervix usually normal. - Bleeding usually irregular, but can sometimes present as heavy menstrual bleeding. - Pelvic cramping/pain usually not present. 5. Suspect on TVUS but Dx is confirmed by sonohysterogram - Biopsy or D&C for pathologic diagnosis and to rule out malignancy 6. Hysteroscopic resection - D&C and/or polypectomy

How can you ensure a Pt's pessary fits before they leave the office?

1. Pessary is visible at the introitus but will not descend past the hymen with bearing down. 2. Returns to prior position with relaxation. 3. Comfortable with walking, squatting, sitting, and bearing down.

2 main causes of ovarian amenorrhea and their hormone levels (FSH/LH/estradiol/other)?

1. Polycystic ovarian syndrome: -normal estradiol -low FSH -high LH -high androgens 2. Premature ovarian insufficiency/failure: -low estradiol -high FSH/LH So measure hormones if you suspect!

What are 3 types of support pessaries?

1. Ring pessary 2. Gehrung pessary 3. Hodge pessary

What are two classes of medications with the most evidence of effectiveness for PMS/PMDD? Lifestyle recs?

1. SSRIs 2. OCPs (monophasic - particularly with 4 d vs 7d placebo) NSAIDs for pain and spironolactone for fluid retention have also shown to be somewhat helpful GnRH agonist if no response and severe No FDA approved meds - stress reduction - exercise - vit E decr mastalgia

How is a vaginal mucosal ulceration or infection secondary to pessary treated?

1. Stop using pessary x2-3 weeks, but continue topical estrogen. 2. Antibiotics may be needed if large ulceration or does not heal. Then once pt starts using pessary again, talk to pt about increasing cleaning interval and leaving it out overnight.

What is to be done at the first prenatal visit?

1. Thorough H&P - including extra considerations: - LMP and menstrual history - OB hx - contraception use? h/o STIs? - PMH of asthma, DM, HTN, thyroid dz, cardiac dz, seizures, rubella - prior surgeries particularly abdominal - FHx: twin pregnancies, DM, HTN, bleeding d/o, other heritable d/o, cognitive delays, problems with anesthesia - ask about teratogenic meds - determine from hx if she is needs closure f/u (i.e. high rish OB pt) 2. Labs 3. Plan for genetic testing

What fetal investigations are performed when toxoplasmosis infection is suspected?

1. U/S scan for fetal anomalies 2. Amniocentesis PCR: After 15 weeks gestation - detects fetal infection

What studies/labs should you order if primary amenorrhea?

1. UPT 2. Abd US (uterus?) If absent --> Karyotype, serum Testosterone - XX with nl female T level = Mullerian agenesis - XY with male level testosterone = AIS If uterus present and no other anatomical abnormalities: - B-hCG - FSH (high = POF, low is hypothalamic, normal = secondary amenorrhea w/u)

What are 5 associated symptoms of pelvic floor dysfunction?

1. Urinary stress incontinence 2. Urinary retention 3. Defecatory dysfunction 4. Sexual dysfunction (from the discomfort) 5. Vaginal discharge or bleeding.

How do you screen for *HPV*?

1. Visual inspection of the cervix 2. Pap smears 3. *HPV HR DNA*: 13-14 of oncogenic subtypes 4. *HPV 16/18 Genotype testing*

What are the three components of FHR that a NST evaluates? When can you say a NST is reactive vs. non-reactive?

1. baseline FHR (nl is between 110-160 bpm) 2. FHR variability: reflects intact and mature brain stem and heart (moderate variability is nl = variability in HR of 6-25 bpm) 3. Changes in FHR : accelerations and decelerations - Also note the presence of any contractions To constitute a reactive NST, must have at least 2 accelerations (15 bpm above baseline for 15 secs) in a 20 minute time period If non-reactive at 20 minutes, can continue for 40 more minutes before considered to be non-reactive NST requiring further testing

How can one stimulate contractions to perform a CST?

1. oxytocin 2. nipple stimulation: 2 min self-stimulation through clothes every 5 mins

What percentage of babies will have congenital CMV after maternal infection?

10 percent

What prenatal tests should occur at: 10 weeks? 11-13 weeks? 16-18 weeks? 18-20 weeks? 24-28 weeks? 35-37 weeks?

10 weeks (or later) cell free DNA: - screens for Tri 21, 13, 18, Turner syndrome, and sex of fetus 11-13 weeks First tri screen, includes: - NT via U/S - maternal serum PAPP-A and free BhCG Note: in Down syndrome NT and BhCG are elevated, PAPP-A is decreased 16-18 weeks Quad Screen (15-21 weeks) includes: - uncong estradiol - alpha-fetoprotein (AFP) - BhCG - Inhibin A 18-20 weeks U/S for anatomy abnormalities (cardiac defects, other congenital anomalies) 24-28 weeks One hour GTT to screen for GDM At 28 weeks, recheck Ab and administer RhoGAM if indicated 36-38 weeks GBS culture (don't have to retest for four weeks)

How long does the copper IUD last?

10 years

When can a fetuses heartbeat be detected by Doppler?

10-12 week GA

The First Trimester Screen (FTS) is performed between weeks ______ and _____. What does this entail? What findings suggest Tri 21?

11-13 weeks - maternal blood screening test (maternal hormones: B-hCG and PAPP-A) plus U/S (measuring nuchal translucency) in context of maternal age - identifies Tri 21 mainly, also Tri 18 B-hCG and Nuchal Translucency (NT) are elevated, PAPP-A is decreased "A BEAT (beta) UP neck (NT), makes PAPA feel DOWN." Note: FTS has 85% sensitivity for Tri 21 If abnl FTS, follow with diagnostic testing (i.e. amniocentesis, CVS)

What is the normal baseline FHR in bpm?

110-160 bpm baseline determined from most common HR lasting for at least 10 mins

Fundal height of the gravid uterus should be where at: 12 weeks? 16 weeks? 20 weeks? 36 weeks?

12 weeks - level of pubic symphysis 16 weeks - between pubic symphysis and umbilicus 20 weeks - umbilicus 36 week - diaphragm

Mild preeclampsia is defined by: - BP: ___/___ - Proteinuria: ___ on dipstick or > ___mg/24hr - Symptoms?

140/90 Proteinuria 1+ dipstick or > 300 mg/24hr No subjective symptoms

When is an amniocentesis usually performed? Indications? Risks

15-20 weeks, karyotype from fetal cells or check for fetal lung maturity Indications: US suspected fetal anomaly abnl genetic screening test AMA FHx of congenital abnormalities Risks: cramping spotting 1-2% leak amniotic fluid sx amnionitis (<1/1000) fetal loss (1/500-1/300 or lower w/ experienced providers)

If patient has risk factors for gestational diabetes, when do we perform an early screen?

16 weeks

Severe preeclampsia is defined by: - BP: ___/___ - Proteinuria: ____ mg/24 hr - Symptoms: ________

160/110 > 5g/24 hr, oliguria < 500 cc/24 hr Headache, visual disturbances, epigastric/RUQ pain

What medication is given to women with risk factors or history of preterm labor?

17a-hydroxyprogesterone - given as weekly IM injections starting at 16-20 weeks - relaxes the myometrium - prevents rejection of fetus by suppressing lymphocyte production of cytokines

Describe what happens in each of the 3 stages of labor.

1st (2 phases): Onset of labor (uterine contractions) --> fully dilated (cervix to 10 cm) 2nd: Fetal expulsion = delivery 3rd: Placental separation

What is the IUD expulsion rate in the first year?

2-10% Note: Expulsion rates for immediate postpartum IUD insertion may be as high as 10-27%

What is the treatment for anemia during pregnancy?

200 mg of elemental iron daily (ferrous sulfate, fumarate, or gluconate)

Begin pap smear screening at _____ age.

21 yo

Cervical cancer screening recommendations: ACOG 2016 *Capture "Preferred"*

21-29 yo get pap cytology every 3 yrs 30 - 65 yo recommendation is to do cotesting (pap cytology and HPV) every 5 years; can opt for cytology only and repeat q3y Note: women under 21 yo should not be screened as the the incidence of cervical cx is very low and their immune system allows them to clear the infx typically w/in 8-24 mos

When do we typically order the Glucose Challenge test (1-hr glucola)?

24-28 weeks

Ovulation takes place __________ hr after LH surge and ___________ hr after LH peak.

24-36 hr 12 hour after peak

In eclampsia, when do most seizures occur? Before/during/or after delivery?

25% before 50% during labor 25% within 10 days postpartum

Regular cycle intervals is defined as... Avg length

28 avg +/- 7 days Avg length 4-5 days, with days 2-3 being the heaviest so period occur every 21-35 days If more than 35 days between cycles this is oligomenorrhea If no bleeding for 6+months that is amenorrhea

If the antibody screen is negative, the fetus is presumed to be D positive. One dose of anti-D IgG immune globulin (RhoGam) is given to the mother at _____ weeks.

28 weeks

Grade 3 tumors usually (do or do not?) have steroid hormone receptors while grade 1 tumors (do or do not?)

3 do NOT; 1 do

Hep B Infant infection - long-term consequences?

70-90% of acutely infected infants develop chronic infection that can lead to cirrhosis and HCC

Neonates who are diagnosed with an infection within the first ___hrs were likely infected in utero or transplacentally.

72 hours

Between 21 and 29, pap smears every ___ years.

3 years

Cervical Cancer in Pregnancy: Epidemiology Screening Tx Delivery Considerations

3% of all invasive cervical cx occur in pregnancy 33% are Asx sx: vag bleeding and pink discharge - okay to do pap if due at initial ob visit - colpo with bx okay, ECC NOT okay - cone bx, if necessary for dx and results would alter timing/mode of delivery, done in T2; can cause hemorrhage and PTL Definitive tx is incompatible with continuation of pregnancy - can do platinum-based chemotherapy in T2 - T3 CS with radical hysterectomy and lymphadenectomy Delivery Considerations - tumor hemorrhage during labor - size influencing mode of delivery -- small volume stage 1A may be able to deliver vaginally, > 1A need CS - avoid episiotomy d/t case reports of cancer implantation at scar sites

primary syphilis incubation

3-90 day incubation time; avg 21 days between exposure and lesion development lesion spontaneously goes away

A patient's blood glucose level is > 140 after the glucose challenge test. What test is ordered next to diagnose gestational diabetes?

3-hr glucose tolerance test

_____% of patients treated for cervical cancer will have a recurrence. Where do they tend to recur? WHat to look for?

30% recur - mainly in pelvis: vagina, cervix or lateral pelvic wall Look for: -vag bleeding -hematuria -dysuria -constipation -pelvic and leg pain -fistulas in bladder or bowel -sacral backache or pain in sciatic nerve distribution -u/l leg swelling (LN blocked) -CVA tenderness or flank pain

How much blood loss is considered normal during a period? Is clot formation during a period considered to be normal? If not, what is pathology is it indicative of?

30-50 mL >80 mL is menorrhagia Clot formation is not considered to be normal Clot formation during a period is indicative of anemia

HDN w/ severe anemia: When should labor be delivery occur?

32-34 weeks

At what week is amniotic fluid volume maximal? What happens after max volume is reached?

34 weeks (750-800mL) Decreases to 600mL by week 40

All pregnant women should be screened for GBS between ____ and _____ weeks gestation. What is the management if they are positive?

35-37 weeks ACOG says 36 0/7 to 37 6/7 result good for 5 weeks so no need to re-screen if born at 41 0/7 weeks If positive give intrapartum prophylaxis (PCN 1st, if allergic--> cefazolin)

Suppression with acyclovir starting at _____ weeks is indicated for those with a history of herpes.

36 weeks

HDN w/ mild anemia: When should labor be induced?

37-38 weeks

What is the at risk period for complications during pregnancy associated with parovirus infection ? What are the possible fetal complications?

4-20 weeks gestation Abortion/fetal death Hydrops fetalis: aplastic anemia (1%)

What female factors effect fertility (%)?

40% multifactoral 28% unexplained 21% ovulatory dysfn 14% Tubal Issue 6% Endometriosis Also, Uterine anatomy? Polyps or fibroids?

what percentage of women who become pregnant with an IUD spontaneously abort in the first trimester?

40-50%

How much folic acid should be taken to prevent NT defects? How about if the woman had a prior pregnancy with a know

400 microgram/day aka 0.4 mg/day

What is the average age for *cervical cancer*?

47y/o

If maternal infection ____ days before or after delivery, give VZIG to neonate for passive immunity.

5 days higher risk of infection if infected around labor due to lack of maternal antibodies Deep Dive: - early pregnancy infection leads to congenital malformations - late pregnancy infection has lower risk of dev congenital VZ infx

hCG becomes detectable in blood levels at

8 to 10 days after fertilization = 3 to 3.5 weeks after the LMP

More than _____ years of OCP use decreases the risk of ovarian cancer by 25-50%.

5 years Note: this protection lasts 15 years after discontinuation

Postpartum Thyroid Dysfunction

5-10% of women -transient lymphocytic thyroiditis w/in 1 yr PP Two Phases: 1. Thyrotoxicosis (1-4 mo PP) >>Mild hypERthyroid sx 4-8 weeks postpartum > lymphocytic infiltration > Sx spontaneously resolve after few months 2. HYPOthyroidism (4-8 mo PP) Next develop hyPOthyroid phase with clinical sx > Spontaneous resolution in another few weeks to months Treatment: only in immediate postpartum period, limited to symptomatic patients -b blockers -Low-dose synthroid

What is the breakage rate of condoms?

5-8%

Single dose MTX regimen for ectopic

50 mg/m2 IM on day 1 HCG on post tx D4 /D7 - If > 15% decr -> HCG weekly until neg - If < 15% decr -> 2nd dose of MTX @ same dose and repeat HCG - If no decr x 2 -> surgery

2 Dose MTX regimen for ectopic

50 mg/m2 IM on day 1 and day 4 HCG on post tx D4 /D7 - If > 15% decr -> HCG weekly until neg - If < 15% decr -> 3rd dose of MTX @ same dose on D7 and repeat HCG on D11 *** if > 15% decr -> HCG weekly until neg *** if < 15% decr -> 4th dose on D11 -> HCG D14 - If no decr after 4 doses -> surgery

Average age of onset of menopause?

51 years of age (US)

Menopause occuring after _____ yo is deemed late menopause.

55 yo

Epidemiology of Ovarian Cancer

5th most common cancer for women, but the deadliest 70% diagnosed with stage 3 or 4 lifetime risk is 1 in 70 median age at Dx is 63 yo

If galactorrhea persists > ______ months after cessation of breastfeeding, check TSH and Prl levels.

6+ months

How many adults by age 20 would be immune from parovirus ?

60%

A 20 yo F presents with CC of break through bleeding for more than 3 months of taking low dose OCP. What can you give her to alleviate this side effect?

7 day course of estrogen

Risk Factors for Cervical Dysplasia and Cx

80% cases 2/2 HPV infection - high risk if infx > 6 mos - esp if types: 16, 18, 31, 33, 45 (highly oncogenic) - Early onset of Sexual activity (<18 yo) - Multiple sexual partners - A high-risk sexual partner - History of STIs - History of vulvar or vaginal squamous intraepithelial neoplasia or cancer - Immunosuppression (eg, HIV infection) - genetic predispo - cigarette smoking (incr risk of SCC but not adenocx) - OCPs with use > 5yr (decr condom use in these women)

How effective is lactational amenorrhea in preventing a pregnancy?

82% in the first year

Epithelial Cell Ovarian Cancer: Subtypes

90% of ovarian cancers are epithelial 6 Subtypes: serous (50%) -note: b/l 65% of the time mucinous (25%) endometrial (10%) clear cell (6%) brenner (4%) undifferentiated (5%)

Management of PROM at term, > 37 weeks

90% of term patients go into spontaneous labor within 24 hours after rupture. - Patients in active labor should be allowed to progress - If labor is not spontaneous, it should be induced. C-section for other indications.

____ % of all ovarian germ cell tumors are benign.

95% benign

An endometrial thickness of < 5mm in a postmeno woman with vag bleeding has a neg predictive value of _____ for endometrial cx.

99%

CDC defines anemia in pregnancy as a Hgb < _______ g/dL.

<11 Note: 16-29% become anemic in T3

What is considered prolonged rupture of membranes?

> 18 hours

Cesarean delivery should be considered if fetal weight is > ____ g.

> 4500g

When do you stop performing pap smears?

> 65 yo no h/o abn cells; 3 neg paps or 2 neg cotests (Paps + HPV testing) in the past 10 years

What is vasa previa?

A condition in which the unprotected fetal cord vessels pass over the internal cervical os, making them susceptible to rupture when the membranes are ruptured

Natural Family Planning (NFP): Defined Types Advantages and Disadvantages

A form of BC based on timing of sex around patient's menstrual cycle 1. Basal Body Temp - records temp every morning right upon waking - temp increases .3-1 deg Ffor 3 consecutive days representing progesterone surge which signifies ovulating 2. Ovulation/Cervical Mucus (aka Billings)Method - more, clear, stretchy, wet mucus at time of ovulation - after peak day the mucus gets thick again - if using to avoid pregnancy, abstain for 4 days after peak day 3. Symptothermal Method - temp everyday - mucus quality - symptoms: cramping, spotting, changes in position and firmness of cervix - abstain from first signs to 3days after Temp elevation 4. Lactational Amenorrhea - nurse on demand without supplementation - ~98% efficacy in first 6 months if exclusively BF and amenorrheic

Define infertility.

A woman who is unable to achieve pregnancy after 12 months (under 35 yo) or after 6 months (35+ yo) of unprotected intercourse or inseminations 15%

Class A1 vs A2 Gestational Diabetes (GDM)

A1 is diet-controlled A2 is severe enough to require insulin (fasting > 105, 2h postprandial >120)

Which antihypertensives are NOT given during pregnancy due to teratogenic potential?

ACE inhibitors ARBs - result in hypocalvaria and renal defects

Pap Screening Guidelines for *discontinuing*?

All agree that screening may be *discontinued at 65y/o*, if there are either at least: · *3 consecutive negative cytologies* OR · *2 consecutive negative co-tests in the past 10y, w/ the most recent test w/in the past 5y* ACS, ACP, & ASCCP also require that there must be *no hx of CIN II or greater in the past 20 years.*

What is the treatment of choice for placenta accreta/ increta/ percreta?

All result in hemorrhage in 3rd stage of labor. Treatment of choice is *hysterectomy*

According to ACOG, who should have a Dexa scan to assess BMD? At what interval, should one repeat these scans?

All women aged 65 years or older should have a BMD test. Women who are younger than 65 years and past menopause, should have a BMD test if they have had a bone fracture because of fragile bones or have other risk factors for osteoporosis, such as rheumatoid arthritis, smoking, alcoholism, personal h/o pathologic fx, a history of hip fracture in a parent, or a body weight less than 127 pounds. In the absence of new risk factors, screening should not be performed more frequently than every two years.

What symptom is present in 90% of cases of endometrial cancer?

AUB - bleeding in post menopausal women (classic) - Meno/Metrorrhagia (in pre-menopausal women) - abn pap in 1-5%, not diagnostic, if find abn glandular cells investigate further (EMB)

Abnormal Uterine Bleeding (AUB)

AUB is menstrual bleeding of abnormal quantity, duration, or schedule Ask Pt: onset, menstrual hx, # tampons/pads use (type i.e. regular, super?), night time changing, bleeding through clothes/sheets, clots, h/o anemia or epistaxis or gum bleeding or PP bleeding, or post-surgical bleeding, FHx of bleeding, etc.

Not otherwise specified cause of abnormal uterine bleeding

AV malformation

min criteria to Dx PID?

Abdominal pain + CMT or adnexal ttp or uterine ttp

Define cervical dysplasia. Describe the 3 possible paths for dysplasia.

Abnl cells of the cervix that can be precancerous - Pap smears assess for dysplasia Dysplasia can... 1. progress to cx 2. Remain the same 3. Regress

What male factors effect fertility?

Abnormal Sperm --> Count, Motility, Morphology Obstruction of ductal system (seminiferous tubules to urethra) Also think... -Sexual dysfunction? -Medications? -Developmental? (XXY, cryptorchidism?)

What are the complications of preeclampsia?

Abruption Eclampsia w/ intracranial hemorrhage, blindness Coagulopathy Renal failure Hepatic subcapsular hematoma Uteroplacental insufficiency

What is the perforation rate at the time of insertion for LNG-IUD vs copper IUD?

According to ACOG, Perforation is rare, occurring in 1.4 per 1,000 LNG-IUD insertions and in 1.1 per 1,000 copper-IUD insertions (32).

1st stage of labor Active phase classifications

According to the rate of cervical dilation: - Acceleration phase - Phase of maximum slope - Deceleration phase

Easy way to remember: Accreta = ____ Increta = ____ Percreta = ____

Accreta = Attaches Increta = Invades Percreta = Penetrates

condyloma acuminata vs condyloma lata

Acuminata : ass w HPV (6 and 11) which have a pearly and plaque like or cauliflower appearance Lata : Genital warts in secondary syphilis which are non painful, raised, grayish white lesion

HSV suppressive therapy

Acyclovir, Valacyclovir or Famiciclovir taken daily leads to 70-80% reduction in # outbreaks

How do you measure AFI? What number represents oligohydramnios? polyhdramnios? What is the median AFI between 20-35 weeks GA?

Add up the amniotic fluid pockets in 4 quadrants seen on U/S oligohydramnios is an AFI of 5 or less polyhydramnios is an AFI of 25 or higher median AFI is 8-18 cm between 20-35 weeks, decreases after that.

A 45 yo woman presents to your office with chief complaints of long term heavy menstrual cycles and dysmenorrhea. Her pain and heavy bleeding sometimes limit her ability to work during the first few days of each menstrual cycle. Her cycles are regular. She is generally healthy and is up to date on all routine health maintenance, including Pap smears. On exam she has a normal appearing cervix. Her uterus is diffusely enlarged with no distinct masses. Ultrasound confirms a slightly enlarged, globular appearing uterus. Endometrial biopsy is performed and is benign. She is slightly anemic. She reports prior use of several types of hormonal birth control (including oral contraceptives and the Levonorgestrel IUD) without significant improvement. After counseling, she proceeds with hysterectomy. What is the diagnosis?

Adenomyosis

Compare and Contrast: Adenomyosis vs Endometriosis

Adenomyosis - older, multiparous women - not responsive to hormones - non-cyclic pain Endometriosis - young, nulliparous - tissue responsive to hormonal stimulation - cyclic pain

Why are obese women at higher risk of endometrial cancer?

Adipose tissue converts androgens into estrogens which lead to hyperestrogenism which is a known risk factor for endometrial cancer

PMS - Symptoms - Diagnosis

Affective Sx - depression - anxiety - angry outburst - irritability - confusion - social withdrawal Somatic Sx - breast tenderness - HA - bloating - extremity swelling DX - At least one affective and somatic symptom in the 2nd half (luteal) of the cycle. - symptoms gone shortly after onset of mesnes - symptoms occur for 3 cycles - exclude other diagnoses

Describe the ovarian follicle changes that occur as a women approaches menopause.

Aging ovarian follicles decline in size and number (atresia) - oocytes undergo apoptosis and become resistant to FSH - women ovulate less frequently, have a shorter follicular phase but same length in luteal phase

Discuss the utility and assement of endometrial stripe in the postmenopausal patient ONLY.

An endometrial stripe of < or equal to 4mm has an excellent negative predictive value and can be used to rule out endometrial cancer. If the endometrial stripe is <4mm in a postmenopausal patient then the likely cause of the bleeding is endometrial atrophy. Endometrial biopsy is not usually warranted in these patients. Note: Premonpausal women have widely fluctuating thickness depending on time in their cycle thus we do not use this in diagnosing AUB

Lichen Planus on vulva: What is it? Types? Si/Sx Dx Tx

An inflammatory condition of vulva, scalp, skin, nails, & mucous membranes Can present as vulvo-vaginal-gingival syndrome usually postmenopausal 50-60s Si/Sx small, shiny, purple, itchy lesions - thicken with scratching - most are found on the inner vulva and vagina - may see Wickham striae (like the strings on a ham netting) = fine, thin, white lacey pattern vulvar pruritus, vulvar pain & burning, vaginal discharge, Dyspareunia or postcoital bleeding, Note: has an autoimmune component 3 Types: Papulosquamous LP: affects vulva. Hypertrophic LP: affects perineum; perianal area. Erosive LP: affects vulva, introitus and vagina Tx: topical steroids

Paget Disease of the Vulva What is it? Presentation Bx Tx Follow up?

An intraepithelial disease of unknown etiology Affects mostly caucasian postmenopausal women in 60s May be assoc with underlying local or distant adenocarcinoma (GI, breast) - Vulvar soreness and itching - Appears as a red lesion with superficial white coating (eczematoid), often multifocal - Usually found on: Thighs, mon pubis, rectum, vagina Bx shows paget cells Tx - Assessment for other potential concurrent adenocarcinomas - Then surgical removal by wide local resection to SQ fat of the involved area with a 2-cm margin - Intraoperative frozen section of the margins can be done to ensure complete resection; high likelihood of recurrence F/U follow annually with breast exams cytology of cervix and vulva screen for GI dz

condylomata acuminata presentation

Anogenital warts caused by HPV Flesh-colored, soft papules or plaques May cause burning, itching, bleeding

HMB in adolescents is most likely due to _______________.

Anovulatory cycles, but good to r/o coagulopathy esp if other symptoms like easy bruising

Types of vaginal/vulvar cysts?

Bartholin cyst/abscess Sebaceous (aka epidermoid) cysts Hidradenomas Cyst of canal of Nuck Skene's duct cyst

List methods for assessing ovulation.

Basal Body Temp - rises about 0.5 - 1.0 F during the luteal phase due to progesterone incr Day 21 mid luteal serum progesterone - ovulatory if > 3 ng/mL Ovulation predictor kit - tests for the LH surge

Is endometrial cancer diagnosed early or late typically?

Bc it usually presents with obvious symptoms, it is most often diagnosed at an early stage Avg age Dx 61

Sensitive way to pick up IUGR

Fundal height

Vaginal cultures for GBS

Weeks 35-37

Single most important characteristic of baseline FHR?

Beat to beat variablitity (BTBV) measured from peak to trough - Variation of successive beats in the FHR - controlled primarily by the ANS = index of fetal CNS function - min variability if < 28 weeks' GA because fetus is neurologically immature - Divided into Short-Term Variability (STV aka BTBV) and Long-Term Variability (LTV)

Are you more likely to have breast pain before menopause or after?

Before

What is the difference between GnRH secretion before and after puberty?

Before - continuous After - pulsatile (which causes menstruation to become cyclical)

What test is used to diagnosis a PE in pregnancy?

CT pulm angiography or lung scintigraphy (V/Q scan)

Compare and contrast amniocentesis and CVS.

CVS: - transvag or trans abd asp of precursor cells - evaluates chromo abnormalities but not NTDs - done earlier at 9-12 weeks - higher risk (fetal loss at 1% risk and limb issues if done at less than 9 weeks) Amnio: - transabdominal fluid asp via US guidance - evaluates chromo abnormalities - done later between 15-20 weeks - indicated for AMA - risk of fetal loss is <1%

Magnesium toxicity (7-10 mEq/L) is associated with loss of patellar reflexes (hyporeflexia), respiratory depression, and cardiac arrest. What is the antidote?

Calcium gluconate 10% solution 1g IV

Vaginal bleeding, foul smelling yellowish discharge and post-coital spotting think...

Cervical cancer

Uterine aspiration

Can differentiate b/w early pregnancy loss vs ectopic Should see decr in HCG (>15% in 12 hours) Path - chorionic villi Consider before MTX

What is the CNS anomaly most specific to DM?

Caudal regression: sacral agenesis/hypoplasia, abnormal development of the lower spine

This medication should be avoided in late pregnancy because it adversely affects hemostasis of fetus and displaces bilirubin from protein binding sites

Aspirin

Most common cause of acute endometritis

Bacterial infection after delivery or miscarriage

Which fontanelle is larger, anterior or posterior?

Anterior

If the mother is Rh-(D-) and her fetus is Rh+(D+), she may become sensitized to the D antigen and develop ______ against the baby's RBCs.

Antibodies

What is the treatment of DVT during pregnancy?

Anticoagulation with: - Unfractionated heparin OR - Low-molecular-weight heparin If possible, stop at least 6 hours before delivery and resume 6 hours post VD or 6-12 hours post CD; essentially stop it once labor starts.

What is the treatment for preexisting HTN, transient HTN, and chronic HTN in pregnancy?

Antihypertensive medications vs close observation. *Methyldopa, Nifedipine*

Which is the most thrombogenic of the heritable coagulopathies?

Antithrombin III deficiency

What stage of maternal syphilis nfection can result in fetal infection?

Any stage

Causes of hyperprolactinemia?

Anything that decreases Dopamine (which inhibits Prl)

What are the most common indications for surgery in pregnancy?

Appendicitis Adnexal Masses Cholecystitis

Problems that anticoagulants are used to treat

Arrhythmias, DVTs, stroke, and CAD

what is PID/consequences

Ascending infection *from cervix to fallopian tubes* Frequently seen in the ER Leads to *infertility, scarring, sepsis, chronic pelvic pain*

SECONDARY AMENORRHEA Possible causes of uterine dysfunction?

Asherman Syndrome Endometrial ablation (for menorrhagia) Infection (Endometriosis,TB)

Risk factors for hereditary breast cx?

Ashkenazi Jew Personal hx of breast/ovarian cx < 40 yo 2= relatives with BCx (<50 yo)

Expectant management for ectopic is appropriate when...

Asymptomatic Objective evidence of resolution (no incr HCG) Counseled about risks: rupture, hemorrhage, emergent surgery HCG < 200 -> 88% resolve

When should HRT be initiated? Indications? Dosing recs?

At onset of symptoms To control moderate to severe VMS aka hot flashes Prescribe the lowest effective dose for the shortest duration and increase gradually

Lichen Sclerosus on vulva: What is it? Si/Sx Dx Tx

Atrophic lesion with paper-like appearance on both sides of the vulva epidermal contracture leads to loss of vulvar architecture Si/Sx thinning, white/pale, shiny paper-like appearance often in a figure 8 formation around vulva and perianal area - not usually painful - may bleed as it is very thin - severe cases, contractures and fissures on posterior fourchette Bx - Epithelial thinning with layers of collagen and inflammatory cells - Loss of rete ridges Note: SCC risk (highest out of all three lichens) Tx: Topical steroids

What is the purpose of the ligaments in the pelvis?

Attach the pelvic organs to the bony pelvis.

If a woman is infected with chickenpox near term, what is the protocol?

Avoid elective delivery for 5-7 days after rash appears WHY? to allow for the transfer of maternal Abs to the fetus

What immunosuppressant can be safely used in pregnancy?

Azathioprine

Initial lab tests to order for AUB in reproductive-age women? Other diagnostic tests?

B-hCG (prego? or complication of pregnancy i.e. abortion) FSH (POF) TSH (thyroid d/o) Prl (hyperprolactinemia) Pap smear (cx?) Coag panel (bleeding d/o?) Pelvic US -Trans Vaginal (TVUS) is preferred for eval of AUB - Trans Abdominal (TAUS) Sonohysterogram (pelvic US + intrauterine saline infusion to outline uterine cavity) Hysteroscopy (can diagnose and treat)

What are contraindications to tocolysis? BAD CHU

BAD CHU B - Severe *bleeding* from any cause A - Severe *Abruptio placentae* D - Fetal *Death*/life incompatible anomaly C - *Chorioamnionitis* H - Severe pregnancy-induced *HTN* U - *Unstable* maternal hemodynamics

List the general methods for preventing pregnancy.

Barrier Hormonal (oral, injectable, implant) Intrauterine Device (IUD - hormonal, copper) Sterilization Abstinence

Anorexia as a cause of amenorrhea (effects, other similar causes, possible mechanism)

Below 17% body fat or around 10% less than ideal (but variable - can be more susceptible depending on genetics), women often stop menstruating due to hypothalamic dysfunction (a defense mechanism to prevent unwanted pregnancy in times of stress). So hypogonadotrophic hypogonadism! This also occurs in times of severe psychological stress like war, imprisonment, hospitalization, or with marked exercise and severe fat restriction. May have to do with the fact that leptin is made by adipocytes and seems to regulate menstrual function (if given exogenously, can restore menstruation in underweight women) The lack of estrogen from lack of hypothalamic stimulation importantly can also stimulate bone loss.

What is the most common germ cell tumor?

Benign cystic mature teratoma (dermoid cyst) - contain hair, teeth, skin-> mixed with sebacous substance

What are leiomyoma variants?

Benign metastasizing leiomyoma and leiomyomatosis (benign but disseminated nodules); both rare

What is the mechanism of action of Ritodrine & Terbutaline as tocolytic agents?

Beta agonist: B2 receptor stimulation on myometrial cells --> increases cAMP --> decreases intracellular Ca --> decreases contractions

hCG structure and function is similar to what other compounds?

Beta subunits - similar to LH Alpha subunits - identical LH, FSH, and TSH

Anti-resorptive therapy for osteoporosis and osteopenia?

Biophosphonates: Alendronate (Fosomax), risendronate (Actonel) Second-line: raloxifene, Calcitonin, PTH Denosumab ( monoclonal Ab, inhibits activity)

At 32-34 weeks in A2 gestational diabetics, what tests should be ordered for fetal monitoring?

Biophysical profile Growth US every 4 weeks

If a NST is nonreactive, it should be followed by a ___________.

Biophysical profile (BPP) **See "Antepartum" flashcard set

Diagnosis of vulvar cancer

Biopsy Always biopsy an itch white lesion

Cervical cancer diagnosis

Biopsy and sometimes excisional procedure

If a new pt with vaginal vault prolapse comes in with ulceration, what should you do first?

Biopsy the ulceration to exclude vaginal neoplasm.

If fertilization has occurred and a zygote is formed, what is the name of the group of cells that implants into the endometrium? What are the two layers of the blastocyst, and in very general terms what does each layer eventually become?

Blastocyst 1. Inner cell layer becomes the fetus 2. Outer cell layer becomes the trophoblast, which will eventually embed itself into the endometrium and become the placenta

A 65 yo patient presents with vaginal bleeding. She has been menopausal for about 15 years with no bleeding. She is in overall good health, but recently developed an arrhythmia (atrial fibrillation) and her primary care doctor placed her on Xarelto (Rivaroxaban) for prevention of clots/stroke. She has always had normal Pap smears. Her examination shows atrophy of the genital tissues, but otherwise no abnormalities. Her uterus is small and mobile. Ultrasound should a thin endometrial lining of 5 mm. She undergoes hysteroscopy and D&C and no abnormalities are seen. Curettage specimen is benign. After counseling on options, she elects to undergo endometrial ablation, which successfully decreases her bleeding. What is the diagnosis?

Bleeding due to anti-coagulants

When are you able to hear fetal heart tones? With which tools?

Doppler at 10-14 weeks Fetoscope at 17-20 weeks

PRIMARY AMENORRHEA What condition(s) result in primary amenorrhea without breasts and a uterus? Describe the condition(s) and interventions.

Both uterus and breasts absent 17 alpha hydroxylase deficiency (in XY karyotype) - have testes thus AMH so no female internal genitalia - lack enzyme to make sex steroids, so low T --> thus female external genitalia - low E means no breasts

PRIMARY AMENORRHEA What conditions result in primary amenorrhea in the presence of both a uterus and breasts? Describe the condition(s) and interventions.

Both uterus and breasts present Imperforate hymen fails to canalize, blood accumulates behind. Corrected surgically transverse vaginal septum failure of upper vagina to fuse with lower vagina. Corrected surgically - Nl breast and uterus - cyclic pelvic pain but without menses --> hematocolpos (acc of blood in vagina behind hymen) may feel as perirectal mass Tx: excise hymen

Chemical from lungs causing the closure of the ductus arteriosus, ductus venosus, and umbilical vein

Bradykinin

Main side effects of Progestin-Only Pills (POPs)?

Breakthrough bleeding (esp first few months) Nausea (10-30%)

Mastitis What is mastitis? How does mastitis present? How do we treat mastitis?

Breast infection, often as a result from breastfeeding/clogged ducts, but spontaneous cases are possible too, especially in smokers, IVDU Si/Sx -red (wedge), hot, swollen, tender breasts +/- abscess (fluctuant mass --> US) -can cause flu-like sx Tx -antibiotics if systemic (Keflex, Duricef, dicloxacillin (breastfeeding), Bactrim, Clindamycin) -proper breast feeding technique, incr frequency -hot compresses -I&D/aspiration for abscesses -surgery (rare) -imaging +/- biopsy if no improvement

Mastalgia How do we typically treat mastalgia? Is mastalgia often tied to hormonal changes?

Breast pain -support garments -NSAIDs/tylenol -low fat diet -less caffeine -OCPs It can be (cyclical) or it may be unrelated (noncyclic) Note: cyclic is more common.

Hereditary Ovarian Cancer Syndromes: List them and their associations to other cancers and genetics.

Breast-Ovarian Cancer Syndrome - cancer of breast and ovary linked to mutated BRCA-1 and 2 tumor suppressor genes foun on chromo 17 Lynch II Syndrome aka Hereditary Nonpolyposis Colon Cx - sites include breast, ovary, uterus and colon Site-Specific Ovarian Cancer <1% of hereditary ovarian cx - strong genetic link - 2 or more 1st deg relatives have it

PRIMARY AMENORRHEA - Causes When... Breasts absent, uterus present Breasts present, uterus absent Both absent Both present

Breasts absent, uterus present - no estrogen from ovaries 1. Gonadal Dysgenesis (hypergonadotropic, hypogonadism) aka streak ovaries aka Turners 2. 17 alpha hydroxylase deficiency in XX karyotype 3. Hypothalamic-pituitary d/o (e.g. anorexia) low estrogen due to low GnRH a) anatomic lesion of hypothalamus e.g. prolactinoma b) inadequate GnRH release (hypogonadotropic hypogonadism 4. Kallman Syndrome: low gonadotropins due to failure of cell migration during development 5. Isolated Gonadotropin deficiency (pituitary dz) Breasts present, uterus absent Androgen Insensitivity (AIS) aka testicular feminization - XY with normal functioning testes - missing or lack of response of androgen receptors Mullerian Agenesis Congenital failure of mullerian duct to form - missing uterus, fallopian tubes, and upper 2/3 vagina - Normal ovulating ovaries, so nl female hormones Both uterus and breasts absent 17 alpha hydroxylase deficiency (in XY karyotype) - have testes thus AMH so no female internal genitalia - female external genitalia Both uterus and breasts present Imperforate hymen or transverse vaginal septum

PRIMARY AMENORRHEA What condition(s) result in primary amenorrhea with a uterus present and absent breasts? Describe the condition(s) and interventions.

Breasts absent, uterus present - no estrogen from ovaries 1. Gonadal Dysgenesis (hypergonadotropic, hypogonadism) aka streak ovaries aka Turners - internal and externally female - FSH/LH are high no negative feedback from estrogen 2. 17 alpha hydroxylase deficiency in XX - low cortisol, low adrenal/gonadal sex hormone secretion - HTN, hypernatremia, hypoK 2/2 xcs mineralcorticoid Tx: replace sex steroids and cortisol 3. Hypothalamic-Pituitary Disorder (e.g. anorexia) low estrogen due to low GnRH a) anatomic lesion of hypothalamus e.g. prolactinoma b) inadequate GnRH release (hypogonadotropic hypogonadism) - if given GNRH --> nl FSH/LH levels Tx: E+P to induce breast development and bone epiphyseal closure 4. Kallman Syndrome - low gonadoptropins due to failure of GnRH neurons to migrate during development - anosmia and low GnRH 5. Isolated Gonadotropin deficiency (pituitary dz) Other manifestations: - pre-pubertal hypothyroidism - kernicterus - mumps encephalitis thalasemmia major --> iron accumulates in pituitary - retinitis pigmentosa

PRIMARY AMENORRHEA What condition(s) result in primary amenorrhea with breasts and NO uterus? Describe the condition(s) and interventions.

Breasts present, uterus absent Androgen Insensitivity (AIS) aka testicular feminization - XY with normal functioning testes (testosterone and DHTare wnl) but may be intrabdominal or in inguinal canal - missing or lack of response of androgen receptors - thus, wollfian ducts do not develop as cells cannot sense testosterone - no internal male genitalia - nl external female genitalia with a short or absent vagina - breasts, no axillary or pubic hair Tx: remove testes due to increase risk malignancy and then give estrogen for development of female sexual characteristics, raised as female Mullerian Agenesis Congenital failure of mullerian duct to form Thus, missing uterus, fallopian tubes, and upper 2/3 vagina Normal ovulating ovaries, so nl female hormones Normal Breasts (since estrogen from ovaries) Normal axillary and pubic hair Renal and Skeletal abnormalities Tx: reconstruction of vagina for functionality

By which week are most of the embryo's organs formed & it is considered a fetus

By 7th-8th week

Metastasis of Cervical Cancer

By direct extension: RIB Eye steak Rectal Intra-abdominal Bladder Endometrial By hematogenous spread (BLBL): Breast Lung Bone Liver

3 ways to approach amenorrhea diagnosis...

By system involved: hypothalamus, pituitary, ovary, outflow By presentation: presence of breasts, uterus, progesterone symptoms By primary vs. secondary (all secondary causes can also present primary depending when they occur) - if approaching this way, note if breasts and/or uterus present ALSO ALWAYS RULE OUT PREGNANCY!

______ is a tumor marker that is elevated in 80% of epithelial ovarian cancer cases.

CA-125 Note: it is useful in tracking progression and response to treatment but not useful for screening as it may be elevated in many other conditions (e.g. fibroids, endometriosis)

What are the causes of postpartum hemorrhage? CARPIT

CARPIT C - Coagulation defect A - Atony of uterus R - Rupture of uterus P - Placenta retained I - Implantation site bleeding T - Trauma to GU tract

Labs to check before MTX admin?

CBC CMP

What labs might you order at the first prenatal visit?

CBC (Hct/Hgb) Rh factor Blood type Ab screen Pap smear (if due and over 21 yo, many woman defer until after, but it is safe to perform in pregnancy) Gonorrhea and chlamydia cultures UA (bacteria, pro, glu, ketones)/UCx Screening for infection: rubella, syphilis (RPR), HIV, hep B antigen, TB (if indicated) Cystic fibrosis screen Hgb electrophoresis (if high risk - e.g. FHx of unknown blood d/o or maternal anemia)

Initial Labs if you suspect coagulopathy?

CBC, PT, aPTT, and fibrinogen for clotting factor deficiency If suspect vWD, send to hematology so they can properly order and assess results of tests

Estrogen Preparations for HRT

CEE: conjugated equine estrogen EE: ethinyl estradiol

Most common congenital infection causing IUGR

CMV

What is the most common cause of perinatal infections in the developed world?

CMV

CYTOMEGALOVIRUS How is CMV infection transmitted ?

CMV is a dna virus sex, blood, bodily fluids, vertical transmission so infection can be intrauterine, intrapartum, or postpartum (BF) - day care is common source

What are the features of congenital rubella syndrome ?

CNS - Deafness - CNS defects -IUGR Eyes - Cataracts, congenital glaucoma (blind) Heart - VSD - PDA Abdomen - hepatosplenomegaly Other - Rash -Thrombocytopenia Note: newborns shed virus for many months

Signs of CMV in fetus

CNS involvement (microcephaly), chorioretinitis, intracranial calcifications, pneumonitits, hepatosplenomegaly (HSM), thrombocytopenia

Protective factors include...

COCPs Multiparity Reg ovulation

When is a CST deemed: Negative? Positive? Equivocal? Unsatisfactory?

CST looks at FHR decelerations in response to contractions Negative if no late or significant variable decels Positive late decels following 50% or more of contractions Equivocal if intermittent late or significant variable decels Unsatisfactory if fewer than 3 contractions in 10 mins

If the NST is non-reactive, what are your options to further investigate fetal well-being?

CST or BPP (favored as stimulating contractions incurs risks)

f/u after ovarian cancer surgery?

CT chest/abd/pelvis CA-125

Cause of low AFP?

Down syndrome

Patient who receives Depo injections desires pregnancy. She wants to stop the contraception and wonders how soon will her fertility return?

Can remain infertile for 9+ months

Invasive Breast Cx What is invasive breast cancer? What are exam findings of invasive breast cancer? How do we make the diagnosis?

Cancer that has grown through the lobules or ducts and has potential to spread through the lymphatic system to other areas of the body. -a palpable mass -nipple/skin changes -bloody nipple discharge -lymphadenopathy -imaging: US + biopsy

What are contraindications to using Ritodrine & Terbutaline as tocolytic agents?

Cardiovascular disease Hyperthyroidism Uncontrolled DM

gonorrhea tx

Ceftriaxone and Azithromycin

Cephalohematoma vs. Caput Seccedaneum

Cephalohematoma is collection of blood under the periosteum of the skull - does not cross suture lines - is due to rupture of vessels - resolves spont in a few weeks Caput is a temporary swelling from prolongation of engagement of fetal head, CROSSES suture lines, resolves in 1-2 d

A 45 year old G2P2 presents to your office complaining of intermittent heavy bleeding. She first noticed this as bleeding after intercourse for the past several years. It was light spotting at first, but has now become heavy and less predictable. She has also just recently noticed a foul smelling discharge. On review of systems, she reports that she has had some significant back/flank pain on her right side over the past month. She admits she has had minimal primary healthcare and thinks her last Pap smear was at the time of the birth of her last baby 20 years ago. She is thin, BMI 18. She is a heavy smoker - 1.5 PPD. On exam, she is noted to have a fungating mass replacing the cervix entirely. The mass is hemorrhagic and necrotic. Imaging shows severe hydronephrosis on her right side and her creatinine is mildly elevated What is the diagnosis?

Cervical cancer

Post-coital spotting think...

Cervical cancer

Transvaginal cervical length measurement is used to assess preterm labor. There is high risk of preterm delivery if the cervical length measures < ____ mm, especially with funneling.

Cervical length < 25 mm - high risk of preterm delivery

Differentials for cervical bleeding? Differentials for vaginal bleeding?

Cervical: 1. Cervical cancer (not cervical dysplasia) - Dx by observation 2. Cervicitis 3. Endocervical polyp Vaginal: 1. Trauma 2. Vaginal cancer (rare)

Basic treatment of cervical cancer confined to the cervix? If beyond the cervix?

Cervix only = radical hys + pelvic and paraaortic lymphadenectomy Beyond is chemo + rad

What is a pulsion enterocele? What other conditions are commonly seen after a pulsion enterocele? Tx?

Cervix prolapses and brings along the anterior margin of the cul-de-sac. Rectocele and cystocele. Excision of the enterocele sac and suspension of the vaginal vault.

What is the only management for placenta previa?

Cesarean delivery

What is the first step in management of the isoimmunized pregnant patient?

Check paternal erythrocyte antigen status

How often do you need to follow up with a patient using a vaginal pessary?

Check pessary 2-7 days after placement, then have her return every 3 months for an exam/cleaning if not done herself.

Main cause of bacterial STI in US is _______________.

Chlamydia 40% transmission rate per encounter

What is the most common cause of neonatal opthalmia?

Chlamydia trachomatis

What are complications of PROM?

Chorioamnionitis Placental abruption Neonatal infection Umbilical cord prolapse Preterm Labor

What conditions and procedures cause fetal-maternal bleeding?

Chorionic villus sampling Amniocentesis Spontaneous/ induced abortion Threatened/incomplete abortion Ectopic pregnancy Placental abruption/bleeding placenta previa Vaginal or cesarean delivery Abdominal trauma External cephalic version

Turner Syndrome: Hormone profile Characteristic features Tx

Chromosomal abnormality (45 XO) resulting in gonadal malformation, so will present with high LH and FSH but low estrogen - primary amenorrhea - streak ovaries - webbed neck - short fourth metacarpal and cubitus valgus - coarctation of Aorta - renal abnormalities - hypothyroidism TX: give E and P at puberty for secondary characteristics, also give GH

Causes of chronic endometritis

Chronic pelvic inflammatory disease, postpartum or post abortion with retained placental tissue, old IUDs, tuberculosis, idiopathic

Single most preventable cause of IUGR

Cigarette smoking

What is a contraindication for trial of labor after cesarean (TOLAC)?

Classical (vertical) uterine scar is a contraindication for TOLAC.

List 3 main ovulation induction agents.

Clomiphene citrate (GnRH agonist --> FSH incr --> LH surge) Gonadotropins (FSH/LH) Aromoatase Inhibitors (e.g. letrozole)

What is the treatment for mild preeclampsia in a patient who is preterm?

Close monitoring for worsening disease. Fetal testing: NST, BPP

What is the treatment for severe preeclampsia, preterm <28 weeks?

Close monitoring in hospital, select patients only.

A 16 yo patient presents with very heavy cyclic bleeding. She went through menarche at age 12 and periods have been excessively heavy from the beginning. Her bleeding remains heavy for nearly a full 7 days with each cycle. She often bleeds through her protection. She is otherwise healthy and has had otherwise normal pubertal development. She is not sexually active. Examination and ultrasound are normal. On further questioning on review of systems, she does report a tendency for nose bleeds and easy bruising with contact when playing soccer. What is the diagnosis?

Coagulopathy

In PPH if coagulopathy is suspected, use red top tube for clot retraction test. Normal coags if clot forms < 8 min. Coagulopathy if no clot in > ___ min.

Coagulopathy if no clot in > 12 min

List sites of implantation of ectopic endometrial tissue.

Common Ovary (bilaterally) 60% Peritoneum over the uterus anterior and posterior cul-de-sacs Broad ligaments/round ligaments/fallopian tubes uterosacral ligaments bowel appendix Less Common Rectosigmoid 10-15% cervix vagina bladder Rare Nasopharynx lungs CNS abdominal wall surgical/episiotomy scar

Who to treat medically?

Confirmed (e.g. unruptured mass) or highly suspicious for ectopic Ectopic pregnancy < 3.5 cm r/o IUP VSS No absolute contraindications for MTX Patient reliable for f/u hCG < 5k (differs depending on source) No fetal cardiac activity

Women with a seizure disorder have an increased risk of _____, even when they are not taking anticonvulsant medications. What is the fetus at risk of?

Congenital anomalies aka birth defects; 2x the risk of fetal malformations and preeclampsia (HTN plus proteinuria) when taking anticonvulsants - more prone to sz due to stress and fatigue Fetus is at risk of megoblastic anemia

In A2 gestational diabetics, when should early delivery be considered, and what testing may be indicated?

Consider delivery at 39-42 weeks if good dating. If poor glycemic control, consider amniocentesis for fetal lung maturity and delivery at term.

Non-hormonal IUD: Types? Mechanism? Best For? Disadvantages and Contraindications?

Copper IUD - Paraguard

What is the management of Vasa previa / velamentous cord insertion?

Correction of shock and immediate delivery (usually cesarean)

Bartholin's glands are analogous to the male ________ glands.

Cowper's gland aka bulbourethral glands which secrete thick alkaline fluid during sex

Other rare cysts: Cyst of canal of Nuck Skene's Duct cyst

Cyst of canal of Nuck - a hydrocele (persisten processus vaginalis) contains peritoneal fluid Skene's Duct cyst (paraurethral) - rare and very small - if sx, excise

What is the most commonly encountered form of pelvic organ prolapse?

Cystocele

Surgical Options: When is a D&C indicated for treatment of AUB? How about Endometrial Ablation? Myomectomy? Hysterectomy?

D&C for heavy bleeding leading to hemodynamic instability - start on medical management after acute bleeding episode is over Endometrial Ablation for when you fail medical management, don't care about fertility, but don't want hysterectomy Myomectomy allows for fertility preservation Hysterectomy for women done with child-bearing and want definitive treatment

Indications for osteoporosis screening?

DEXA scan of spine and hip in... - Women ≥ 65 years of age with no risk factors for fracture - Menopausal women with known risk factors for fracture

Reversible causes of urinary incontinence?

DIAPPERS Delirium Infx Atrophic vaginitis Pharma Psych Excessive UOP (DM, CHF) Restricted mobility (post-surg) Stool impaction

List 4 methods for assessing Ovarian Reserve.

Day 3 FSH Clomiphene Citrate Challenge Test (CCCT) - 100mg of clomiphene days 5-9 and measurements of FSH days 3 and 10 and estradiol on day 3 Antral Follicle Count - TVUSin early follicular phase Anti-Mullerian Hormone - marker of ovarian function, levels decline with follicle pool which declines with age

What are the fetal complications which can arise from rubella infection in pregnancy ?

Death Congenital Rubella Syndrome

Fetal Heart Tones (FHTs) detected by

Doppler US as early as 8 w GA If not by 11 w GA, US evaluation should be performed to document a viable intrauterine pregnancy

What is colostrum?

Deep yellow-colored liquid secreted by the breasts that contains minerals, protein, fat, antibodies, complement, macrophages, lymphocytes, lysozymes, lactoferrin, and lactoperoxidase - secreted by day 2 PP, persists for 5 days and graudally converted to mature milk by 4 weeks - has more minerals and protein, and less fat and sugar than breast milk - IgA Ab protect against enteric orgs

If good glycemic control: Consider _____ at 38 weeks, without fetal lung maturity and without amniocentesis.

Delivery

What is the only definitive treatment for preeclampsia?

Delivery

What is the management of a patient with PROM and chorioamnionitis?

Delivery is performed despite GA. Broad spectrum abx initiated (ampicillin, gentamicin)

What happens in the THIRD stage of labor? start/end?

Delivery of placenta Start: immediately after fetal delivery End: Delivery of the fetal and placental membranes

What is the treatment for severe preeclampsia, > 28 weeks?

Delivery, vaginal delivery attempted first. MgSO4 for seizure prophylaxis.

What is the treatment for mild preeclampsia in a patient who is term?

Delivery. MgSO4 for seizure prophylaxis. Vaginal delivery attempted first, Cesarean for other obstetrical reasons.

_____________ is an injectable hormonal contraceptive.

Depo (medroxyprogesterone acetate aka DMPA)

What is uterine prolapse? Risk factors? Tx?

Descent of the uterus and cervix down the vaginal canal. RFs: 1. Multiparity 2. Gravity 3. Loss of estrogen 4. Repetitive increases of intra-abdominal pressure. **Usually d/t injury of the endopelvic fascia and relaxation of the pelvic floor musculature Procedure of choice = hysterectomy Can use pessaries + topical estrogen if mild, temporarily, or pre-op.

Table: Early, Late, and Variable Deceleration FHR Patterns

Different meaning depending on when they occur in relation to contractions *Know Table* Note: variable, late, or prolonged decels in FHR interrupt fetal oxygenation

Most common cause of IUGR

Diminished placental functioning (MCC of placental dysfunction is HTN)

Why are diuretics not used to manage blood pressure in pregnancy?

Diuretics are not used in pregnancy because they decrease plasma volume and this may be detrimental to fetal growth. For this same reason, salt restriction is not recommended.

Staging for Endometrial Cx

Don't need for exam, may be helpful for wards

When is glucose screening done? Why is it done on everyone?

Done no later than 28 weeks gestation, usually at 24-28 weeks. Everyone gets tested because risk factors alone cannot identify all patients with gestational DM.

Treatment of hyperprolactinemia?

Dopamine agonists (bromocriptine or cabergoline) to inhibit synthesis of prolactin OR stop offending agent (e.g second gen antipsychotics) Should promote return of menses.

What is a cystocele? How is it properly evaluated? Tx?

Downward displacement of the bladder due to a defect in the anterior vaginal wall. 1. Inspect vagina at rest and with valsalva. 2. Q-tip test to evaluate bladder neck mobility (usually done by a urogynecologist). Pessary or surgery.

What is a vaginal vault prolapse? What is the biggest risk factor for this?

Downward displacement of the vaginal apex due to loss of apical support. —> usually follows hysterectomy

What adjuvant chemotherapy is used in endometrial cx? well known SEs?

Doxorubicin (cardiotoxicity) Cisplatin (nephrotoxic) Carboplatin and paclitaxel

Describe the lymphatic drainage and blood supply of the breasts.

Drains to nodes in axilla and clavicle Blood supply: internal thoracic, lateral thoracic, posterior intercoastal, and thoracoacromial arteries

Average Duration of 1st stage of labor in Nulliparous vs. Multiparous women? How many hours to determine prolongation of the latent phase? When would you say you have a protraction disorder of the active phase?

Duration of labor is typically shorter in the multiparous woman than in nulliparous women 1st stage (avg); nulli without epidural 7.7-13.3h nulli with epidural 10.2h multi w/o epidural 5.7 - 7.7h multi w/ epidural 7.4h Prolonged Latent Phase (0 to 4-6 cm) if: Nulliparous > 20 hours Multiparous > 14 hours Protraction of Active (fast) Phase (4-6 cm to 10 cm) if cervical change in: Nulliparous is < 1.2 cm/hr (minimum rate) Multiparous is < 1.5 cm/hr

During what phase of the menstrual cycle should PMS and PMDD occur in order to meet diagnostic criteria?

During the luteal phase

Diagnosis and treatment of uterine sarcoma?

Dx > 10 mitosis/10 high powered fields with aypia Tx TAH/BSO +/- lymphadenectomy peritoneal washings

What is the name for the pelvic pain that is associated with the menstrual cycle? Cause?

Dysmenorrhea - pain from endometrial prostaglandin release

Vulvar Intraepithelial Neoplasia (VIN): Main Categories Treatment

Dysplastic squamous lesions that may progress to cancer (like CIN) Two Main categories: VIN Unusual Type - Assoc with HPV - VIN 1 = condylomata acuminata - VIN 2 = lower 2/3 of epi involved - VIN 3 = full thickness of epithelium VIN Differentiated Type - <5% of VIN - Postmeno women - NOT assoc w/ HPV - maybe assoc with lichen scleroses Tx Note lesions are multifocal, requiring treatment of many areas According to size: Small, well circumscribed VIN = Wide Local Excision Multifocal lesons = Laser vaporization after r/o cancer Extensive (large) lesions : Skinning Vulvectomy Topical - Imiquimod, 5-fluorouracil

What are the signs and symptoms of a Pulmonary Embolism?

Dyspnea, chest pain, cough, syncope, hemoptysis. Tachypnea, tachycardia, apprehension, rales, hypoxemia. Note: 25% of unTx DVTs become PEs, if DVT is treated this goes down to 5% Note: half of women resenting with a DVT will have a silent PE

Most common indication for primary cesarean delivery

Dystocia

How do you manage *endometrial cells or Pt >40y w/ AUB*?

EMB & ECC

Presentation of cervical cancer (early, middle, late stage) On PE?

Early Asx irregular bleeding pink discharge Middle stage postvoid bleeding dysuria, hematuria Advanced post-coital bleeding severe pelvic pain weight loss LE edema (block lymphatics) PE abnormal looking cervix may appear friable and bleed easily on contact (although the most likely cause is chlamydia trachomatis) - there may be an ulcerating or fungating lesion or foul smelling serosanguineous vaginal discharge

Signs & Sx of Ovarian Cancer

Early stage Asx si/sx usually 2/2 mets: pelvic mass/pain bloating abdominal mass from omental caking ascites ventral hernia urinary freq and urgency early satiety N/V change in bowel habits increasing abdominal girth

What are the histologic subtypes of endometrial cancer ? Which are the most common?

Endometrioid is #1

What is the most common cause of infertility in menstruating woman over the age of 30 w/o a h/o PID?

Endometriosis

Anatomy of the cervix

Ectocervix cell type= stratified squamous epithelium - cervical biopsy Endocervical cell type =simple columnar (glandular) epithelium Squamocolumnar Junction (SCJ): transition from squamous epithelium of vagina to columnar epithelium of the uterus; most cervical cancers occur here - In nulliparous women, SCJ is usually at the ext cervical os - In pregnancy it migrates out and is visible to the naked eye (ectropion) Transformation Zone: the area between the original SCJ and the new SCJ

Pharmacotherapy Overflow Urinary Incontinence

Either decrease in function of bladder contraction or obstruction of bladder 1) Cholinergic agents 2) Bethanecol (structurally similar to acetylcholine) trial if atonic bladder = need to rule out obstruction Stimulates detrusor muscle contraction/bladder contraction a) Avoid with asthma, heart disease 3) Surgery to remove obstruction, if present 4) Alpha 1A blockers if outlet obstruction, males and females Receptors found in prostate tissue, but also bladder neck, urethra, periurethral tissue in women (will discuss in benign prostatic hypertrophy lecture)

What is the minimum fetal head decent in cm/hr during the second stage of labor?

Either protraction or arrest of descent when the fetal head descends: < 1 cm/hr in a nullip < 2 cm/hr in a multip Note: epidural can take longer

Type 2 endometrial cancer classically occurs in what type of population?

Elderly thin women with no signs of hyper-estrogenism

Most common population to get endometrial cancer

Elderly white women

Relative contraindications to MTX

Embryo cardiac activity High HCG (>5000 -> 14% failure rate) > 4 cm Refusal of blood transfusion

A postmenopausal woman has bleeding. What is the next step?

Endometrial Biopsy Even if cause thought to be polyp as visualized, still do the EMB! 5-10% postmeno bleeding will have endometrial cx

Most common cause of bleeding in post-menopausal women?

Endometrial atrophy

A 65 year old nulliparous woman presents with several months of postmenopausal bleeding. She reports her periods were irregular all her life until age 50 at which time she went through menopause. She had no further bleeding until now. Most of her bleeding has been light, but it seems to be gradually increasing. She has multiple medical problems including HTN, DM, hyperlipidemia and morbid obesity with a BMI of 45. She has always had normal pap smears. Her uterus is a normal size on exam, but her endometrium is thickened on ultrasound. What is the diagnosis?

Endometrial cancer

Most common gynecologic malignancy is...

Endometrial cancer

What are the 4 categories endometrial hyperplasia? Proliferation of what cell types? What percentage progress to cancer?

Endometrial hyperplasia is prolif of endometrial glands, may proceed to or coexist with endometrial cxwith Simple hyperplasia without atypia 1% progress to cx well differentiated Complex hyperplasia without atypia 3% progress Simple hyperplasia with atypia 10% progess Complex hyperplasia with atypia 30% progress Note: atypical hyperplasia is more likely to progress to cx in older women

Stromal plasma cells on biopsy

Endometritis

Layers of the uterus (inside to out)?

Endometrium - mucosal lining (columnar epithelium) Myometrium - 3 interlacing layers of smooth muscle (Outer longitudinal, Middle oblique, Inner longitudinal) Perimetrium - outermost serosal layer; the visceral peritoneum Mesometrium - portion of the broad ligament that supports the uterus laterally Parametrium - fibrous tissue that separates the supravaginal portion of the cervix from the bladder

What additional pelvic organ prolapse is commonly seen with a rectocele?

Enterocele

Why can patients with preeclampsia experience RUQ abdominal pain?

Epigastric or RUQ pain is the result of hepatocellular ischemia and edema that stretches the hepatic capsule. Elevated AST/ALT

List the different types and subtypes of ovarian tumors.

Epithelial: -Serous -Mucinous -Endometroid -Brenner tumor -undifferentiated Germ cell: -teratoma -benign (mature) Malignant (immature) -Choriocarcinoma -embryonal carcinoma -polyembryoma Sex cord tumors: -Granulosa theca tumors -Sertoli-leyding cell tumor -Gonadoblastoma Metastatic Unclassified

Side Effects of: Estrogen Progesterone

Estrogen - breast tenderness - Nausea - HA Progesterone - Depression - Acne - Weight gain - Irregular bleeding

Indications for colposcopy?

Evaluate abnormal pap smear results ASC-H, HSIL AGC 25+yo with ASCUS and HPV+ 25+yo LSIL Any age: 2 unsat pap cytology results

In hypothyroidism, how often do you check TSH if no change in medication is needed?

Every trimester; due to increased thyroxine requirements in advancing pregnancy (usually reqs 20% increase in levo dose once pregnancy is Dx)

Differential Diagnosis of postmenopausal bleeding?

Exogenous estrogens Atrophic endometritis/vaginitis Endometrial Cancer Endometrial/ Cervical polyps Coagulopathy Endometrial Hyperplasia

True or False: Fundal pressure should be applied during delivery with shoulder dystocia? Why or why not?

FALSE. DO NOT apply fundal pressure in shoulder dystocia. It causes further impaction of the shoulder behind the symphysis pubis.

Fetal Bradycardia: Define Mild vs Severe Causes

FHR < 110 Severe if <80 Causes: - maternal beta blocker admin - hypothermia - hypoglycemia - fetal heart block

Fetal Tachycardia: Define Mild vs Severe Causes

FHR > 160 for >10 mins - Mild = 161 to 180 bpm - Severe = ≥ 181 bmp Causes: - chorioamnionitis (intrauterine infection) = MC - maternal fever - severe fetal hypoxia (congenital heart disease, fetal arrhythmias, , etc.) - drugs

Declining ovarian steroid production disrupts the negative feedback inhibition of ___ secretion

FSH

How do you distinguish between hypogonadotropic hypogonadism and gonadal failure (POF)?

FSH will be low in hypogonadotropic hypogonadism bc low GnRH FSH high in POF because no ovarian follicle released thus no Estrogen negative feedback

FTS evaluates risk for _______. Quad screen evaluates risk for ______.

FTS evaluates for Tri 21 and 18. Quad evaluates for Tri 21, 18 and neural tube defects (NTDs).

What is the most common thrombophilia? It is most often diagnosed when an asymptomatic woman starts combination oral contraceptive pills and develops a blood clot.

Factor V Leiden (resistant to degradation by protein C)

What is Kallman Syndrome and how does it result in primary amenorrhea?

Failure of GnRH neurons to migrate --> amenorrhea defect in olfactory bulb --> anosmia

Meig syndrome (benign tumors of fibroblasts)

Female with ovarian FIBROMA tumor + Pleural effusion + Ascites

What fetal investigations are required for syphilis infection? Findings?

Fetal U/S Edema - hydrops - thick placenta

Fetal heart monitoring - what is the significance of sinusoidal heart rate pattern?

Fetal anemia (from any cause)

What happens in the SECOND stage of labor? start/end?

Fetal expulsion aka the pushing stage Start: Fully dilated cervix at 10 cm End: Delivery of the fetus

*Shoulder Dystocia*: Definition RFs Complications

Fetal shoulder is impacted behind the pubic symphysis after the fetal head has been delivered RFs: - Macrosomia - Gestational diabetes - Maternal obesity - Post-term delivery - Prolonged stage 2 of labor Complications: - Fetal humeral/clavicular fracture - Brachial plexus nerve injuries - Hypoxia/death

Meconium in the amniotic fluid may indicate what in the neonate?

Fetal stress and Meconium aspiration syndrome (MAS)

What is velamentous cord insertion?

Fetal vessels insert in the membranes and travel unprotected to the placenta. This leaves them susceptible to tearing when the amniotic sac ruptures.

Whose blood is lost with a ruptured vasa previa?

Fetal-placental circulation (more than maternal)

What is a complication of PROM at < 37 weeks?

Fetus is at risk of being born prematurely with its associated complications

What is the clinical presentation of rubella infection ?

Fever Malaise Coryzal symptoms Arthralgia Rash Lymphadenopathy

What are signs and symptoms of chorioamnionitis?

Fever > 100.4*F Leukocytosis Maternal/fetal tachycardia Uterine tenderness Malodorous vaginal discharge

criteria that support PID (add to less vague findings)

Fever > 101 Mucopurulent discharge Elevated inflammatory markers (ESR, CRP) known positive gonorrhea or chlamydia Fluid-filled tubes, free fluid, TOA on imaging

Define oligomenorrhea.

Fewer than 9 periods/yr OR Cycle length 35+days

Ovarian Sex Cord Stromal Tumors in Brief: Fibromas Pure Thecomas Sertoli-Leydig Cell

Fibromas - relatively common benign tumors composed of fibroblasts, predominantly u/l & generally hormonally inactive Pure thecomas - rare but may be hormonally active Sertoli-Leydig cell tumors - commonly present w/ masculinization (testosterone) & < 5% recur or metastasize

Uterine Factor: How can you assess anatomic obstruction as the cause for infertility?

First, checked semen and ovarian function and WNL Hysterosalpingogram - tubal patency and uterine cavity - dye into uterus and goes through tubes to peritoneal space - perform in early follicular phase to avoid pregnancy Note: subfertile woman that undergo HSG have higher preg rates than those that are subfertile and do not (Dx and therapeutic) Hysteroscopy Telescope with camera inserted to visualize uterine cavity - Dx and therapeutic - operation for: asherman's (lysis of adhesions), polypectomy, congenital anomalies (septum removal), submucousal fibroids Sonohysterogram - fluid inserted and with pelvic US - outlines polyps US - in office polyps Laparoscopy - mullerian malformations and endometriosis

What is a pessary made of?

Flexible silicone - does not cause allergic reactions or absorb odor.

Your patient is G3P2104 What is her likely OB history?

G3 - pregnant three times P2104: T2 - 2 parous experiences to term (37+) P1 - 1 parous experience preterm (20-36 weeks) A0 - 0 abortions L4 - 4 living children (So, if she has 4 children, 2 were likely a set of twins delivered preterm counting as one parous event)

Pseudogestational sac is...

Fluid/blood in uterine cavity on US in women with ectopic Deep Dive: - generally irregularly-shaped with pointed edges and/or filled with debris, sometimes referred to as 'beaking' - centrally located in the endometrial cavity, rather than - eccentrically located within the endometrium - displaces the anterior and posterior endometrial cavity surfaces -does not demonstrate a yolk sac - a double decidual layer is compatible with intrauterine pregnancy, but lack of this sign is not specific for pseudogestational sac - may be surrounded by a thick decidual layer - It should be distinguished from an intradecidual sign, which is also a sign of an early pregnancy.

When giving preconception counseling, what supplement do you advise that your patient take to prevent neural tube defects? What dose?

Folic acid supplements (400 mcg/d unless h/o a pregnancy with NTD then 4,000 mcg/d)

What are the two phases of the menstrual cycle and for how long do they occur? What is the dominant hormone during each phase? If the oocyte is not fertilized, what event marks the end of the luteal phase and the beginning of the follicular phase?

Follicular phase ~ 14 days (highly variable) Luteal phase = 14 days (not variable) Follicular - estrogen Luteal - progesterone then menses! Follicular phase is highly variable Luteal phase is not variable Note: the event that separates these two phases and typically occurs on day 14 of the cycle is Ovulation

Follow up of HRT

Follow-up 3 mo after initiation and annually

Frequency of colon cx screening (by modality)?

For prevention AND early detection: colonoscopy every 10 yrs sigmoidoscopy every 5 years CT colonography every 5 yrs Dbl contrast enema barium every 5 yrs For early detection only (annually): Guaic fecal occult blood test Fecal immunochemical test Stool DNA tests (every 1-3 yrs)

What type of pessary is best for a rectocele?

Gehrung pessary

Differential Dx of AUB in postmenopausal women? List the top 4.

Genital tract atrophy Endometrial polyps Endometrial hyperplasia Endometrial cancer

What are the maternal complications associated with pre/gestational diabetes?

Gestational HTN Preeclampsia Preterm Delivery Cesarean section Polyhydramnios Infections Impaired wound healing

What are the criteria for Preterm Labor?

Gestational age < 37 weeks with regular uterine contractions and: (one of the following) - Progressive cervical change - Cervix is 2 cm dilated - Cervix is 80% effaced - Ruptured membranes

When are corticosteroids given for preterm labor? What is one exception?

Given in preterm labor from 24-34 weeks Not given if mother has an infection

Describe the hormonal profile in those with Gonadal Dysgenesis (e.g. Turner Syndrome).

GnRH, FSH and LH high E and P low due to streak ovaries

HIV tx and goals

Goal to prevent progression, maintain high cd4 Antiretroviral therapy= mainstay of treatment

Diagnosis of endometrial hyperplasia: Gold standard? Other procedures ?

Gold Standard: Endometrial Bx (EMB) Other: Pap - if endometrial cells on pap in woman > 45 yo suspect endometrial cx Endocervical curretage (ECC) TVUS with thickened endometrial stripe in postmeno women Hysteroscopy with uterine curettage if bx inadequate

What infections are associated with preterm delivery?

Gonorrhea Chlamydia BV

Most important prognostic indicator in endometrial cancer is __________.

Grading

Ovarian Sex Cord-Stromal Tumors: Granulosa Cell Tumors Characteristics

Granulosa cell tumors - the most common malignant tumors in this category - indolent tumors that can recur 10-20 years after resection of the primary tumor - often hormonally active (estrogen) hence... - are assc. w/ endometrial hyperplasia/cancer --> should do endometrial biopsy and D&C

Gravidity vs Parity: -general definitions - TPAL

Gravidity is the number of times a woman has been pregnant regardless of the outcome (complete or incomplete, normal or abnormal; doesn't matter include it ALL) - so yes include abortions, hydatidiform moles, and ectopics Parity is when a pregnancy led to birth > 20 weeks or an infant > 500 g (dead or alive) Can further delineate using TPAL: Term (37 weeks min) Preterm (20-36 weeks) Abortions (spontaneous aka miscarriage or elective - must be less than 20 weeks to be included here) Living children Note: if gave birth to twins you would say G1P1 as the number of fetuses does NOT determine parity - you can break it down further with TPAL (lets say they were born at 35 weeks)... so G1P0102

Follow-up of Cervical Carcinoma

H&P plus pap every 3 mo for first 2 yrs every 6 months years 3-5 yearly after yr 5 CXR and CT abdomen yearly

What vaccine should be administered to baby shortly after birth?

HBV vaccine

What is the pathophysiology of fetal hemolytic anemia leading to death?

HDN: fetal anemia --> fetal hyperbilirubinemia + Kernicterus + heart failure, edema, ascites, pericardial effusion --> death

What is the management for Shoulder dystocia? HELPERRR

HELPERRR H - Call for Help E - Episiotomy L - Legs up (McRobert/s position) P - Pressure suprapubically E - Enter vagina for shoulder rotation (Woods corkscrew) R - Reach for posterior arm R - Rupture clavicle or pubic symphysis R - Return head unto vagina for C-section (Zavanelli)

Table - Perinatal Infections and how they are transmitted.

HIGH YIELD

THE cause of cervical cancer

HPV

ASC-US How do you manage ASC-US? In pregnant women?

HPV HR DNA reflex testing (preferred) Note: without genotyping OR Repeat Pap in 1y If HPV (-) 21-24 continue with normal screening 25+ yo repeat cotesting in 3y If HPV (+) 21-24, repeat pap 1y as they can clear it within 8-24 months 25+ yo, do colposcopy In pregos: Manage as if not pregnant except.. · No endocervical curettage or EMB · May *defer colpo until 6 wks postpartum* · Biopsy only suspected high grade lesions

Primary prevention for cervical cancer

HPV vaccination

MOA of HPV causing cervical cancer

HPV viral DNA E6 inhibits p53 which is a protein involved in apoptosis while viral E7 binds to the retinoblastoma protein and releases E2F which causes cellular proliferation

HSIL How do you manage HSIL? In pregnant women?

HSIL 25+ then LEEP or CKC or colpo regardless of HPV status If 21-24 or prego, do colpo first - if inadequate colpo or CIN2+ found, must do an excisional procedure

What is the utility of posterior shoulder delivery in shoulder dystocia? How is this performed?

Hand is inserted into vagina and posterior arm is pulled across chest, delivering posterior arm and shoulder. This creates a shorter distance between the anterior shoulder and posterior axilla, allowing the anterior shoulder to be delivered.

What are the late signs of congenital syphilis infection ?

Hearing loss with bone (frontal bossing, short maxilla, high palate arch, saddle nose deformity) and teeth abnormalities (Hutchinson teeth) Gummatous ulcers Periosteal lesions Paresis Optic atrophy Interstitial keratitis Sensorineural deafness

What is an objective measure of volume loss due to bleeding?

Hemoglobin or hematocrit levels

What is an enterocele? Why do some people get a congenital enterocele?

Herniation of the bowel and lining of peritoneal cavity through the cul-de-sac/pouch of Douglas. Because they have an abnormally deep cul-de-sac.

Drugs associated with IUGR

Heroin - questionable if solely heroin or lifestyle Methotrexate - 1st trimester Warfarin - assoc. with uterine hemorrhage Immunosuppressive drugs - cyclosporine, azathioprine, corticosteroids

What're the lab results associated with poor egg quality or decreased ovarian reserve?

High FSH/Estradiol + low AMH levels

What causes hyperemesis gravidarum?

High levels of hCG, estrogens, or both

What medication can be used to treat a lupus flare during pregnancy?

High-dose methylprednisolone

Two broad types of uterine sarcoma

Homologous and heterologous (i.e. cartilage, bone)

What do the inactive pills (last week of package) simulate?

Hormone withdrawal --> endometrial lining sheds (menses)

Semen Analysis: How to perform? Normal Results Abnl results and next steps

How to perform? Need at least 48 hours of abstinence exam sperm within 2 hours from collection two analyses at least 1-2 weeks apart Note: analysis reflects sperm production that occurred 3 months ago Normal Vlm > 2mL semen count > 20 million/mL motility > 50% with fwd movement morphology > 40% wnl If Abnl - refer to urology - smoking and alcohol cessation - avoid lube with intercourse Meds/Interventions - clomiphene or aromatase inhibitors to block neg feedback of estrogen to increase LH, FSH, and testosterone - Assisted reproductive technologies: intrauterine insemination, intracytoplasmic sperm injection (ICSI), in vitro fertilization

Pregnancy Hx, Sx, Clinical Signs

Hx: Amenorrhea from the last menstrual period (LMP) in a regularly menstruating woman Sx: - N/V (2-12 w GA) - Breast enlargement and tenderness (6 w GA) - Areolar enlargement and inc. pigmentation (6 w GA) - Colostrum secretion (16 w GA) - Urinary frequency, nocturia, and bladder irritability Signs: - Chadwick sign: Congestion and bluish discoloration of the cervix and vagina - Hegar sign: Softening of the cervix

What medication is used to lower blood pressure in eclampsia?

Hydralazine, Labetalol.

What are the features of congenital toxoplasmosis?

Hydrocephalus Intracranial calcifications Chorioretinitis Hepatosplenomegaly Thrombocytopenia IUGR Infected fetus can usually clear infection from organs, but CNS remains infected

SECONDARY AMENORRHEA Possible causes of ovarian dysfunction?

Hypergonadotropic hypogonadism: POF PCOS Surgical (b/l salpingo-oopherectomy aka BSO)

Coexistant maternal diseases that may cause IUGR according to HARM mnemonic

Hypertension Anemia Renal disease Malabsorption

Patients with hyperemesis gravidarum and gestational trophoblastic disease are noted to have ↓ TSH, ↑ Free T4, and symptoms of ____.

Hyperthyroidism

What type of acid-base disorder can occur with hyperemesis gravidarum? What is the treatment?

Hypochloremic alkalosis Tx IVF 5% dextrose Antiemetics

List common causes of anovulation and their Tx.

Hypogonadotropic(low FSH/LH) Hypogonadism (low E/P) (hypothalamic amenorrhea) - stress, malnutrition, overexercise - Tx: lifestyle and ovulation induction Hyperprolactinemia - Tx: bromocriptine (DA agonist --> DA suppresses Prl) +/- ovulation induction Normogonadotropic normogonadism (PCOS) - ovulation induction agent +/- metformin and weight loss Hypergonadotropic hypoestrogenic aka premature ovarian failure (POF) - Tx: IVF with donor eggs

how often do patients get the Depo Provera injection?

IM every three months

For how many years is the progestin IUD approved for by the FDA? How about the copper IUD? And the implant? ACOG recs?

IUDs: Mirena (releases 20 micrograms of LNG/d) and Kyleena (17.5/d) IUD FDA approved for 5 years but ACOG say 7 years Liletta (18.6/d) is 4 yr Skyla (14/d) is 3 yr Copper FDA says 10 ACOG says 12 yrs Note: trials are being done on the other hormonal ones, data not yet available Implant is FDA approved for 3 years, ACOG says okay up to 5 years

What are the features of congenital CMV infection?

IUGR Microcephaly Intracranial calcification Chorioretinitis (blind) Sensorineural deafness hemolytic anemia --> hepatosplenomegaly Skin Rash pneumonitis Intellectual disability

Pregestational diabetes causes [macrosomia or IUGR], especially due to concurrent maternal vascular disease.

IUGR - intrauterine growth restriction

Absolute contraindications to MTX?

IUP Immunodeficiency Anemia, leukopenia, thrombocytopenia (<100k) Sensitivity to MTX Active renal/pulmonary disease (not asthma) Active PUD (MTX may worsen) Hepatic/renal dysfunction Breastfeeding Ruptured ectopic hemodynamically unstable Cannot follow up

parenteral pid tx

IV cefotetan and doxy

If HSG shows tubes are blocked, what is the most effective treatment?

IVF - egg cells fertilized outside the uterus after ovarian stim and egg harvested, transfer in embryo Note: increases chances of multiple gestations also 20% effective but can also try: lap to lyse adhesions, neosalpingostomy (open tubes)

What qualifies as arrest of dilation in the first-stage?

If contractions are deemed to be adequate (>200 montevideo units) and pt is 6+ cm dilated (i.e. active phase) with ROM, arrest of dilation is determined when there is no cervical change after 4h If contractions are not adequate (<200), pt has 6h before labeled as arrest of labor. You do not diagnose arrest of labor before you reach 6 cm as patient is likely still in the laten phase - can use oxtocin/AROM to augment labor

What do you do if US shows cystic mass?

If cystic mass --> aspirate - if fluid is cloudy/bloody --> excisional biopsy and imaging - if clear and cyst resolves --> monitor - if cyst remains --> excisional bx

When can a neonate get varicella zoster infection from his/her mother ?

If maternal infection occurs 1-4 weeks prior to delivery to 1 week postpartum

Discuss the need for immunization during peuperieum.

If mom is non-isoimmunized Rh neg and baby is Rh pos, give 30 micrograms of anti-D immunoglobulin (RhoGam) within *72h* of delivery. If *Rubella* non-immune, vaccinate mom prior to discharge If not given during pregnancy, offer *influenza and Tdap*

When should you do a sterile speculum exam prior to a digital vaginal exam?

If you suspect: ROM PTL placenta previa (good amt of VB)

If pt palpates a mass that a clinician does not feel, what is indicated?

Imaging: - Over 40 do diagnostic mammo - < 40 start with US as the breast tissue is too dense for mammo US helps differentiate cystic vs solid mass

What is the treatment for appendicitis in pregnancy?

Immediate appendectomy: - Laparoscopy in early pregnancy - Laparotomy in later pregnacny

What is the management for uterine rupture?

Immediate laparotomy and delivery. May require a cesarean hysterectomy if uterus cannot be reconstructed.

What tests can be used to monitor fetal health? When are these generally employed?

In T3 can use: fetal movement (kick) counts non-stress test (NST) contraction stress test (CST) biophysical profile (BPP) the modified BPP (mBPP) Doppler US

Summary of the menstrual cycle.

In a nut shell... Follicular Phase: 0- ~14d pulsatile GnRH --> FSH and LH high FSH stimulates maturation of follicle ----> estradiol --> endometrium proliferation LH surge is Ovulation Luteal Phase - corpus luteum secretes progesterone --> maturation of endometrium for implantation, highly vascular and incr glandular secretion

What do you do for a non-palpable mass found incidentally on imaging?

In general, either follow with imaging or biopsy - depends on how suspicious it looks (if spiculated bx)

What is the easiest way to examine a patient with pelvic floor dysfunction?

In lithotomy position, using only bottom 1/2 of the vaginal speculum. This will allow you to isolate the anterior and posterior walls one at a time. Examine while the patient is relaxed and when bearing down. Should also be examined standing (seems awkward though and I haven't seen this in practice). **May also want to do rectovaginal exam.

How does a *persistent HPV infection* lead to *CIN*?

In the transformation zone it results in some cells progressing toward malignant change

Breast Cancer In Situ What is in situ breast cancer? Tx?

In-situ cancer is contained within the ducts or lobules. It includes an abnormal number and morphology of cells lining the duct, but does not extend into the breast tissue. Often appears as a new pleomorphic calcifications or linear branching, but it is not usually palpable. Currently ductal carcinoma in-situ is treated like cancer so that it does not progress to invasive cancer. It is usually not life-threatening and some may never progress, but we treat all of them anyways!

Indications for Cone or LEEP

Inadequate view of TZ on colpo Positive ECC 2+grade discrepancy btwn colp and pap CIN2+ Tx Adenocarcinoma in situ Tx If cx cannot be excluded after colpo, bx, and ECC

What complication of pelvic organ prolapse constitutes a surgical emergency?

Incarceration of the prolapsed contents! - Prolapsed bladder may fill with urine - Bowel may become necrotic - Uterus may swell

Lab values in PCOS?

Increased LH/FSH ratio (normally 1:1) Increased Testosterone and DHEAS, androstenedione Decreased SHBG (sex hormone binding globulin - binds testosterone in circulation) Increased AMH (because more small follicles which secrete this) Normal other hormone levels (to exclude other causes) High HBA1C/abnormal glucose tolerance if insulin resistant

Epidural analgesia effects on labor

Increased duration of labor Increased incidence of: - Chorioamnionitis - Low-forceps procedures - Cesarean deliveries - Maternal pyrexia

Effect of ERT on CHD risk?

Increased risk

Health Risks assoc w/ HRT (combined)

Increased risk of: -Thromboembolic disease: PE, VTE, MI, Stroke -Gallbladder disease (cholelithiasis and cystitis) -Invasive breast cancer - Endometrial hyperplasia/cancer Note: estrogen creates a hypercoagulable state d/t increased production of hepatic coagulation factors

Risks associated with COCPs?

Increases risk of: thromboembolism/stroke (3/10k) MI (in smokers >35) Mood disturbance Migraines

What tocolytic agent is used for GA < 32 weeks?

Indomethacin

Meconium aspiration syndrome (MAS) Sx

Infant tachypnea, costal retractions, cyanosis, coarse breath sounds, etc. Leads to respiratory distress and lung damage --> pulmonary HTN - may require ECMO to bypass lungs and oxygenate baby

Septic abortion: Dx Mgmt

Infected POC, usually polymicrobial - infx can spread from endometrium through to the periteneum Less likely to occur with spont abortion, more likely to occur with induced abortion Dx - fever, hypoTN, tachy, generalized pelvic discomfort, uterine tenderness, peritoneal signs; PE malodorous vag d/c, leukocytosis cervical and blood cultures, IV antibiotics (anaerobic coverage), D&C promptly after starting Abx and stabilizing the patient.

morbidity of STIs

Infertility, increased ectopic rate, chronic pelvic pain, fetal morbidity, cancer, immunocompromised state, severe systemic infection

Inflammatory Breast Cx What is inflammatory breast cancer? Si/Sx Workup/Dx

Inflammatory breast cancer is a rare type of breast cancer that develops rapidly, making the affected breast red, swollen and tender. Inflammatory breast cancer occurs when cancer cells block the lymphatic vessels in skin covering the breast, causing the characteristic red, swollen appearance of the breast. Si/Sx -RAPID change in the appearance of one breast, over the course of several weeks -thickness, heaviness or visible enlargement of one breast -discoloration of the breast -warmth of the affected breast -dimpling or ridges on the skin of the affected breas -tenderness, pain or aching -enlarged lymph nodes under the arm, above the collarbone or below the collarbone -flattening or turning inward of the nipple -doesn't commonly form a lump Workup -diagnostic mammogram -punch biopsy of skin -core biopsy if mass evident Tx Usually, chemotherapy first followed by mastectomy and systemic treatment.

Treatment in cases of rape

Infx prophylaxis for gonorrhea, chlamydia, trichomoniasis: ceftriaxone IM + azithro po x1 or doxy BID x7d + metronidazole po x1 Hep B vaccine anti-virals for HIV prophylaxis Pregnancy prevention - Plan B - OCPs - Copper IUD

When should the antibody screen be done?

Initial prenatal visit, and at 28 weeks

What test determines the number of fetal RBCs in the maternal circulation? This number determines the dose of _____.

Kleihauer-Betke test Determines the dose of RhoGAM

What medication should be given to A2 gestational diabetics during labor?

Insulin drip

Insulin requirements ____ during the second trimester due to the antagonistic effect of pregnancy hormones. Immediately following delivery, insulin requirements dramatically ____.

Insulin requirements *increase* during the second trimester due to the antagonistic effect of pregnancy hormones. Immediately following delivery, insulin requirements dramatically *decrease*.

What are the clinical signs and symptoms of Pruritic urticarial papules and plaques of pregnancy (PUPP)?

Intensely pruritic cutaneous eruption that usually appears late in pregnancy. Erythema, vesicles, and eczemtaous target lesions may be seen. Begins on the abdomen and spreads to arms and legs.

Fetus has not met its growth potential, with estimated fetal weight at or below 10th percentile for gestational age with altered body composition

Intrauterine growth restriction (IUGR)

What are the two most common causes of anemia during pregnancy and the puerperium?

Iron deficiency & acute blood loss

What are Braxton Hicks Contractions?

Irregular, nonrhythmic, usually painless contractions that begin as early as gestation and increase as term approaches. May be confused as true labor.

First things to do in management of possible ectopic ?

Is pt hemodynamically stable? is the ectopic ruptured? Rhogam if pt is D negative

Is mastalgia often tied to hormonal changes?

It can be (cyclical) or it may be unrelated (noncyclic)! Cyclic is more common.

Paget Disease what is mammary paget's disease? How does it present? Tx?

It is almost always associated with an underlying breast cancer (92-100% of cases) -50% will present with a mass -thickened, eczematoid crusted lesion with irregular borders -scaly, erythematous, crusty pigmentation and thickened plaques on the nipple -advanced lesions show skin thickening, redness, erythema, erosion of the nipple and scaling around the nipple-areola Tx Surgery, radiation and/or chemo/endocrine therapy

How frequent should you perform cervical checks during labor?

Keep to minimum - use sterile gloves, diameter of internal os assessed with index and middle finger 1st phase: Latent phase - q4h active phase - q2h 2nd phase: qh Indications for cervical check: epidural feels urge to push any non-reassuring FHT

D antigen status: - Kell ___ - Duffy ___ - Lewis ___

Kell kills Duffy dies *Lewis lives*

your patient wants to be on birth control but does not want to have associated symptoms, as in OCP. What options does she have?

Patch or ring

___________ is very effective for treating menorrhagia.

LNG-IUD

General significance of the following results: LSIL HSIL CIN 1 CIN 2 CIN 3

LSIL - generally associated with transient HPV infection HSIL - associated with persistent and transforming infection and cancer risk CIN 1 - a manifestation of acute HPV infection and has a high rate of regression to normal cells - usually managed expectantly CIN 2 - represent a mix of low-grade and high-grade lesions CIN 3 and AIS - clearly cancer precursors - Progression from persistent infection to cancer is slow, and the time course from CIN 3 to invasive cancer averages between 8.1 years and 12.6 years Note: the threshold for treatment is CIN 2+ except in a few special populations, particularly young women and pregnant women

How should the fetus be delivered if the mother gets primary HSV infection within 6 weeks before delivery?

LTCS to decrease the high risk of vertical transmission assoc with SVD

List common complications/signs of anorexia.

Lanugo: fine body hair Bradycardia Constipation Low BP Hypothermia Higher mortality risk

An infant with estimated fetal weight that is >90th percentile

Large for gestational age

What is the Zavanelli maneuver?

Last resort in management of shoulder dystocia: The fetal head is returned to the uterus by reversing the cardinal movements of labor. At this point, a C-section can be performed.

Types of male condoms?

Latex - allergies Synthetic (polyurethane) - non-allergenic Natural (lamb skin) - nonallergenic - least protective against STIs

Most common pelvic tumor in women?

Leiomyoma

A 38 yo African-American woman presents with a long history of very heavy, but regular menstrual cycles. They have been worsening as she gets older. She also notes a sense of "heaviness" in her pelvis during menses. She has never had an abnormal pap smear. On examination, cervix appears normal. Her uterus is noted to be greatly enlarged on bimanual exam and a firm mass can be felt below her umbilicus on abdominal exam. Ultrasound shows several solid masses in the pelvis, most likely arising from the uterus. Endometrial biopsy is negative for malignancy. Laboratory studies show severe anemia with a Hemoglobin of 6. She has no acute symptoms from her anemia but has noted chronic fatigue. What is the diagnosis?

Leiomyoma aka fibroids

___________ may help prevent recurrence of polyps.

Levonorgestrel (any progestin)

Treatment for hypothyroidism during pregnancy?

Levothyroxine replacement Note: Usually requires 20% increase in levo dose once pregnancy is Dx

When is *HPV 16/18 genotyping* indicated? What is it *NOT* used for?

Limited to *(+)* HPV HR DNA screen *NOT* used for HPV *f/u for ASCUS*

Protective against endometrial cancer

Progesterone

Lichen Simplex Chronicus (LSC) What is it? What does it look like (si/sx)? Dx Tx

Localized variant of atopic dermatitis, one of the most common causes of vulvar itching History of contact dermatitis, allergies or asthma Chronic irritant --> itch --> scratch --> raised, white, thick lesions with labial edema Dx with Bx, microscopy shows: acanthosis and hyperkeratosis Tx attempt to ID any irritants Topical steroid cream and anesthetics

Where are the Bartholin glands located? Function?

Located on each side of the vaginal orifice at 4 and 8 oclock, typically non-palpable unless a cyst (ASx)or abscess (painful) is present Function: secrete mucous for vaginal lubrication

What is the term used to describe vaginal discharge (endometrial sloughing) during puerperium? Give time frames for the different types.

Lochia rubra --> serosa --> alba (see table)

Osteoporosis/Osteopenia: Defined Pathophys Consequences

Low bone density per densitometry (DEXA scan) of hip and spine: -Osteopenia: T-score < -1 to -2.5 -Osteoporosis: T-score ≤ -2.5 Declining estrogen leads to decreased bone formation Low impact fractures

Climateric (aka perimenopausal) menstrual irregularity tx agents?

Low-dose OCP Progestin IUD Endometrial ablation GnRH Agonist Hysterectomy in refractory cases

At birth, what are some changes that occur immediately in the fetal lungs?

Lungs become aerated, pulmonary vascular resistance decreases --> causing pulmonary blood flow to increase

Immunologic causes of spontaneous abortion?

Lupus anticoagulant Anticardiolipin Ab (antiphospholipid syndrome)

Endometrial cancer is part of what hereditary cx syndrome?

Lynch syndrome

When is a breast MRI indicated?

MRIs have no role in screening for the average risk patient d/t higher rate of false positives. -BRCA 1 or 2 gene mutation -first degree relative w/ BRCA 1 or 2 -increased lifetime risk -h/o mammographical occult breast CA -radiation therapy to the chest wall at a young age

RFs of syphilis

MSM HIV+ High-risk sexual behavior Commercial sex workers Geographic Area of high prevalence - South Carolina Men 20-29 Racial/ethnic minority status

Fixed multiple dose MTX regimen for ectopic

MTX 1 mg/kg IM on D1, 3, 5, and 7 Alternate with folinic acid 0.1 mg/kg IM on D2, 4, 6, 8 Measure HCG on MTX dose days and continue until HCG decr by 15% from prior measurement - If >15% decr, d/c MTX and measure HCG weekly - If no decr after 4 doses -> surgery

Endocrine causes of spontaneous abortion?

Progesterone deficiency PCOS uncontrolled DM

How are vaginal pessaries used properly?

MUST be used in combo with topical estrogen to prevent vaginal ulceration. *Good idea to use vaginal estrogen for several weeks prior to starting with pessary. Type and size of pessary fitted individually in office.

What is the major fetal complication of gestational diabetes?

Macrosomia

What agents are used for tocolysis?

Magnesium sulfate Nifedipine Ritodrine, Terbutaline Indomethacin

What is the treatment of choice for seizure prophylaxis in pregnant female with pregnancy induced HTN?

Magnesium sulfate (MgSO4)

What medication is used for seizure prophylaxis in preeclampsia? Is this also effective treatment for HTN?

Magnesium sulfate. It does not treat HTN.

Mechanism of Action of POPs vs the COCPs

Main difference is: - POPs inhibit LH thus no surge thus no ovulation but still follicle formed just not released - COCPs estrogen inhibits FSH so no follicle even

Types of emergency contraception?

Main types: Progestin pills (one or two pill regimens aka Plan B) Copper IUD Ulipristal Acetate (Ella) Yuzpe Regimen (particular doses of everyday OCPs, only some brands and different dosing regimens)

How do we diagnose he majority of breast cancers?

Mammography

How do you manage *adolescent women* who were *inadvertently tested*?

Manage according to *21-24y/o* guidelines

What is the first step in management of postpartum hemorrhage?

Manually compress and massage the uterus.

Types of Hormone Replacement Therapy (HRT)

Many natural and synthetic estrogen and progesterone derivatives exist: -Estrogen only (indicated in women s/p hysterectomy) -Progesterone only -Combine E:P preparations (progesterone component needed in those with a uterus to protect the endometrium from unopposed estrogen which can cause hyperplasia/cancer) - Oral, trans-dermal and trans-vaginal options exist

What investigations are necessary with a potential rubella infection?

Maternal : Rubella serology antibody IgG , Ig M Fetal : U/S scan Rubella RNA in CVS, amnio, and fetal blood can confirm

What is the management of parovirus in pregnancy?

Maternal : symptomatic relief Fetal : Intrauterine blood transfusion

Segments of the broad ligament

Mesometrium - Surrounds the uterus; largest subsection; also encloses the proximal part of the round ligament Mesovarium - associated w/ the ovaries, enclosing its neurovascular supply but does not cover the surface of the ovary itself. Mesosalpinx - Originates superiorly to the mesovarium, enclosing the fallopian tubes.

What maternal investigations are done when toxoplasmosis infection is suspected?

Maternal Blood Ab levels against T gondii Ig G: represents immunity/ previous infection Ig M: active infection, peak after 1 month In Depth Dive: IgM neg, IgG pos - indicative of remote infection and pose no concern for fetal transmission in an immunocompetent woman Both neg - indicate either the absence of infection or a recent acute infection without enough time for seroconversion Both pos - the patient has had either a recent infection or a false-positive result. If acute infection is a possibility, serologic testing should be repeated in 2-3 weeks to look for an increase in IgG antibodies consistent with recent infection.

What are the maternal complications of hyperthyroidism during pregnancy? Fetal complications?

Maternal complications: - Preeclampsia - Heart failure - Stillbirth, preterm labor - thyroid storm Fetal complications: Neonatal thyrotoxicosis (1%) due to placental transfer of thyroid stimulating Abs Fetal goiter/hypothyroidism secondary to PTU admin

What are contraindications to using Nifedipine (CCB) as a tocolytic agent?

Maternal hypotension, cardiac disease Use with caution in renal disease Avoid concomitant use with MgSO4

What is McRoberts Maneuver?

Maternal thighs are sharply flexed against maternal abdomen. This flattens the sacrum and the symphysis pubis and may allow the delivery of the fetal shoulder

List common Maternal and Fetal CONTRAindications for IOL.

Maternal: - placenta or vasa previa - prior uterine surgery (controversial) - classical CS - active genital herpes infection - previous myomectomy Fetal: - in distress - transverse lie - cord prolapse

List common Maternal and Fetal indications for Induction of Labor (IOL).

Maternal: Fetal demise prolonged pregnancy chorio severe preE or eclampsia DM, certain renal, pulm or cardiac diseases, APA syndrome, etc. Fetal: IUGR Abn fetal testing infection Isoimmunization Oligohydramnios postterm (~41 weeks+) PROM

What investigations are needed for suspected CMV infection ?

Maternal: CMV antibody IgG & IgM Note: viral latency with reactivation --> immunity does not prevent recurrence, reactivation, exogenous infx, congenital infx, infection from a different strain Fetal Investigations : U/S scan for fetal anomalies - microcephaly - ventriculomegaly - intracranial calcifications amniocentesis or fetal blood PCR (6-9 weeks s/p maternal infx)

What are complications of uterine rupture?

Maternal: Hemorrhage, hysterectomy, death Fetal: Permanent neuro impairment, cerebral palsy, death.

What are the maternal and fetal side effects of Ritodrine & Terbutaline?

Maternal: Pulmonary edema, tachycardia, headaches Fetal: Tachycardia (beta agonist)

Cost effectiveness of medical vs surgical tx for ectopic

Medical is cost effective if HCG < 1500 Surgery is better if resolution time is prolonged or there is high relative risk of medical management (high/incr HCG, cardiac activity)

CNS Symptoms of menopausal women

Memory loss Mood changes Sexual dysfunction Hot flashes (Sleep cycle and quality) Increased appetite Weight gain

At what age does menarche typically occur and what hormone is responsible?

Menarche typically begins at age 12 and is caused by an increase in estradiol

Which medication for hyperthyroidism is safe in T2, T3?

Methimazole - readily crosses placenta - assoc with aplasia cutis (absence of skin, with or without the absence of underlying structures, most common on scalp); hence, use after organogenesis is complete

Define Krukenburg tumor.

Mets to the ovaries, primary is usually GI

Hormonal (progestin aka levonorgestrel aka LNG) IUD: Types? Mechanism? Best For? Disadvantages and Contraindications?

Mirena, Kyleena, Liletta, and Skyla Types (see column 2) Mechanism (column 3) Best For (column 4) Disadvantages and Contraindications (column 5)

What are the maternal complications which can arise from rubella infection in pregnancy ?

Miscarriage Pneumonia Arthropathy Encephalitis ITP

What are the complications of CMV infection in pregnancy ?

Miscarriage Stillbirth Congenital CMV

In a patient with hyperemesis gravidarum, which medical conditions should be ruled out?

Molar pregnancy (high hCG) Thyrotoxicosis GI etiology

Pathophysiology of PCOS.

Most common endocrine problem in reproductive women Familial link, but likely polygenic Some disturbance leads to hypersecretion of LH and overproduction of androgens. This results in multiple small cystic follicles that produce a sufficient estradiol ouput to induce low-level positive feedback on LH (less progesterone) and negative on FSH (makes follicles more resistant). This stimulates more androgen production (which is also stimulated by the high AMH from small follicles) that causes more insulin resistance and keeps the follicles from developing more in a vicious cycle. Furthermore there is less SHBG so more free androgen, which is associated with hyperinsulinemia (which again, stimulates androgen production and maybe LH too)

Are kegel exercises helpful for pelvic organ prolapse?

Not really - they are helpful earlier on in course, after pelvic surgery or childbirth.

Most women will spend _____ of their lives in menopause.

Most women will spend 1/3 of their lives in menopause

How are ovarian and fallopian cancers spread?

Mostly by peritoneal fluid Note: recent data suggests many serous epithelial ovarian cancers have fallopian tube precursor lesion

Any pregnancies after the first pregnancy, a woman is considered a ____________gravida.

Multigravida

What are risk factors for developing pelvic relaxation/POP?

Multiparity Operative vaginal delivery (often due to larger babies) Obesity (extra pressure on pelvic floor) Advanced age Prior pelvic surgery (direct damage to mm and fascia) Estrogen deficiency (urogenital atrophy)/menopause Constipation Less commonly.... - Neurogenic dysfunction of the pelvic floor - Connective tissue disorders (genetic predispo) - Chronic increased intra-abdominal pressure.

Effects of estrogen

Multiple effects throughout the body and plays role in other organs

Single vs multiple doses

Multiple more effective (possibly), but more adverse effects 1 dose is similar to 2 dose in efficacy with similar risk of adverse effects 2 dose may be better if HCG > 3600 initially

What is a contraindication to using Magnesium sulfate as a tocolytic agent?

Myasthenia gravis

chlamydia dx

NAAT

gonorrhea diagnosis

NAAT: PCR analysis of cervical, urine, or vaginal swab culture is less sensitive but *needed for abx susceptibility* DNA probe very sensitive/specific

List 2 things that are protective against leiomyomas.

Parity (pregnancy) and smoking

Progesterone only preparations for HRT

Provera: medroxyprogesterone acetate (MPA) Norethindrone acetate

List three common side effects of COCPs.

Nausea HA Bloating

Bethesda classifications of cervical cytology (pap results)

Negative for intraepithelial lesion or malignancy (normal) Atypical squamous cells of undetermined significance (ASCUS) Atypical squamous cells cannot rule out high grade lesion Low grade squamous intraepithelial lesion High grade squamous intraepithelial lesion Carcinoma in situ Squamous cell carcinoma Adenocarcinoma Atypical glandular cells

gonorrhea organism

Neisseria gonococcus - gram-negative diplococci

How is congenital CMV treated?

Neonatal ganciclovir (BJOG recent recommendation) postnatal valganciclovir/ganciclovir treatment should be considered and commenced within the first 4 weeks of life. There is evidence that treatment can reduce or prevent progression of SNHL and improve long‐term neurodevelopmental outcomes in some infants

SECONDARY AMENORRHEA Possible causes of pituitary dysfunction?

Neoplasms: prolactinoma, adenomas Lesions: damage from anoxia, thrombosis, hemorrhage --> low FSH/LH AACTH TSH so can have hypothyroidism and adrenal insufficiency - Sheehan syndrome - Simmonds disease

Adjuvant therapy for Grade I, stage IA?

No further Tx beyond the staging procedure

What is the overall etiology of patients that do not have breast development, but do have a uterus?

No ovarian estrogen (gonadal dysgenesis like streak ovaries in Turner syndrome)

A woman with an IUD has become pregnant and wishes to carry out the pregnancy. If the IUD is left in place, will the baby be malformed?

No, but there is a greater chance of preterm labor

What is the first assessment of fetal well-being if fetal compromise is suspected?

Nonstress Test: FHR tracing analyzed over 20 minutes for "reactivity" aka appropriate changes in HR secondary to fetal movement Reactive if: 2 accelerations within 20 minutes that are ≥ 15 bpm above the baseline and last at least 15 seconds

How can you differentiate between "normal" edema of pregnancy vs. abnormal?

Normal - edema of feet and ankles during the day Abnormal - generalized edema of face, hands, abdomen, ankles

Pharmacotherapy Stress Urinary Incontinence: Alpha-agonists

Not recommended 1) Pseudoephedrine a) Dose = 15 to 60 mg three times daily 2) Phenylephrine a) Dose = 10 mg four times dailiy b) Mechanism of action = alpha-adrenergic stimulation on detrusor muscle (inhibition) and proximal urethra c) Efficacy: mildly effective d) ADRs: Many contraindications/precautions Such as ??

FIGO Staging of Vaginal Cx

Note: having CIN or VIN is a risk factor

A woman who is not pregnant and has never been pregnant is called _________gravida.

Nulligravida

A woman who has never given birth _________para.

Nullipara Clarification: So lets say the the woman had a spontaneous abortion (so less than 20 weeks otherwise would have called it a stillbirth), this does not count in terms of para and thus the woman is still considered a nullip

Pattern of fetal descent in 2nd stage of labor (hours from fully dilated to expulsion).

Nulliparous < 2 hours (3 if epidural) Multiparous < 1 hour ( 2 if epidural)

Type 1 endometrial cancer classically occurs in what type of population?

Obese women with hyperestrogensim and a metabolic syndrome picture

Biggest risk factors for endometrial cancer

Obesity and unopposed estrogen (i.e. estrogen therapy)

If a patient is suffering with pelvic pain secondary to the pelvic organ prolapse, is this likely to be relieved with surgery?

Often no....this is because the pain is typically d/t muscle spasm or strain, vaginal atrophy, or chronic conditions such as chronic low back pain and fibromyalgia.

DSM-V Criteria for PMDD

One or more sx: - feeling sad, hopeless or having self-deprecating thoughts - anxiety or tension - mood lability and crying - persistent irritability, anger and interpersonal conflicts Also one or more of these (total of 5 between the two lists): - problems concentrating change in appetite - anhedonia - decr energy - overwhelmed - physical sx like breast tenderness, bloat - sleep disturbances

What maternal investigations are done for suspected tertiary syphilis?

One screening test followed by one confirmatory test: 1. Non-treponemal (screening) test : RPR or VDRL Then.. 2. Treponemal (confirmatory) test : FTA-ABS or MHA-TP

Causes of elevated AFP?

Open neural tube defects Omphalocele & other abd wall defects Multiple gestations Duodenal atresia Fetal death Placental abnormalities

Approximately for how many days does a menstrual cycle occur? How many days does a period normally last? What are the heaviest day of this period? What is still considered to be normal menstruation and not AUB?

Optimal q28/4-5, with days 2-3 being the heaviest Normal q21-35/3-7

A 30 year old patient presents to your office complaining of irregular periods. Her periods have always been irregular, and she has felt this was just "normal for me." She is now seeking care as she has recently gotten married and thinks she may want to conceive in the next year or two. She reports that she often goes 3-5 months without a period and then will have a heavy bleeding episode. She is obese, BMI 40. She has "pre-diabetes" and also recently got diagnosed with hypertension. On ROS, she notes some abnormal hair growth on her face. She has recently had some lab work that included thyroid testing and prolactin. These were within normal range. Her examination is normal. An endometrial biopsy is done and is normal. You counsel her on diet and exercise with a goal of controlled weight loss. She will have frequent office visits during this time for progress checks and will also keep a menstrual calendar to evaluate changes in her cycles. What is this?

Ovulatory dysfunction (AUB-O)

_________ stimulates milk letdown/ejection whereas _________ stimulates milk production.

Oxytocin is ejection, prolactin is production Note: progesterone inhibits production

P450 _________ decrease the efficacy of OCPs. Give examples.

P450 inducers (increase the clearance of OCPs), examples: phenytoin rifampin griseofulvin carbamazepine alcohol barbituates

Differential for AUB?

PALM(structural lesions) Polyps (would write diagnosis as AUB-P) Adenomyosis Leiomyoma (aka fibroid) Malignancy (endometrial, cervical, estrogen producing ovarian tumors i.e. granulosa-theca cell tumors) COEIN (non-structural lesions) Coagulopathy Ovulatory dysfn Endometrial Iatrogenic Not yet classified Atrophy of the endometrium (postmenopausal patients)

syphilis tx in primary or secondary

PCN G benzathine once

List possible findings on PE that may help in establishing the following causes of AUB: PCOS Fibroids Thyroid d/o Hyperprolactinemia COagulopathy

PCOS: obesity, hirsutism, acanthosis nigricans Fibroids: bimanual exam reveals bulky irreg shaped uterus/discrete fibroids Thyroid d/o: exopthalmos, goiter, delayed LE DTRs, dry skin/hair Hyperprolactinemia: visual field defects, galactorrhea Coagulopathy: petechiae

Investigations for Cervical Cancer

PE (often under anesthesia) to evaluate tumor and spread - bimanual - speculum - rectovaginal - palpate groin and supraclavicular nodes Colposcopy w/ biopsy and ECC - some cases are detectable on smear - most cases are detected in pre-malignant phase on cervical screening In those with PCB, smears should only be taken if they are overdue. If there is persistent unexplained PCB then refer for colposcopy UPT- urine pregnancy test and rule out infection with vaginal FBC - anaemia for irregular bleeding Once diagnosis confirmed - hysteroscopy, cystoscopy and sigmoidoscopy for spread to uterine lining, bladder or bowel - FBC, U+E and LFTs - MRI of the pelvis - CT abdo and chest all used for staging.

complications of chlamydial infection?

PID neonatal conjunctivitis endometriosis infertility

DDX for ectopic pregnancy?

PID ovarin torsion ruptured ovarian cyst tubo-ovarian abscess threatened abortion appendicitis degenerating fibroid

When is a cervical biopsy contraindicated?

PID and cervicitis NOT pregnancy

What is the main difference between Premestrual Syndrome (PMS) and Premenstrual Dysphoric Disorder (PMDD)?

PMS - symptoms DO NOT interfere with daily functioning PMDD - symptoms DO interfere with daily functioning

What are two causes of prolonged rupture of membranes?

PROM Abnormally long labor (not PROM)

What are the complications of maternal anemia during pregnancy?

PTL IUGR Low birth weight (LBW) + need for blood transfusion if blood loss in delivery

What are possible complications of trichimoniasis in pregnancy? Treat with?

PTL PPROM Tx: MTZ

signs of PID

Pain with intercourse; Post-coital bleeding Discharge Fever, chills (see more in PID vs straight up gonorrhea) Lower abdominal pain/pelvic pain possibly N/V CMT

Third trimester bleeding, think: - Painful bleeding = ______ - Painless bleeding = ______

Painful = placental abruption Painless = placenta previa

After age 30 how often are pap smears performed?

Pap test PLUS HPV cotesting every 5 years Note: If no HPV testing, then still Pap every 3 yrs

tertiary syphilis tx

Pcn G benzathine once weekly for three weeks

What is the preferred imaging modality for assessment of the uterus and adnexal structures?

Pelvic US specifically TVUS over TAUS

What is the management of syphilis ?

Penicillin G - regardless of stage - if allergic to PCN, desensitize and still Tx with PCN Deep Dive: Jarisch-Herxheimer Reaction - PCN causes dead spirochetes which can occlude placenta circulation and result in uterine contractions and late decels

What is the far and away the most common population to get cervical cancer?

People who are not up to date with their pap screening

Define menopause.

Permanent cessation of menses dx after 1 year of amenorrhea - signifies the depletion of oocytes

Hyperemesis Gravidarium

Persistent vomiting that results in weight loss, dehydration, acidosis from starvation, alkalosis from loss of HCl in vomitus, and hypokalemia

When performing amniocentesis, fetal lungs are determined to be mature if: (3 criteria)

Phosphatidylglycerol is present Surfactant-albumin ratio > 55 Lecithin-sphingomyelin ratio > 2

How is Herpes simplex virus spread ?

Physical contact Sexual contact Vertical transmission

VARICELLA-ZOSTER How is varicella zoster virus transmitted? When is someone who has varicella zoster virus infectious ? What is the clinical presentation of varicella zoster virus infection?

Physical contact, Aerosol, Vertical transmission 2 days before rash onset Fever, malaise, pruritic rash (becomes vesicular then crusts over - lesions in various stages of healing)

Sponges: Placement Timing Composition

Placement - placed over cervix Timing - can be inserted up to 24h in advance - leave in place for 6h after sex Composition polyurethane sponge with nonoxynol-9 (spermicide)

Never do a digital vaginal exam in third-trimester bleeding until ______ is ruled out.

Placenta previa

Patient has third trimester bleeding. US shows the baby is lying transversely. What do you suspect?

Placenta previa

Which substances promote the closure of the fetal circulatory shunts?

Prostaglandin inhibitors, acetylcholine, histamine, catecholamines

What are the two most common causes of third trimester bleeding?

Placenta previa and abruption

What is placenta accreta?

Placental villi attach directly to the myometrium rather than to the decidua basalis.

What is placenta increta?

Placental villi invade the myometrium

What is placenta percreta?

Placental villi penetrate through the myometrium. May invade the bladder. *Percreta = Penetrates*

Pattern of plasma hCG elevation and measure at 1, 2, 3, 4, 5, 6, 8-12, and 20 - 40 w GA.

Plasma hCG levels in early pregnancy should double every 48 hours 1 - 5-50 2 - 50-500 3 - 100-10,000 4 - 1,000-30,000 5 - 3,500-115,000 6 - 12,000-270,000 8-12 - 15,000-220,000 20 - 40 - 3,000-15,000

Trending B-hCG

Plateaus at 10 wk (100,000) Increases less with increasing gestational age ACOG reports: < 1500 -> 49% incr >3000 -> 33% incr What you really need to know is that every 48 hours BhCG should incr by at least 66% in the first 6-7 weeks after day 9 Note: A woman with decreasing hCG values and a possible ectopic pregnancy should be monitored until nonpregnant levels are reached because rupture of an ectopic pregnancy can occur while levels are decreasing or are very low. Note: B-hCG does NOT correlate with size , potential rupture, location, or GA of ectopic

What maternal complications can occur with varicella zoster infection in pregnancy?

Pneumonia Hepatitis Encephalitis Death

Endometrial cells seen on pap smear in a woman over the age of 45, think ____________.

Polyps

A 48 yo woman presents to care for abnormal vaginal bleeding. She has always had regular menses every 28-30 days and continues to have regular cycles currently. But she has noticed some light spotting between her periods that she has never had before. She is obese with a BMI of 31, but is otherwise healthy. She has been faithful with routine preventative health and has always had normal Pap smears. On examination, a small amount of blood is seen in the vaginal vault. The cervix appears normal grossly. On bimanual exam, her uterus is mobile and a normal size. No tenderness on palpation. Ultrasound is obtained and shows a focally thickened endometrium. What does this person have?

Polyps = Polypoid mass of hyperplastic endometrial glands. Almost always benign.

Position in Leopold Maneuvers describes...

Position refers to the relation of the presenting part to the right (R) or left (L) side of the birth canal and its direction anteriorly (A), transversely (T), or posteriorly (P) using the anterior and posterior fontanels Cephalic Occipital Presentations: - Occipital anterior (OA) - Occipital posterior (OP) - Left occipital anterior (LOA) - Right occipital anterior (ROA) - Left occipital posterior (LOP) - Right occipital posterior (ROP) - Left occipital transverse (LOT) - Right occipital transverse (ROT) Left references mother's left Note: 90% presenting in OP, "sunny side up", rotate spont into OA

Maternal complication common with LGA delivery

Post-partum hemorrhage

Where does the placenta normally implant?

Posterior or anterior aspect of the fundus

Most common symptom of endometrial cancer

Postmenopausal bleeding

List two types of labor inducing agents.

Prostaglandins - ripen (soften) cervix Then... IV Oxytocin - increases strength and frequency of contractions

What is PGT-A/M?

Pre-implantation genetic testing for aneuploidy and mitoscore: screening of embryos for chromosomal abnormalities prior to transferring embryos

Women with chronic hypertension (HTN) are at high risk for developing _____; what might worsening blood pressure and proteinuria represent?

Preeclampsia; chronic hypertension with superimposed preeclampsia

Which substances/conditions sustain the patency of the fetal shunts

Prostaglandins and intrauterine/neonatal asphyxia

Protraction vs Arrest

Protraction is a slow rate of cervical dilation or descent. Arrest is no change.

Sickle Cell Disease and Pregnancy: Pregnancy Complications Delivery Complications Mgmt

Pregnancy Complications: - thromboses (cerebral vein, DVT, PE) - PNA - pyelo - sepsis syndrome - gestational HTN, preE, eclampsoa Delivery Complications: - placental abruption - PTL - fetal growth restriction - stillbirth Mgmt: - Folic Acid supplementation, 4 mg/day (high dose) accomodates the fast cell turn over seen in sickling - IV hydration and pain control during crises - Oxygen via nasal cannula Note: Prophylactic blood transfusion throughout pregnancy is controversial

What are the fetal side effects of using Indomethacin as a tocolytic agent?

Premature construction of ductus arteriosus Pulmonary HTN Reversible decrease in amniotic fluid

What is placental abruption? (abruptio placentae)

Premature separation of placenta from uterine wall before the delivery of the baby. Pregnant woman + Vaginal bleeding + *Pain* = abruption

Normal standards have been developed based on birth weight, length, and head circumference according to gestational age. Prematurity is birth weight < _____g, and macrosomia is birth weight greater than _____g.

Prematurity <2500g Macrosomia >4500g

Define Endometriosis Incidence Si/Sx Pathophysiology Dx Tx

Presence of endometrial glands and stroma OUTSIDE of the uterus 10-15% of reproductive women, mainly in 20s and 30s, 30-50% have infertility ectopic endometrial tissue is functional and thus responds to hormones (esp estrogen) and you get ectopic menses which results inflammation fibrosis and eventually adhesions Note: a woman with a 1st degree relative has 7% chance of being affected vs unrelated of 1%, develop sx at an earlier age 1/3 Asx Dysmenorrhea - pain can start a few days before period and continue through menses Chronic pelvic pain Dyspareunia - pain with deep penetration vs. with insertion; due to implants in the pouch of Douglas and rectovaginal septum Dyschezia (pain with defecation) - same etiology as dyspareunia Infertility Hematuria Note: sx may improve with pregnancy secondary to cessation of menses Fixed, tender, retroflexed uterus Nodular uterosacral ligaments or thickening Ovarian endometriomas ("chocolate" blood cysts) = tender, palpable, freely mobile adnexal masses Blue, brown vaginal implants (rare) Laparoscopy with biopsy (gold standard) or laparotomy with visualization Biopsy showing: endometrial glands, stroma, and hemosiderin-laden macrophages Tx Goal: induce amenorrhea and regression of implants to do this we suppress estrogen 1st line: NSAIDs (dysmenorrhea) + OCPs (cyclic or continuous) Others: GnRH agonists (Leuprolide) --> suppresses FSH Progestins: Depo, po, implant, IUD Danazol: an androgen derivative that supresses FSH/LH, not often used due to androgenic effects Surgery: Conservative = laparoscopic lysis, ablation, or excision of adhesions and implants Definitive: hysterectomy +/- BSO if refractory and child-bearing done

CD and Surgical Site Infx: Presentation Management

Presentation: fever with erythema, persistent tenderness and purulent drainage from incision site Mgmt: - Gram stain and culture - Drain, irrigate, and debridement of wound - Abx - wet to dry packing if superficial - debrdement if necrotic tissue, consider necrotizing fasciitis

What is the Woods corkscrew maneuver?

Pressure is applied against scapula of posterior shoulder to rotate the posterior shoulder and "unscrew" the anterior shoulder

What are the fetal complications associated with pre/gestational diabetes?

Preterm birth Macrosomia Fetal-growth restriction Congenital anomalies (e.g. caudal regression, cardiac, NTDs) Stillbirths, perinatal death Neonatal hypoglycemia Respiratory distress syndrome Neonatal hypocalcemia Neonatal hyperbilirubinemia Polycythemia

____________ is the most common indication for CD.

Previous CD

What are the risk factors for preterm labor?

Previous history of preterm delivery Hydramnios Multiple gestations Cocaine UTI Vaginal infections Abruption

Ovarian Germ Cell Tumors: Subtypes Signs/Symptoms

Primary cause of ovarian cx in women < 30 yo - most are benign - account for 5-8% of ovarian cancers Subtypes: Teratoma Dysgerminoma Endodermal Sinus (Yolk Sac) Signs/Sx rapidly enlarging pelvic mass --> acute abdomen - fever - some produce hormones --> elevated hcG and AFP - precocious puberty or pregnancy sx (hcG)

Post-delivery hemostasis - what can be done to minimize blood loss?

Primary mechanism = myometrial contraction → vasoconstriction 1. Fundal massage 2. Oxytocin (Pitocin) in the 3rd stage of labor after delivery of anterior shoulders → myometrial contractions → reduces maternal blood loss

When is it recommended to perform an endometrial biopsy (EMB)? What are you assessing for?

Primary use is to assess for endometrial cancer or endometrial hyperplasia Indicated for patients >45 years old with AUB May be performed on patients <45 years old if they are felt to be at increased risk for cancer (i.e. history of unopposed estrogen exposure such as seen in obesity or PCOS) or failed medical management and with persistent AUB

Definition of primary vs. secondary amenorrhea

Primary: -no menses by age 13 and absence of secondary sex characteristics (usually genetic or anatomic abnormality) -or no menses by age 16 with normal growth - take note of presence or absence of breasts and uterus Secondary: -absence of menses for 6+ months after menses had already begun (underlying medical condition)

3 stages of syphilis

Primary: chancre (w lymph) Secondary: rash Tertiary: diffuse disease (neurosyphilis, aortic insufficiency, meningitis, cranial nerve dysfunction)

Describe how a prolactinoma results in amenorrhea.

Prl inhibits GnRH resulting in hypogonadotropid hypogonadism

What is the clinical presentation of HSV in pregnancy?

Prodromal: numbness, tingling, pain Vesicles on an erythematous base that heal w/o scars (unlike with VZ)

Common in late pregnancy, initially found on stretch marks of abdomen, sparing the umbilicus. Found later on the thighs and buttocks.

Pruritic Urticarial Papules & Plaques of Pregnancy (PUPPP)

In *Mitral Stenosis*(MS) and Aortic Stenosis, increased preload due to normal increased blood volume of pregnancy results in left atrial overload. Increased pressure in the left atrium is transmitted to the lungs, resulting in ____.

Pulmonary hypertension --> can lead to pulmonary edema esp with tachycardia at onset of labor Note: 25% of women with MS have cardiac failure for the first time in pregnancy

What is a complication of PROM at < 24 weeks?

Pulmonary hypoplasia. PROM leads to oligohydramnios --> pulmonary hypoplasia. Survival at this age is low.

Describe the hormonal mechanisms/shifts that occur throughout a women's cycle.

Pulsatile GnRH (gonadotropin releasing hormone) produced in the hypothalamus is transferred to the anterior pituitary. GnRH (gonadotropin releasing hormone) then causes the release of gonadotropins (FSH and LH). FSH - stimulates maturation of a follicle in the ovary - The follicle that secretes more estrogen than androgen will be the follicle that releases an egg (primary) The primary follicle will begin to systemically release estradiol (E2), which at low concentrations will cause an inhibitory response to the anterior pituitary, which will inhibit FSH and LH secretion As estradiol levels rise, the endometrium will begin to proliferate FSH will gradually fall as estradiol levels increase LH will be steady, because estradiol secretion from the maturated follicle will be inhibiting its release After ovulation, the follicle remnant is called the corpus luteum --> releases estrogen, inhibin, progesterone Inhibin creates a negative feedback on FSH secretion at the level of the anterior pituitary Inhibin creates a negative feedback on FSH secretion at the level of the anterior pituitary Progesterone creates a negative feedback on GnRH secretion at the level of the hypothalamus 2. progesterone cause a thickening of the endometrium in order to allow for possible implantation of the fertilized oocyte If a fertilized oocyte does not implant on the endometrium, the corpus luteum undergoes luetolysis and now is called the corpus albicans which is essentially scar tissue. Inhibin and progesterone levels will fall, which allow the anterior pituitary and hypothalamus to begin to secrete FSH and GnRH respectively, and will also cause the endometrial lining to fail and slough off, during which time the women will experience a period.

What are the 4 P's of lichen planus?

Purple Pruritic Polygonal Papule

How can the *Apt test* be used to determine maternal vs fetal source of vaginal bleeding?

Put blood from vagina in tube with *KOH*. - Turns brown for maternal - Turns pink for fetus

DDx for of the acute abdomen in pregnancy?

Pyelonephritis pancreatitis cholecystitis appendicitis ovarian torsion ectopic pregnancy labor

What characteristics of nipple discharge have a higher risk of cancer?

Risk of cancer is higher when the discharge is spontaneous (stains clothes), bloody or guaiac positive, unilateral/uniductal associated with a breast mass, or in women over 40 yo more likely to be intraductal carcinoma or papilloma - image and bx

Pregnant women with asymptomatic bacteriuria (5%) should be treated because of their increased risk of developing ______. The majority of cases grow what organism? Discuss routine urine dipstick testing in pregnancy.

Pyelonephritis (25% of those with asx bacteruria) due to: - hydronephrosis (R>L) - progesterone decreasing ureteral tone --> incr vesiculoureteral reflux - gravid uterus compresses lower ureter - urinary stasis 10-20% w/ acute pyelo develop bacteremia Ecoli is cultured in ~80% of cases Note: Routine screening for asx bacteriuria is recommended at 1st prenatal visit, but subsequent screening is governed by site-specific protocols- ACOG guidelines for perinatal care states that in the absence of RFs for UTIs, renal dz, and preeclampsia, and in the absence of sx of UTI, HTN and unusual edema, there has not been shown to be a benefit of routine urine dipstick testing

RUBELLA What organism causes rubella infection? How is rubella transmitted ? When is Rubella most infectious ? Which stage of pregnancy carries the greatest risk of rubella infection?

RNA virus Aerosol route and Vertical transmission 1 week before + 5 days after rash appears T1

Ferning Test

ROM test - swab fluid, allow to dry, place on slide, look for fern pattern under microscope which is due to estrogen's effect on sodium chloride in amniotic fluid - 85 to 98% accurate of ROM

Nitrazine Test

ROM test using nitrazine pH sensitive paper that turns blue in presence of amniotic fluid (basic) - Amniotic fluid (*pH = 7-7.5*) is more alkaline than vaginal secretions (4.5 to 5.5 during preg) - 90 to 98% accurate of ROM

Contraindications to intercourse?

ROMs placenta previa preterm labor

Treatment of Bulky Central Disease

Radical hysterectomy + adjuvant or neoadjuvant radiation Tumor cytoreduction = cytotoxic chem prior to definitive tx with radiation or radical surgery

Stages I-II Vulvar Cx Treatment

Radical vulvectomy and lymphadenectomy (wide local excision is sometimes possible for certain small lesions < 1 cm)

Stages III-IV Vulvar Cx Treatment

Radical vulvectomy and lymphadenectomy plus removal of affected organs and adjunct radiation therapy

syphilis dx

Rapid plasma reagin (RPR) Venereal Disease Research Laboratory (VDRL)

Heterotopic Pregnancy: Define Incidence

Rare complication of pregnancy in which both extra-uterine (ectopic pregnancy) and intrauterine pregnancy occur simultaneously. 1/4000 - 1/30,000 1/100 of IVF

Common side effect of sterilization?

Regret : ( especially in women under 25, not married at the time, or if tubal less than 1 yr postpartum) 10-12% men 13-25% women

What is involved in a radical hysterectomy?

Removal of: uterus cervix parametrial tissue upper vagina

What are contraindications to using Indomethacin as a tocolytic agent?

Renal or hepatic impairment Peptic ulcer disease

When reading FHR tracings, what are four things you must report on?

Report on FHR: 1. Baseline 2. Variability (minimal vs moderate) - moderate variability is nl = variability in HR of 6-25 bpm 3 . Presence of accelerations - note: a reassuring FHR tracing, which means a well oxygenated fetus w/ an intact neuro and CV system, has at least 2 accels within 20 mins of 15+ bpm over baseline lasting 15+secs (2-20-15-15) 4. Presence of decelerations - remember VEAL CHOP for causes of the different types of decels 5. Contractions (note frequency) Also note changes to pattern over time.

Trial of breech vaginal delivery requirements and risk

Requires: - *Frank breech* - GA > 36 weeks - Fetal weight 2,500 to 3,800 g - Fetal head flexed - Favorable pelvis Risks - Birth trauma (esp. *brachial plexus* injuries) - *Prolapsed cord* that traps the aftercoming head

What creates the perfect storm that leads to increased risk of UTIs during peurperium?

Residual urine + bacteruria in a traumatized bladder + dilated ureters and pelvices

Abnormal implantation of the placenta in the uterus (accreta, increta, percreta) can cause _____ & _____ after delivery.

Retention of placenta & heavy bleeding

What is Hemolytic Disease of the Newborn? (HDN)

Rh- mother, Rh+ baby. Mother develops antibodies against the baby's RBC, which cross the placenta and attack the fetal RBCs. This results in fetal RBC hemolysis. The hemolysis results in significant fetal anemia, resulting in fetal heart failure and death.

At birth, the infant's D status is noted. If the infant is D negative, no Rhogam is given to the mother. If the infant is D positive, Rhogam is given within ___ hours of delivery.

Rhogam is given within 72 hours of delivery.

With which pessary is coitus still possible when in place?

Ring pessary! This is only used for mild prolapse.

Counseling for MTX

Risk of ectopic rupture Avoid certain foods/supplements or drugs Avoid pregnancy until resolution and for at least 1 ovulatory cycle after MTX (4-12 wks); some delay pregnancy for 3 months Does not affect subsequent fertility Avoid vigorous activity/sex (rupture) Avoid folic acid, NSAIDs (can potentiate nephro toxicity) Avoid narcotics/alcohol/gas producing foods (mask sx of rupture) Avoid sun exposure (MTX dermatitis) Fetal death/teratogenic effects during pregnancy

Depression Symptoms

SIGECAPS (MDD is 2+ weeks of depressed mood or anehedonia PLUS 4 below): Sleep decreased (Insomnia) Interest decreased in activities (anhedonia) Guilt or worthlessness (Not a major criteria) Energy decreased Concentration difficulties Appetite disturbance or weight loss Psychomotor retardation/agitation Suicidal thoughts

DDx of cervical cancer

STIs --> cervicitis (particularly in the younger age groups) - may present with vaginal discharge associated with post coital or intermenstrual bleeding, dysuria and deep dyspareunia as well as lower abdominal pain. Endometrial cancer- post menopausal bleeding Endometriosis- dysmennorhoea, dyspareunia, pelvic pain and abnormal bleeding Cervical ectropion and cervical polyps may cause post-coital bleeding COCPs - may cause unscheduled bleeding- particularly when first prescribed and ectropion

How should the fetus be delivered if the mother gets primary HSV infection more than 6 weeks before delivery?

SVD + intrapartum acyclovir

Which vaccines are considered safe in pregnancy? Which are unsafe?

Safe: inactivated polio, inactivated typhoid, inactivated influenza, diptheria, tetanus, rabies, meningococcus, Hep B Unsafe: live vaccines, should wait 4 weeks before getting pregnant

Salpingostomy vs Salpingectomy

Salpingostomy Possibly high rate of subsequent IUP with -ostomy by preserving tube; incr r/o repeat ectopic with -ostomy Follow HCG down to non-prego levels if -ostomy; consider MTX x 1 if concerned for incomplete resection as preg tissue left behind may continue to grow Salpingectomy Severe tube damage/excessive bleeding -> -ectomy If other tube is okay, consider -ectomy If woman wants sterilization can do b/l salpingectomy

Psoriasis of vulva: Describe the lesions Tx

Same thing as like that on elbows and knees: red, silver scaly plaques - variable pruritus Tx - steroid cream - topical vit D analogs

Bishop Score

Scoring system that helps determine whether the cervix is favorable or unfavorable for delivery - Cervix must be evaluated if induction of labor is indicated - Score ≥ *8* indicates probability of vaginal delivery after labor induction is similar to those that present with spontaneous labor

If a pt have oligospermia, what's included in the work up?

Screen for testosterone use, chromosome analysis for deletions in Y, hormone panel.

Cervical Cancer: *Screening*vs *Diagnostic* exam

Screening: PAP cytology Dx: Colposcopy with biopsies

Intrapartum Tx of GBS positive mothers.

See Table.

When is intrapartum GBS prophylaxis indicated?

See flow chart.

What is eclampsia?

Seizure or coma without another cause in a patient with preceding preeclampsia. Eclampsia >> hemorrhagic stroke >> death

Puerperal fever duration and DDx

Seldom persists for > 4 to 16 hrs Other causes of fever must be excluded: - mastitis - endometritis - UTI/pyelo - thrombophlebitis

Ovarian cancer that metastisizes to the umbilicus is known as _________________________.

Sister Mary Joseph's nodule so palpate those belly buttons!

Describe the normal structure of an umbilical cord

Semi-rigid cord which circulates blood propelled by fetal heart through 2 arteries and a vein

Structural abnormalities that can cause abortion?

Septate/bicornuate uterus: 25-30% Cervical incompetence Leiomyomas (especially submucosal) Intrauterine adhesions (e.g. from previous curettage)

What is hyperemesis gravidarum?

Severe vomiting that results in - weight loss - dehydration - metabolic derangements

Sheehan Syndrome vs Simmonds Disease

Sheehan (necrosis) - hypotensive episode during or shortly after pregnancy that leads to hemorrhage (vessels are fragile as they are immature recruited quickly as the pituitary enlarges in pregos to support needs for BF) Simmonds (damage) - pituitary damage unrelated to pregnancy

Pituitary condition causing failure to lactate?

Sheehan Syndrome: - excess blood loss (hemorrhage) or extremely low blood pressure during or after labor --> pituitary necrosis --> no prolactin secretion --> no lactation

A 25-year-old G1P0 is delivering her first child. Her labor course was protracted and she pushed for 3 hours. The head is seen to deliver and then retracts, forming the turtle sign. The infant's shoulder does not deliver with gentle symmetric traction. What is the diagnosis? What is the next step in management?

Shoulder dystocia. Calling for help is essential to perform the additional maneuvers.

Fetal circulation has 3 shunts that are not present in adult circulation - what are these structures and what is their purpose?

Shunts deliver oxygenated blood to arterial circulation to bypass non-functional fetal lungs. 1. Ductus venosus - connects portal sinus to IVC 2. Foramen ovale - R to L atrial shunt 3. Ductus arteriosus - between L pulm artery and arch of aorta

What are the features of neonatal HSV infection ?

Skin : - vesicles - ulcers - pustules CNS : Intracranial calcification microcephaly seizures meningoencephalitis EYE : chorioretinitis cataracts

What are the early features of congenital syphilis ?

Skin lesions Lymphadenopathy Hepatosplenomegaly (jaundice) Meningitis Hydrocephalus Seizures/ Intellectual disability Rhinitis (Syphilis Sniffles) PNA Myocarditis Nephrosis

What fetal complications can occur with varicella zoster infection?

Skin scarring Eye defects ( micropthalmia, cataracts, chorioretinitis) Limb hypoplasia Neurological abnormalities (microcephaly) Varicella infection of the newborn

An infant <10th percentile in size

Small for gestational age

Risk factors for cervical cancer

Smoking, immunocompromised, and high risk sexual practices (always associated with HPV)

What is the specific change in cervical mucous during ovulation?

Spinnbarkeit

What is the treatment of toxoplasmosis infection?

Spiramycin sulfadiazine + pyrimethamine (presumptive Tx in late pregnancy - higher toxicity)

Ddx for T1 bleeding?

Spontaneous abortion Ectopic pregnancy Molar pregnancy Vaginal/Cervical lesions or lacerations Implantation bleeding Trauma Infection

Transformation zone

Squamocolumnar junction (most common area for cervical cancer)

List the tanner stages.

Stage 1: prepubertal Stage 2-4 is developmental stages Stage 5 - adult

Vaginal cancer Tx

Stages I-II :Surgical resection +/- radiation Stages III-IV : Chemoradiation only (platinum-based)

In diabetic patients, what is the appropriate management of glycemic control during active labor?

Start insulin drip

Station - Define and describe the two systems.

Station describes the degree of descent of the fetal head (or presenting part) in relation to ischial spines (IS) Two Systems: Both have ischial spine as zero station; both negative above IS and positive below IS 1. Stations −3, −2, −1, 0, +1, +2, +3 Note: +3 station is at the introitus, +1 is just past the ischial spines 2. Divided by centimeters, stations: 5, −4, −3, −2, and −1, 0, +1, +2, +3, +4, +5 +5 station = introitus, +1 is 1 cm past the ischial spines BPD is what is referenced in relation to the IS

Pooling Test

Sterile speculum is used to look for fluid in the *posterior vaginal fornix* to determine if ROM occurred If none seen, ask patient to valsalva to see if fluid is coming from cervical os

Treatment of anticoagulants

Stop medications, but often unable to do so, so endometrial ablation or hysterectomy

Pharmocotherapy/Tx for Urinary Incontinence

Stress - topical estrogen - pessary device - alpha adrenergic - surgery a) Burch retropubic colposuspension b) midurethral sling Urge - timed voiding (urinate in prescribed hour - anti-muscarinic agents (incr bladder capacity, blocks release of Ach during bladder filling) , watch out for dry mouth, changes in vision, and constipation - Mirabegron (3-adrenoreceptor agonist) Overflow - relieve if obstruction - detrusor underactivity - tx underlying neuro condition (DM, B12 def) Overflow

Vaginal Lacerations What type of tear is most common? What tissue is involved in each tear: 1st degree? 2nd? 3rd? 4th?

Stretching of perineum and anus --> spontaneous laceration and anterior tears involving the urethra and labia 1st Degree: Involves perineal skin and vaginal mucosa, but not the underlying fascia and muscle 2nd Degree: 1st D + perineal *fascia and muscle* but not the rectal sphincter 3rd Degree: 2nd D + *anal sphincter* 4th Degree: 3rd D + *rectal mucosa* to *expose the lumen of the rectum*

Rubella can also cause IUGR - what are some problems seen due to congenital rubella?

Structural cardiovascular defects, CNS defects (microcephaly, deafness, glaucoma, cataracts)

↑ TSH, Normal Free T4

Subclinical Hypothyroidism (more common)

What is grading of POP according to the Baden-Walker classification?

Subjective measurements - Not used in clinical trials, but commonly used in daily practice bc easy to understand and implement Grade I = to the level of the ischial spines Grade 2 = between ischial spines and introitus Grade 3 = at the introitus Grade 4 = past introitus

ACOG on breast cancer screening

Summary of ACOG's Updated (2017) Recommendations for Screening Mammography Women at average risk of breast cancer should be offered screening mammography starting at age 40 years. If they have not initiated screening in their 40s, they should begin screening mammography by no later than age 50 years. The decision about the age to begin mammography screening should be made through a shared decision-making process. This discussion should include information about the potential benefits and harms. Women at average risk of breast cancer should have screening mammography every one or two years based on an informed, shared decision-making process that includes a discussion of the benefits and harms of annual and biennial screening and incorporates patient values and preferences. Women at average risk of breast cancer should continue screening mammography until at least 75 years. Beyond age 75 years, the decision to discontinue screening mammography should be based on a shared decision making process informed by the woman's health status and longevity.

Treatment of PCOS?

Suppress ovarian androgen and either control the cycle or allow for pregnancy (as desired): -can use oral contraceptives or insulin sensitizers (metformin) -if pregnancy desired, use clomiphene citrate, letrozole, and insulin sensitizers (metformin) and induce ovulation Eliminate hair growth/loss and acne -can use spironolactone or insulin sensitizers Weight/metabolic control with lifestyle mods Protection of endometrium from damage

Treatment of Invasive Cervical Cancer

Surgery Simple aka Extrafascial Hysterectomy (remove uterus and cervix) - used in microinvasive disease (IA) vs. Radical Hysterectomy (remove uterus, cervix, parametrial tissue and upper 1/3 vagina)with LN dissection (external, internal, common iliac and para-aortic nodes) - performed only if low stage disease (IB-IIA) Radiation - high-dose to cervix and vagina - minimal to bladder and rectum 1. Ext beam whole pelvic radiation 2.. Transvaginal intracavitary cesium - allows for larger doses to the surface of cervix

Pharmacotherapy Stress Urinary Incontinence: Imipramine

Surgery and/or lifestyle modifications are 1st line 1) Dose = 25 to 100 mg at bedtime 2) Mechanism of action = detrusor inhibition by peripheral blockade of norepinephrine reuptake (maybe) & enhanced alpha-adrenergic effect on bladder and urethral smooth muscle (maybe) 3) Evidence: not enough to support use 4) ADRs = anticholinergic 5) Geriatric concerns? Impaired cognition, dry mouth, confusion Not recommended for elderly

Pharmacotherapy Stress Urinary Incontinence: Duloxetine

Surgery and/or lifestyle modifications are 1st line 1) Dose = 40 to 80 mg/day 2) Mechanism of action = SNRI = increased serotonin & norepinephrine release in synapse leads to increased stimulation to urethral sphincter muscle 3) ADRs/Considerations a) *Best* clinical data in terms of efficacy, *but* many drug-drug interactions (2D6/1A2) b) Avoid in liver disease (hepatotoxic) and CrCl under 30 mL/min c) Can increase BP d) Increase suicide risk even in patients without depression (usually with younger patients) e) Withdrawal syndrome

Treatment of GCTs

Surgery: unilateral salpingo-oopherectomy and complete surgical staging Adjuvant chemo: for all malignant GCTs except those in stage 1A, grade 1 immature teratomas (high cure with surgery alone) - The BEP regimen: Bleomycin (SE = pulm fibrosis), Etoposide (blood dyscrasias), cisPlatin (nephrotoxicity) Prognosis is good (found early with rapid growth causing abd pain) with 5 year survival: - 85% dysgerminomas - 75% immature teratomas - 65% yol sac tumors

How is vaginal vault prolapse treated?

Surgical repair using either: - Colpectomy (removal of the vagina) - Colpopexy (suspension of the vaginal apex) *Can use pessaries temporarily in symptomatic patients.

Presentation of polycystic ovarian syndrome (PCOS).

Suspect in hairy (face, chin, back, butt, genitalia, inner thigh), overweight, women with acne, acanthosis nigricans and irregular periods who are having trouble conceiving. Sx highly variable and can be more mild or more severe. Also usually progresses slowly from puberty (if acute, think tumor).

A patient with postpartum hemorrhage has a boggy uterus. What do you suspect?

Suspect uterine atony.

*Autonomic Nervous System* and Urination

Sympathetic system = responsible for normal bladder filling Parasympathetic system = responsible for normal bladder emptying This is important to know when thinking about drug targets Filling of the bladder, sympathetic nervous system inhibits contraction of the detrusor muscle Do not want contraction of the detrusor muscle as the bladder is filling Beta receptors exist all throughout the detrusor muscle As have beta stimulation, have inhibition of the contraction of detrusor muscle As the bladder is filling, want stimulation of the internal sphincter----want that to be closed Bladder is filling, beta stimulation is causing the detrusor muscle to relax Want internal sphincter to be closed = typically is alpha stimulation When time to urinate---parasympathetic system takes over and overrides the sympathetic system As bladder is about ready to void, then parasympathetic causes stimulation (through acetylcholine), and bladder wants to contract at that point Coming out of the parasympathetic = have inhibition of internal sphincter External sphincter is primarily involuntary process

Most common maternal complication causing IUGR

Systemic or pulmonary hypertension

Indication for Anti-resorptive therapy for osteoporosis and osteopenia in post-menopausal women?

T-score < -2.0, or <-1.5 w/ risk factors

EDC US Variability vy Trimester

T1 US +/- 4 days T2 +/- 14 days T3 +/- 21 days

For patients who take multiple medications, you should inform them that OCPs may interact with...

TCAs and anticoagulants, since they are all metabolized in the liver

T/F Pregnant women w/ influenza are more likely to develop severe illness, be hospitalized and require ICU level care.

TRUE

T/F Removal of fallopian tubes at the time of hysterectomy/sterilization has been shown to decrease the lifetime risk of ovarian epithelial cancer

TRUE

T/F Smoking decreases the age of onset of menopause by 3 years.

TRUE

T/F? Use of oral contraceptives decreases a woman's risk of ovarian and uterine cancer.

TRUE Decreases risk of ovarian cx by 75% Decreases risk of endometrial cx by 50%

T/F? The patch and ring may have reduced efficacy in obese women.

TRUE If 200+ lbs

T/F A history of regular menses is a strong indicator of nl ovulation.

TRUE esp if accompanied by menstrual sx (breast tenderness, bloating)

T/F Influenza vaccine should be given to any pregnant woman at any GA.

TRUE the inactivated form of course

T/F Polyps can regress on their own.

TRUE spontaneous regression can occur if asx and < 10 mm

How do the thyroid hormones change during pregnancy?

TSH - unchanged Free T4 - unchanged (euthyroid) Total T3 & T4,and Thyroid-binding globulin - Increased

Lab Testing Timeline and Frequency: TSH (thyroid stimulating hormone) Cholesterol Fasting Glu +/- HgbA1c TB testing

TSH starting at 50 then every 5 years - with periodic screening (19-64) if strong FHx or if autoimmune dz Cholesterol - one time between 17 and 21 - if nl and low risk, start screening every 5 yrs beginning at 45 yo - if nl and high risk, screening starts at age 35 RFs: - familial lipid d/o -FHx of premature coronary artery dz (<55yo) - DM, -mutiple coronary heart dz risk factors (smoking, HTN, obesity) - elevated cholesterol - h/o parent/sibling with blood chol 240+ Fasting Glu +/- HgbA1c begin at age 45, then every 3 yrs younger and more freq if RFs: - DM first-degree relative - obesity - h/o GDM - HTN - high risk ethnicity (Hispanic/African American/Native American) - h/o PCOS - h/o vascular dz - age 45+ - sedentarty - dyslipidemia TB Skin Testing - regular testing for teens - HIV (immunocompromised) - IV drugs used (IVDU) - exp to TB infected person - medically underserved/low-income pop immunocompromised - resident of long-term care facility - recent TB test converter

How can the *Kleihauer Betke test* be used to determine maternal vs fetal source of vaginal bleeding?

Take blood from mother's arm and determine percentage of fetal RBC's in maternal circulation. > 1% = fetal bleeding

Colposcopy Results

The biopsy is reported in terms of CIN

What is the definition of menopause? Why do FSH levels increase during menopause?

The cessation of menstruation (marked by cessation of menses) for 12 months, with low levels of estrogen and high FSH and LH 1. Less negative feedback loop to the anterior pituitary secondary to a decrease in ovarian estrogen 2. Due to the lack of corpus luteum development, there is a lack of inhibin creating a negative feedback loop on the anterior pituitary 3. Resistant oocytes require more FSH to successfully mature

What is uterine rupture?

The disruption of uterine musculature through all of its layers, usually with part of the fetus protruding through the opening

Asherman Syndrome: Defined Causes Dx Tx

The endometrial lining is scarred by surgery or infection, a common cause of secondary amenorrhea ex. curettage after hemorrhage, extensive uterine surgeries (c-section, myomectomy, metroplasty) or TB, schistosomiasis (rare in US) Dx: hysterosalpingogram or hysteroscopy Tx: hysteroscopic resection of intrauterine adhesions, then Estrogen to stimulate regrowth of endometrium

A 35-year-old G4P2012 at 26 weeks gestation is diagnosed with anti-Kell antibodies with titer of 1:16. Amniocentesis shows the fetus is positive for the Kell antigen. In addition to fetal testing with a biophysical profile, what other testing is critical for the fetus?

The fetus should be monitored with middle cerebral artery Doppler, which indicate the severity of anemia.

In order to meet diagnostic criteria for PMS or PMDD, what must occur to symptoms?

The patient must be symptom free during the follicular phase essentially, symptoms resolve shortly after menses begins

What is a NST used to assess? When is it considered a reliable predictor of fetal well being?

The test is used (after 32 weeks gestation) to determine if a fetus is at risk for intrauterine death or neonatal complications, usually secondary to high-risk pregnancies or suspected fetal hypoxemia Note: at 24-28 weeks 50% are non-reactive, and between 28-32 weeks 15% are nonreactive

The function of the pampiniform plexus is to keep the testicles cool, why is this important in regards to fertility?

The testicles need a cooler environment for sperm development!

PUBERTY What is the name given to when females begin to experience breast budding? When does this typically begin and what hormone is responsible? What is the name given to the stage at which females begin to note axillary and pubic hair development? At what age does this typically occur and what hormone is responsible?

Thelarche - begins at 10 years old and is cause by increased estradiol Pubarche - begins at 11 year of age and is cause by adrenal hormones Menarche - 12 yo and is caused by an increase in estradiol

Diagnosing PCOS.

There are specific Rotterdam criteria (2 out of 3 of the following): -oligo or amenorrhea -clinical or biochemical signs of hyperandrogenism -visible polycystic ovaries on US (>12) Most important is exclusion of other causes (CAH, androgen tumors, cushings, exogenous, thyroid disease, primary ovarian insufficiency, acromegaly, prolactinoma)

When is a fetus considered viable?

This is evolving but typically at a gestational age of 23-24weeks or weight of 600g+

Describe briefly the workup of an infertile couple.

Thorough H&P for both partners Male factor: semen analysis Ovulation factor: serum progesterone, day 3 FSH, Prl Uterine factor: US, hysterosonogram, hysterosalpingogram, hysteroscopy Tubal factor: hysterosalpingogram, laparoscopy Endometriosis: laparoscopy

How are pelvic organs supported?

Through a complex pelvic floor musculature (uterosacral ligaments; levator ani and cocccygeal muscles), fascia, and nervous system. Disturbance of any of the following can lead to prolapse: - bony structure - cardinal, broad or round ligaments - endopelvic fascia - pelvic diaphragm - urogenital diaphragm - perineum

Spermicides (foams, creams, & suppositories): Timing (insertion and efficacy) Types (mechanism)

Timing - placed in vagina ~30 mins before - effective for ~ 1 hour - efficacy is much higher when combined with other barrier methods Types nonoxynol-9( most common) octoxynol-3 - both disrupt the sperm cell membrane

Neonatal conjunctivitis: Is it chemical or gonorrhea or chlamydia or HSV?! What helps you figure it out?

Timing: Chemical conjunctivitis (Typically presents within first 24 hours following birth) Neisseria gonorrhea (3-5 days after birth) Chlamydia trachomatis (5-14 days) HSV (1-2 weeks)

List the 4 terrible Ts - causes of postpartum hemorrhage.

Tissue - retained placenta Trauma - instrumentation, lacerations, episiotomy Tone - aTony Thrombin - coag defects, DIC

The different appearances of... Lichen Simplex Chronicus vs Sclerosus vs Planus

To make things simple: Chronicus is hypertrophic thick white (red if itching) Sclerosis is atrophic paper-thin Planus also hypertrophic but shiny purple lesions with white lacey over it

Environmental causes of spontaneous abortion?

Tobacco: >= 14 cigarettes/day = ↑ abortion rates Alcohol Irradiation Environmental Toxin Exposure Caffeine: > 5 cups/day Trauma

What is tocolysis, and what is its role in preterm labor?

Tocolysis is pharmacologic inhibition of uterine contractions. May prolong gestation by a few days to allow the administration of steroids and transfer to facility with NICU. It is used when fetus is < 34 weeks gestation.

HPV vaccination

Traditionally ages 9-26, as of last year ACOG recommended shared decision making in vaccinating 27-45 yo woman stating: Although administration of the HPV vaccine is safe and effective in patients ages 27 to 45, the target age for vaccination - the age at which vaccination confers the greatest benefits to the patient - continues to be 11 to 12 years.

How is placenta previa diagnosed?

Transabdominal ultrasound MRI

Discuss the medical treatment options for chronic anovulatory bleeding with respect to fertility.

Treatment for Chronic Anovulatory Bleeding: 1. Women Desiring Contraception OCs, Depo, Levonorgesterel IUD 2. Women Not Desiring Contraception NSAIDs - start at menses through end of period - decreases bleeding 20-40% - Exs: naproxen 500 mg BID or ibuprofen 600 mg q6-12 or mefenamic acid 250-500 mg q6-12 Tranexamic Acid (Lysteda) - an oral antifibrinolytic agent reducing menstrual bleeding by 1/3 - 650 mg x2 tabs TID at onset of menses for 5 days max - incr risk of thrombosis if taking with OCs - SEs include cramps, HA, back pain

SYPHILIS What organism causes syphilis infection? Transmission

Treponema Pallidum (spirochete) sex, blood, and vertical transmission

syphilis organism

Treponema pallidum

high hCG and inhibin A in Tri _____, low in Tri ______.

Tri 21 (higher number) high Tri 18 low

T/F HRT and Alendronate have similar effect on spinal BMD.

True

T/F There is decreased risk of invasive breast CA w/ ERT but increased risk with HRT.

True - Consider in light of WHI study findings with HRT

True or False: Tocolytics have not been proven to prolong pregnancy.

True.

List and describe the different methods of tubal sterilization.

Tubal sterilization: Laparscopic Tubal Occlusion 80-90% done laparoscopically Electrocautery cauterize 3cm zone of isthmus Clipping clip applied at 90 degree angle on isthmus Banding draw up the isthmus in the end of trocar and pace silicone band or Fallope ring at the base Hysteroscopic Occlusion (Essure) Post-Partum Tubal Occlusion - Pomeroy method: lift isthmus tie suture around to make a loop and excise leaving a gap between dital and proximal end of tubes (most common) - parkland method: tie proximal and distal to isthmus remove piece between - Madlener method: its like pomeroy where you tie a loop but instead don' excise - Irving method: cut isthmus and bury proximal end into myometrium and distal end of tube into mesosalpinx - Kroener method: ligate proximal ampulla and resect distal ampulla with fimbrae - Uchida method: inject epi in serosa of isthmus, reflex mesosalpinx off tube, ligate and excise prox end of tube, mesosalpinx is reattached to proximal stump with the long distal end left to dangle outside of the mesosalpinx Partial or Total Salpingectomy - remove part or all of fallopian tube Other female sterilizations: Colpotomy - enter through posterior vaginal wall near the posterior cul-de-sacand occlude tube using techniques used in laps Hysterectomy - remove uterus either vaginally or abdominally - rarely used for sterilization

Endometrioid endometrial cancer

Type 1

Which type of endometrial cancer has a better prognosis?

Type 1

Endometrial cancer with serous, clear cell, and mixed mullerian morphology

Type 2

Combined Hormonal Contraceptives: Types? Mechanism? Best For? Disadvantages and Contraindications??

Types (column 2) Mechanism (column 3) Best For (column 4) Disadvantages and Contraindications (column 5)

Permanent Sterilization:term-102 Types? Mechanism? Best For? Disadvantages and Contraindications?

Types (column 2) Mechanism (column 3) Best For (column 4) Disadvantages and Contraindications (column 5)

Progestin-Only Pills: Types? Mechanism? Best For? Disadvantages and Contraindications?

Types (column 2) Mechanism (column 3) Best For (column 4) Disadvantages and Contraindications (column 5)

Injectable Contraceptives: Types? Mechanism? Best For? Disadvantages and Contraindications??

Types (see column 2) Mechanism (column 3) Best For (column 4) Disadvantages and Contraindications (column 5)

How often should the fetus be scanned in a woman who had parovirus infection ?

U/S scan : 4 weeks after illness Every 1-2 weeks until 30 weeks gestation Middle cerebral artery doppler assesses fetal anemia status, sample fetal blood for degree of anemia

When treating a patient with AUB, what is the first step that should be taken in order to create a differential diagnosis

UPT

Ectopic: Diagnostic Workup?

UPT!! will be positive with BhCG > 25 approx 1 week s/p conception At minimum need a TVUS to confirm IUP (gestational sac with yolk sac or embryo) Early IUP may be visualized as early as 5w gest sac with yolk sac can be seen on TVUS between 5 and 6 weeks Traditionally the discriminatory zone for TVUS visualization was BhCG 1500-2000 now recs from 3000-3500 before you can say ectopic if no IUP seen Progesterone > 25 ng/mL suggests nl IUP < 5 suggests abnl preg (ectopic or non-viable) 5-25 ??? so not good to rely on this

What investigation is done to assess for fetal VZ infection? What are the possible findings?

US may show: hydrops hyperechogenic foci in the liver and bowel cardiac malformations limb deformities (hypoplasia) microcephaly fetal growth restriction

criteria for hospitalization for PID

Uncontrollable vomiting/can't tolerate PO TOA Pregnant High-fever Patient unreliable/don't think they will do well outpt Failure of initial antibiotics

What are the underlying causes of many neonatal complications of LGA?

Underlying maternal diabetes Birth trauma

Contraindications to HRT?

Undiagnosed vaginal bleeding Breast cancer or strong risk factors metastatic endometrial/ovarian carcinoma History of stroke or MI History of thromboembolic disease: Strong family hx of, or significant risk factors for VTE Active liver disease or impairment Note: may worsen HTN or migraines

Surgical ectopic candidates?

Unstable Sx of ongoing ectopic rupture Intraperitoneal bleeding Failed medical management Need for concurrent surgical procedure Usually laparoscopic, consider laparotomy if excessive bleeding, unstable or poor visualization

Do fibroids recur after myomectomy?

Up to 1/3 recur

What additional pelvic organ prolapse is commonly seen with a cystocele?

Urethrocele

Urine hCG vs. Plasma hCG

Urine: - Preferred method to recognize normal pregnancy - closely parallels plasma concentration - higher level and accuracy in the morning Plasma: - Used for diagnosing ectopic pregnancy - Monitoring trophoblastic tumors - Screening for fetal abnormalities

How can you test to see if a Pt's pain will likely be reduced with a surgical repair?

Use a pessary! This will help the patient appreciate how she will feel after surgery.

Protective factors for Ovarian Cancer?

Use of OCPs multiparity breastfeeding tubal ligation hysterectomy

Third stage of labor average duration? When is it considered prolonged? What can the provider do to help during this stage?

Usually < 10 minutes Prolonged if > 30 minutes; requires manual removal if longer Shorten third stage and decrease blood loss by: apply controlled traction (pull down on cord towards the anus) and fundal massage/IV oxytocin (helps uterus contract)

Malignancies that may present with vaginal bleeding?

Uterine (type 1 and 2 endometrial cancers and sarcomas) and cervical

Fallopian Tube Blood Supply

Uterine and ovarian arteries

Uterus blood supply?

Uterine arteries: arise from internal iliac artery Ovarian arteries

What is the most common cause of postpartum hemorrhage?

Uterine atony - where the uterus does not contract as expected. Normally the uterus contracts, compressing blood vessels and preventing bleeding.

Threatened Abortion: Defined Mgmt

Uterine bleeding from a gestation that is < 20 weeks without cervical dilation or passage of tissue. - 50% will ultimately lose the pregnancy - Increases the risk of preterm labor and delivery. - nothing to do but observe

Endometritis: Define Presentation Treatment

Uterine infection that develops PP day 2-3 and involves the decidua which may involve the myometrium and parametrial tissue incidence greater in CD vs VD and with GBS colonization Presentation: fever, fundal tenderness, foul-smelling lochia without another source of infection

What is this?

Uterine polyps Thickened endometrial stripe : usually irregular thickening.

Etiologies of Post-menopausal bleeding (PMB)

Vaginal and/or endometrial atrophy Endometrial hyperplasia or carcinoma Endometrial or endocervical polyp

What is the clinical presentation of Vasa previa / velamentous cord insertion?

Vaginal bleeding with fetal distress

How can the *Wright's stain* be used to determine maternal vs fetal source of vaginal bleeding?

Vaginal blood: nucleated RBCs indicate fetal bleed.

How much blood loss is considered postpartum hemorrhage?

Vaginal delivery > 500 mL Cesarean > 1000 mL

What is the most common complication of a vaginal pessary? Other complications?

Vaginal mucosal ulceration - urinary retention - constipation - UTIs - vaginal bleeding

If a woman is not immune to varicella zoster and she may have had exposure to someone who had it, what is the management?

Varicella Zoster Immunoglobulin (VZIG) within 10 days of exposure - decreases maternal infection by 50% Deep Dive: - immunity should have be assessed at first prenatal visit - cannot give vaccine to a pregnant woman or newborn as it is a live vaccine - conception should be delayed for 1 month after the last dose because there is a small chance of mild varicella infection after vaccination with the live, attenuated vaccine - can give postpartum as it does not get into breast milk

What are the two conditions that cause third-trimester bleeding from fetal vessel rupture?

Vasa previa Velamentous cord insertion (often occur together)

What is the pathophysiology of preeclampsia?

Vasospasm

Effects of SNRIs on menopausal sx?

Venlaxafine shown to improve VMS as well as mood

Prevention of osteoporosis in menopausal women?

Vit D (800 IU QD) Weight bearing exercise Calcium (1500 mg QD) ERT

This vitamin, when taken in excess, can be teratogenic.

Vitamin A - causes congenital defects of bone, kidneys, CNS Vitamin C can also be harmful if dose exceeds 1g

Which anticoagulation should not be used during pregnancy due to teratogenic potential?

Warfarin Remember: heparin makes babies happy, warfarin wages war (teratogenic but okay with BF) Note: PP can switch to warfarin or continue with LMWH

Fundal Height During Pregnancy

Weeks Pregnant 12 - Barely palpable above pubic symphysis 15 - Midpoint between pubic symphysis and umbilicus 20 - At the umbilicus 28 - 6 cm above the umbilicus 32 - 6 cm below the xyphoid process 36 - 2 cm below xyphoid process 40 - 4 cm below xiphoid process

Side effects of progestins?

Weight gain Edema Thrombophlebitis Headache Hypertension

List common side effects of the Depo shot.

Weight gain (this is the only one linked to weight gain) Spotting infertility Decreased bone mineral density Hair shedding Mood changes lowers HDL decreased libido

What is the role of RhoGAM (Anti-D immune globulins)?

When a mother is given a dose of anti-D IgG, the antibodies bind to the fetal RBC that have the D antigen on them and clear them from the maternal circulation. The goal is to prevent the mother's immune system from recognizing the presence of the D antigen and forming antibodies against it.

What is secondary infertility?

When a woman is unable to bear a child, either due to the inability to become pregnant or the inability to carry a pregnancy to a live birth following either a previous pregnancy or a previous ability to carry a pregnancy to a live birth

What is primary infertility?

When a woman is unable to bear a child, either due to the inability to become pregnant or the inability to carry a pregnancy to a live birth she would be classified as having primary infertility.

% Efficacy for Preventing Pregnancy: Female condomterm-37 Male condom Diaphragm Cervical cap Spermicide Sponge Combined Oral Contraceptives (COCs) Progestin-Only Oral Contraceptives Transdermal Patch (Ortho Evra) Vaginal Ring (NuvaRing) Injectable Hormonal Agent (Depo) Implant (Nexplanon) Hormonal IUD (Mirena, Lilletta, Kyleena, Skyla) Copper IUD (Paraguard) Emergency Contraception Sterilization

Wide range of efficacy (dependent on proper and consistent use): *differ slightly depending on source, this is FA #s Female condom 79% Male condom 86-97% Diaphragm 80-94% Cervical cap - in nulliparous 80-90% - in multiparous 60-70% Spermicide 74-94% Sponge 84% COCs 92-99.9% Progestin-Only Oral Contraceptives 91-97% Transdermal Patch (Ortho Evra) ~ as COCs Vaginal Ring (NuvaRing) ~ as COCs Injectable Hormonal Agent (Depo) 99.7% Implant (Nexplanon) 99.8% Hormonal IUD (Mirena, Lilletta, Kyleena, Skyla) 97-99.1% Copper IUD (Paraguard) 97-99.1% Emergency Contraception - Levonorgestrel pills aka Plan B (with 72 hours of sex): 89% - Copper IUD (w/in 5 days): nearly 100% Sterilization 99.9%

s/sx gonorrhea

Yellow/green vaginal discharge Dysuria AUB Bartholin's abscesses Pelvic pain *Fever, pelvic pain, cervical motion tenderness (PID)* asymptomatic

Can Leiomyomas cause infertility?

Yes (especially submucosal or intramural) due to cavity distortion --> block tubes or risk miscarriage (poor implantation) - however, pregnancy is typically uncomplicated

Can woman who have active HSV breastfeed?

Yes so long as no lesion on breast

Bacterial Vaginosis (BV): Organisms Diagnosis Antepartum Complications Tx

a condition in women in which there is an abnormal overgrowth of anaerobic bacteria - Gardnerella vaginalis, Mycoplasma hominis; replace normal lactobacillus Amsel Clinical Criteria: white gray d/c coats vaginal walls clue cells on microscopy vaginal pH 4.5 whiff test positive: fishy odor when KOH added to d/c Complications: Preterm labor/birth PROM Chorioamnionitis Tx MTZ for sx but does not improve perinatal outcome Don't Tx partner

pomeroy method: picture and description

a loop is made and a suture is tied around it. Then the tube that makes the loop is cut. When the sutures dissolve, both ends of the tube heal closed

Precursors to vulvar cancer?

a lump or wart like lesion lichen sclerosis lichen planus

What maternal investigations are necessary for HSV?

a) Swabs : viral culture , PCR b) Bloods : HSV ab : if primary infection in 3rd trimester

What are complications associated with appendicitis in pregnancy?

abortion, PTL maternal-fetal sepsis --> neonatal neuro injury Note:rupture isre common in T3 (40%) vs T1 (10%)

How quickly do OCPs take effect?

about 7 days - thus, use barrier protection during that time.

US Findings suggestive of ectopic?

absence of gest sac ectopic sac or cardiac activity outside uterus complex adnexal mass fluid in the cul-de-sac (blood from rupture)

what is the most common type of fallopian tube cancer? What is the most common histologic subtype?

adenocarcinoma - papillary serous (90%)

Define spontaneous abortion. What percentage of all pregnancies end in miscarriage?

aka miscarriage pregnancy loss < 20 weeks GA - 30% of all recognized pregnancies end in spontaneous abortion

Who is routine screening for chlamydial and gonorrheal infx?

all sexually active adolescents and high risk females even if Asx

Risk factors for endometrial cancer

also unopposed estrogen stimulation and fam hx Soo anything that raises level or time of exposure of estrogen

17 YOF presents with CC of breast tenderness, nausea, fatigue, HA and bloating. She feels that the OCPs have caused her to gain weight. What do you do?

alter progestin dose to alleviate symptoms and reassure that the pill does not actually cause weight gain

While breast feeding, a woman may experience lactational amenorrhea, which is

amenorrhea and anovulation due to elevated prolactin

The ___________________ of the fallopian tube is where proper fertilization occurs.

ampulla - hence why the ampulla is the most common site of ectopic pregnancy

What is the most common site of an ectopic?

ampulla of fallopian tube

How is HSV treated in pregnancy?

antenatal acyclovir

after a vasectomy, 50% of men develop what?

antisperm antibodies

tertiary syphilis

aortic insufficiency Gummatous Syphilis- nodular lesions in bones and skin, organs CNS- paresis, tabes dorsalis (loss of coordination)

Hidradenomas (Hidradenitis Suppurativa) What is it? Who gets it? Si/Sx Dx Tx

apocrine sweat gland cysts that get occluded in the intertriginous regions aka skin folds (mons pubis, genitocrural folds, buttocks and axillae) and labia majora often foul smelling , pruritic cysts, that get infected and drain forming sinus tracts that become draining fistulas leading to scarring and pits (Hidratenitis Suppurativa) F>M (4xgreater), overweight, African American women Dx = Bx Tx topical steroid creams and long-term oral Abx Severe, excise infected skin

Define arrest of labor when in the second stage.

arrest in the second stage when there is no descent or rotation for at least: 4h in nullip with epidural 3h in nullip w/o epidural or multip w/ epidural 2h in multi w/o epidural

When to begin screening for colon cx?

at age 50 yo if low risk High risk if IBD, colonic polyps, FHx of familial polyposis coli or CRC, or cancer predisposing syndrome (e.g. HNPCC) Rec is 10 years prior to age at Dx of first degree relative

What constitutes adequate contractions for a CST?

at least 3 contractions in 10 mins contractions last at least 40 secs moderate to palpation

When is risk benefit of ERT best?

at onset of menopause (50-59) with short-term use (<5y) Women's Health Initiative demonstrated sig adverse effects in women 60+ or women using it 10+ year s/p meno

tx chlamydia

azithromycin or doxycycline

Why do most spont abortions go unrecognized?

because they occur before or at the time of the next menses (13-26%)

Which phasality of OCPs are associated with break through bleeding?

biphasic or triphasic

Inevitable abortion: Defined

bleeding and cramping with cervical dilation <20 weeks gestation but no passage of tissue

What is the most common cause for PMB? PE findings? Tx?

bleeding due to atrophy of the genital tract 2/2 hypoestrogenism --> atrophy of vagina and endometrium --> friction --> microerosions in uterus and vaginal walls, prone to spotting PE: pale, dry vaginal epi that has lost its ruggae Tx vaginal estrogen cream (can also treat post-radiation effect)

What may uterine enlargement during the initial few hours PP signify?

bleeding in the uterus

Define postmenopausal bleeding (PMB).

bleeding that occurs after one year of menopausal amenorrhea

If a pregnant patient presents with vaginal bleeding, what important blood test should you order?

blood type and Rh Note: if pt is Rh negative and has a negative D (Rh) antibody screen, give anti-D immunoglobulin (RhoGAM) injection

Mastitis: Incidence Presentation Organisms

breast infection in 1-2% approx, 3% of those will develop abscess - 4 weeks postpartum get fever and chills; focal area of erythema and induration with tenderness but no fluctuance - may culture milk to ID organism Orgs: - S Aureus (40%) - Staph coag negative and Strep viridans (60%) TX: dicloxacillin for 7-10 days CONTINUE breastfeeding (BF), sx improve w/in 48h after start Abx

risk of cervicitis

can lead to ascending infection- PID--*chronic pelvic pain, tubal scarring, infertility*

HSV tx

can't get rid of it so goal is to decr sx/transmission acyclovir, valcyclovir, famicyclovir tx initial outbreak 7-10d treat recurrent outbreak x5d

If PMB is accompanied by a foul smell think...

cervical cancer - it bleeds as it outgrows its blood supply and becomes necrotic which makes it smelly

Complication assoc with repeat CKC or LEEP?

cervical incompetence is a complication of repeat biopsies or excisional procedures of the cervix (especially with repeat conizations) - resulting in preterm cervical dilation and effacement - thus, perform cervical cerclage (stitch)

Adjuvant therapy for stage III+ or any pt with serous or clear cell histology?

chemo (carboplatin and paclitaxel) +/- radiation

______ is the most common cause of spontaneous abortion, accounting for 50% of all miscarriages.

chromosomal abnormalities Frequency: aneuploidy (abnl # of chromos): 85% triploidy 10% tetraploidy 4%

90% of immunocompetent women clear HPV infection over ___________ period. __________ HPV infections have a higher likelihood of developing cervical cancer.

clear over a 2 year period (8-24 mos) · Persistent

If biopsy results reveal CIN2+, what should you do?

cold knife cone biopsy (CKC) - scalpel in OR under general or.. loop electrosurgical excision procedure (LEEP) - wire loop with electrical current excise TZ and endocervix in office or OR with local anesthetic - less invasive, not as deep

___________________ epithelium lines the endocervical canal.

columnar

Barrier Methods (Condoms, Diaphragm, Cervical Cap): What is the only method of contraception that protects from STIs? Disadvantages? Advantages and Contraindications? (see table)

condoms (male and female) - STI protection - female condoms offer labial protection, efficacy in preventing pregnancy - decreased sensitivity and potential to break diaphragm - must be fitted by gyn - does not protect against STIs - increase risk of UTI - if left in too long, may result in toxic shock syndrome (2/2 staph aureus infx) cervical cap - high rate of displacement, toxic shock syndrome and cervicitis

dx after initial hsv outbreak

confirm IgG up to 4m after outbreak and IgM after 2wk can screen IgG and IgM at any time many people have no sx

In patients with SLE, the presence of anti-Ro (SS-A) and anti-la (SS-B) are associated with fetal _______.

congenital heart block

Neonatal gonorrheal infection involves...

conjunctivitis which can result in vision loss (neonatal opthalmia) Thus all neonates are given topical prophylaxis of 1% silver nitrate, 0.5% erythromycin, or 1% tetracycline (all considered equally effective for prophylaxis of ocular gonorrhea infection) Deep Dive: SYSTEMIC gonococcal prophylaxis includes single injection of ceftriaxone 50 mg/kg IM or IV in those neonates born to mothers with untreated or suspected gonococcal infection

Goal of Climateric menstrual irregularity treatment?

control menstrual flow until menopause occurs

This IUD cannot be used in women with wilson's disease.

copper IUD

A woman is interested in using a diaphragm. What instructions do you give her?

cover with spermicide jelly, place inside vagina 6 hours before intercourse. Leave in place for 6-8 hours afterward.

Fetal alcohol syndrome results in...

craniofacial defects, limb and cardiovascular defects, prenatal and postnatal growth retardation, intellectual disability

Name the different surgical approaches that are performed for the following prolapse types: cystocele rectocele enterocele apical prolapse uterine prolapse LeFort procedure/colpoclesis

cystocele - anterior colporrhaphy rectocele - posterior colporrhaphy enterocele - Moschcowitz repair - approximate endopelvic fascia and uterosacral ligaments via abdominal approach to prevent enterocele apical prolapse - sacraspinous ligament fixation, uterosacral ligament suspension uterine prolapse - hysterectomy - usually occurs in conjunction with another prolapse so multiple procedures done at same time LeFort procedure/colpoclesis - surgical obliteration of the vaginal canal in female who is NOT sexually active This procedure can be performed with any type of prolapse

Incontinence Studies: cystometry urodynamic studies

cystometry - catheters placed in bladder and rectum measure the relationship of pressure and volume in the bladder 2nd catheter in bladder is to fill it with water - measure post residual volume, volumes at which an urge to urinate occurs, bladder compliance, flow rates, capacity urodynamic studies - studies that eval lower urinary tract function - e.g. cystometry, bladder filling tests, cystoscopy, uroflowmetry and leak-point pressure tests

Define Premature Ovarian Insufficiency/Failure (POI/POF). Possible causes? Tx?

depletion of oocytes resulting in amenorrhea before age 40; high FSH (FSH > 40 2x at least 1 month apart) - radiation or chemo - autoimmune d/o - Fragile X - Turner term-41 Tx - HRT for bone protection (Estrogen helps maintain BMD)

mastitis Tx

dicloxacillin 7-10 d, improves in 2 days - CONTINUE BF

Effects of HRT & ERT on bone density is ___ dependent

dose

What are the 4 types of uterine sarcoma?

doubt you need for exam, may help on wards

Increased risk of vaginal cancer in pre-menopausal women due to...

due to DES in utero (clear cell adenocarcinoma)

GBS Neonatal Sepsis: early onset vs late onset disease

early onset: <7d after birth, vertical transmission can prevent with intrapartum prophylaxis sepsis, PNA, meningitis late onset: 1 week to 3 mo s/p birth meningitis or bacteremia not preventable infx is community acquired or nosocomial

secondary syphilis sx

fatigue, weight loss, fever, sore throat, myalgias, anorexia Diffuse Lymphadenopathy diffuse, maculopapular rash of trunk, extremities, hands, feet Grey lesions on mucosal surfaces, mouth, perineum: condyloma lata May develop GI distress, ocular manifestations, hepatitis, renal abnormalities (etc)

Missed Abortion: Defined Mngmt

fetal demise <20wks GA without expulsion of any POC Dx: pregnant uterus fails to grow appropriately and sx of pregnancy disappear, VB/brown d/c with closed cervix, B-hCG may decline, plateau or continue to incr, absent fetal activity on US or an empty gestational sac 1. Expectant management a. spontaneous delivery of fetal demise within 2 weeks is most common 2. Risk of consumption coagulopathy if dead fetus is not delivered (Disseminated Intravascular Coagulopathy) a. more so in T2 or T3 pregnancies 3. Risk of incomplete or septic abortion, Suction D and C may be indicated 4. medical evacuation of the uterus with miso

What are two consequences of uteroplacental insufficiency leading to IUGR?

fetal hypoxia acidosis May induce "brain sparing": increased resistance in umbilical artery, decreased in MCA

At subsequent visits, what Qs should you always ask?

fetal movement (FM) vaginal bleeding (VB) leakage of fluid (LOF) Contractions or abdominal pain Preeclampsia sx: HA visual disturbances (blurry vision) RUQ pain

trichomonas vaginalis

flagellated protozoan causes vaginitis and or vulvar irritation

PARVO B19 Infection: Erythema Infectiosum or Fifth Disease Maternal Infection Transmission Abs

flu-like first, then slapped cheeks rash, reticular lacey rash Note: 20-30% asx air droplets hand-mouth day care workers IgM 10-12 days after infx persists up to 6 months IgG shows up a few days after IgM and is there for life (lifetime immunity)

HIV testing timing

generally need to wait about 35 to 45 days checking antibodies

Screening if women has 2+ first degree relatives with ovarian cx?

genetic counselor referral - annual CA125 annual TVUS annual pelvic exam - consider BRCA testing and if positive consider prophylactic ooperectomy

what causes cervicitis?

gonorrhea (25-50% cases), chlamydia increasing incidence

2 main side effects of Copper IUD?

heavier cycles and more painful Note: treat with NSAIDs to help with both SEs

What accts for physiologic anemia of pregnancy?

hemodilution - exp plasma volume faster than RBCs

Where are the vestibular glands located?

greater aka bartholin's glands 5 and 7 o'clock of the inferolateral vestibule (the area btwn the labia minora) lesser vestibular glands (aka Skene's glands) are per

GONORRHEA: Symptoms Screening Dx Tx

green-yellow discharge from the cervix of the uterus or discomfort wile urinating, fever, lower abdominal cramping, pain, vaginal bleeding screen at first prenatal visit repeat later if high risk If +, re-screen 3 months after treatment as reinfection is common Dx via PCR Tx with dual therapy ceftriaxone and azithromycin

Once the trophoblast begins to form the placenta, what is the hormone that it secretes that allows for the corpus luteum to continue to function? On average, for how many days can the corpus luteum maintain the integrity of the endometrium in the absence of hCG until it begins to degenerate and the placenta has to take over?

hCG By allowing the corpus luteum to continue to function it ensures that the endometrium is nourished and maintained by the progesterone that is secreted from the corpus luteum 11 days

who to screen for syphilis

high-risk individuals: HIV positive, MSM, sex-workers, high-risk sexual behavior, pregnancy

adverse effects of tubal ligation via electricity?

higher rate of ectopic pregnancy if it fails, can burn surrounding tissue

Describe the different ways fibroids may degenerate over time.

hyaline degeneration calcify red degeneration = painful interstitial hemorrhage often with pregnancy cystic degeneration --> may rupture into adjacent cavities

what is considered pathognomonic for fallopian tube cancer?

hydrops tubae perfluens - cramping pain relieved by watery discharge - colicky abd pain due to intermittent hydrosapinx

SECONDARY AMENORRHEA Possible causes of endocrine dysfunction?

hyper or hypothyroid DM Hyperandrogenism (neoplasm or exogenous)

SECONDARY AMENORRHEA What percent are due to dysfunction at the level of... hypothalamus? pituitary? ovary? uterus?

hypothalamus 35% pituitary 19% ovary 40% uterus 5% other 1% cervical, endocrine

Treatment of hyperplasia with atypia?

hysterectomy if fertility preservation is not desired if fertility desired, high dose progestin (Megestrol acetate) and rebiopsy in 3 mo

Treatment for all stages of endometrial cancer?

hysterectomy is usually abdominal or robotic approach

_______________ is a method of sterilization that involves a titanium spring that is threaded through the fallopian tubes and causes a tissue-reaction for occlusion

hysteroscopy (Essure) 99.8% effective recommended for obese pts for whom laparoscopy is too risky

Implants: Types? Mechanism? Best For? Disadvantages and Contraindications?

i.e. Nexplanon Types (column 2) Mechanism (column 3) Best For (column 4) Disadvantages and Contraindications (column 5)

3 months after IUD insertion, a woman develops symptoms of an STI. When is IUD removal warranted?

if infection is not cured by Abx and if it spreads to fallopian tubes; otherwise treat with IUD in place

When is combination E-P therapy for HRT required?

if uterus is intact -risk of endometrial CA increases 2-10% with unopposed estrogen -Improves abnormal uterine bleeding

Effect of HRT & ERT on Bone Density

improve bone density

What if fundal height is not within 3 cm of GA - what should you consider?

inaccurate dates (most common - order U/S) multiple gestations molar pregnancy IUGR or macrosomia

Vestibulitis What is it? Si/Sx Tx

inflammation of vestibule --> erythema and severe pain on light touch of vestibule or with attempted vaginal entry - insertional dyspareunia and/or postcoital pain - affected area may turn white with acetic acid under colposcopy , not dysplastic though Tx temporary abstinence TCAs xylocaine jelly (anesthetic) - surgery if unresponsive, recurrence still poss after vestibulectomy

sterilization of a woman remote from pregnancy is called____________ sterilization. When should this be performed and why?

interval sterilization - in follicular phase to avoid time of ovulation and possible pregnancy

Neonatal HSV infection % intrauterine, intrapartum, postpartum? What are the three forms of newborn infections?

intrauterine 5% intrapartum 85% postpartum 10% - localized skin, eye, mouth infection - disseminated Herpes - encephalitis

Side effects of implantable contraceptives include...

irregular vaginal bleeding, even after it is removed (often cited for discontinuation) acne decreased libido adnexal enlargement

The ___________________ of the fallopian tube is where sterilization occurs.

isthmus

A new mother wants to start contraception again with Depo, but is worried it will effect her breast milk and baby. What do you tell her?

it will have no effects on your milk or baby, It may increase the volume

Most common complaint in vulvar cancer?

itching and burning of the vulva You'll also see raised white lesions, bleeding, exophytic mass

________ is the most common site of vulvar dysplasia.

labia majora

Define the following terms: menorrhagia metrorrhagia menometrorrhagia polymenorrhea oligomenorrhia

menorrhagia: heavy (> 80 mL) or prolonged (>7d) bleeding, but regular freq of cycles metrorrhagia: Frequent/irregular bleeding "the metro never comes according to schedule" menometrorrhagia: Heavy, irregular bleeding polymenorrhea: < 21 days between bleeding oligomenorrhia: >35 days between bleeding Note: These are historic terms and are no longer rec to use, instead we use AUB (PALM COEIN designation) paired with the descriptor heavy menstrual bleeding (HMB) and/or intermenstrual bleeding (IMB)

mod variability and the presence of accelerations of the FHR predict the absence of __________________.

metabolic acidosis

Management of pregnant women w/ sz d/o?

minimize dose and the number of anticonvulsants - folic acid - may need more anticonvulsants as progress - check levels initially and regularly

Management of pregnant women w/ Sz d/o

minimize dose, fewer anticonvulsants - folic acid - may need more anticonvulsants as progress - check levels regularly, baseline level to est effective amount

If you suspect baby is asleep during the NST, what can be done to wake baby?

mom can eat or drink, retest 1-2 hours after tech can stimulate baby via sound vibrations

Before Dx as vulvar dystrophy, rule out?

more common things like contact dermatitis

What is the clinical presentation of CMV infection in pregnancy ?

most are asymptomatic If sx, mono-like: Fever Malaise Fatigue Lymphadenopathy Pharyngitis

signs of initial HSV episode

most severe outbreak- painful vesicular ulcers itching lymphadenopathy flu like sx low grade fever/ha dysuria

CHLAMYDIA: SX Maternal complications Screening Dx Tx

mostly asx postpartum uterine infection screen at first prenatal visit repeat at T3 if high risk nucleic acid PCR azithromycin or doxycycline

Hepatitis B Transmission Screening Complications Prevention

mostly due to ingestion of infected fluid in the peripartum period or with breastfeeding small amount of transplacental passage Note: increase risk of infectivity with increase levels of HBeAg screen for HepB surface Ag at first visit increased risk of PTL If mom has HepB, prevent neonatal infection by giving infant HBIG and first dose of Hep B vaccine at birth Note: ok to breastfeed if prophylaxis given

HIV Transmission Screening Management

mostly vertical, also breastfeeding (this is the big contraindication to BF) Screen at 1st prenatal visit (opt-out) Decrease vertical transmission from ~25% to 8% by giving zidovudine (ZDV) before, during and (to baby) after birth Reduce duration of ruptured membranes All HIV pregnant patients should receive antiretroviral therapy regardless of CD4 count, the lower the viral load the lower the risk of transmission - Monitor CD4 counts and viral loads, and blood counts and liver function monthly

PE finding of cervicitis

mucopurulent discharge from cervical os

The contraceptive sponge is more effective for____________ women.

nulliparous women

Acute retroviral syndrome - HIV

occurs just before peak viremia, typically 2-4 weeks post exposure and up to 10 months after diarrhea, fever, HA, lymphadenopathy, rash, sore throat, weight loss

Fat necrosis of the breast

often after surgery or trauma (MVA seatbelt injury) with subsequent bleeding into breast tissue - may present as firm, tender, ill-defined mass - can have skin retraction and calcification that mimic malignancy - biopsy will show fat globules and foamy macrophages

_____________ is a fixed pelvic and upper abdominal mass associated with ascites and is pathognomonic for ovarian cancer.

omental caking

Frequency of Td booster? Other immunizations for women?

once btwn 11-13, then every 10 years MMR in non-immune HepB once if not previously immunized Varicella series if not immunized Hep A if high risk (liver dz, IVDU, travel to endemic area Influenza annually (safe in pregos) Meningococcal before starting is HS if not immunized Pneumococcal if 65 or sooner if: sic[kle cell dz, asplenia, alcoholism/cirrhosis - revaccinate after 5 yrs

Neonatal Chlamydia Infections

opthalmia neonatorum: conjunctivitis, blindness PNA

What drug is recommended for tx and post-exposure prophylaxis in pregnant women?

oseltamivir (neuraminidase inhibitor) If treatment, give within 48 hours of sx onset

Patients who undergo tubal ligation are at decreased risk of ______________ cancer.

ovarian cancer

Malignant conditions that cause elevated CA-125?

ovarian cancer adenocarcinoma of cervix adenocarcinoma of endometrium Breast cancer pancreatic cancer lung cancer

First line chemo for ovarian cancer??

paclitaxel and cisplatin or paclitaxel and carboplatin

signs of recurrent episode

painful vesicular ulcers that are less severe

What are the features of primary syphilis? How long does primary syphilis last?

painless genital chancre (ulcerated lesion) lasts 1 week

Cytologic (pap smear) abnormalities are described using the term _________________ whereas histologic (biopsy) abnormalities are described using the term ______________.

pap - squamous intraepithelial lesion (SIL) bx - cervical intraepithelial neoplasm (CIN)

Skene's glands location?

paraurethral glands

When assessing incontinence, what do you want to look out for or perform on PE?

pelvic exam: check for pelvic organ prolapse (POP), masses, atrophic changes, Q tip test: place in urethra, measure change in angle with straining, normal upward motion of <30 deg is nl, >30 deg is abnl indicating urethral hypermobility 2/2 loss of support from the urethrovesicular junction rectal exam: check for impaction +/- rectocele, assess sphincter tone Cough stress test: have pt stand with full bladder and cough, look at urethra and see if you can see leaking Post-void residual US: < 50-100 mL is wnl

What on exam do you expect to find for an ectopic? How about for a ruptured ectopic?

pelvic exam: nl or slightly enlarged uterus - vag bleeding - pelvic pain - palpable adnexal mass Ruptured: hypoTN tachycardia rebound and gaurding

Who should undergo UA/Bacteruria Screening?

periodically for women with DM or 65 ya

Pregnancy with fibroids carries increased risk of....

placental abruption T1 bleeding dysfunctional labor malpresentation CS

Which of the three (Simplex Chronicus vs Sclerosus vs Planus) have an increase risk of developing vulvar carcinoma?

planus and sclerosus

If a pregnant woman becomes infected with chicken pox, what is the treatment?

po Acyclovir (within 24 hrs of rash forming) IV if mom with PNA Deep Dive: infectivity is 48 hours prior to rash until lesions are crusted over

RFs for IUGR

poor nutrition (weight gain < 15 lbs unless obese) smoking drug use (i.e. cocaine) alcoholism severe anemia thrombophilia prolonged pregnancy preeclampsia chromosomal abnormalities placental infarction/hematoma infections multiple gestations monitor IUGR via serial US

If PP headache that is worse when sitting up think... Tx?

postspinal puncture HA Tx: caffeine, hydration, blood patch (anesthesiologist)

What are the complications of GBS infection in pregnancy?

preterm labor PROM chorio fetal/neonatal infections pyelo endometritis UTI

Chronic HTN in pregnancy is HTN prior to _______week of gestation.

prior to the 20th week Note: BP decreases in T2 so if a pregnant woman presents for the first time in T1, she may appear normotensive.

What component of OCs thickens cervical mucous, inhibits sperm migration and creates an unfavorable environment for implantation (endometrial atrophy)?

progesterone

What component of oral contraceptives prevents ovulation by inhibiting LH secretion?

progesterone

Which OCP must be taken at the same time every day?

progestin only OCPs aka POPs (w/in 3 hours)

What OCP is best for women who are breastfeeding, over 40 yo, or in whom estrogen containing OCPs are contraindicated?

progestin only pills

Treatment of hyperplasia without atypia?

progestin therapy - oral (cyclic or daily) - depo - levonorgestrel IUD High regression rate, rebiopsy in 3-6 months to ensure regression

What are the long-term complications of endometriosis?

prolonged bleeding of ectopic tissue causes scarring (adhesions) which may contribute to infertility, SBO, pelvic pain and difficult surgeries

How often should a woman with an uncomplicated pregnancy visit her OB?

q 4 weeks for the first 28 weeks q 2-3 weeks from 28-36 weeks Then q week from 36-41 weeks If 41-42 weeks twice weekly for fetal testing >42 weeks should be delivered given increased risk of stillbirth

What is of highest concern with post-menopausal bleeding?

r/o malignancy

Adjuvant therapy for Stage 1A (invades myometrium) or IB or II?

radiation +/- chemo

Uterine sarcoma - how are most cases diagnosed ? How serious? how do they present?

rapidly enlarging mass with vag bleeding

Ovarian Germ Cell Tumors: Embryonal Carcinoma - characteristics - biomarkers

rare, malignant, avg age 15 (4-28 yo) - primitive embryonal cells - may cause precocious puberty at AUB - B-hCG and AFP tumor markers

What do you need to do if you can palpate a mass but it is not detected on US or mammo?

refer to surgeon for biopsy/excision

Define labor.

regular contractions that result in cervical change

Most hysterectomies start by ligation and transection of the _______________ ligament.

round ligament

_____________ is the most common cause of oligohydramnios.

ruptured membranes - but evaluate for GU malformations/congenital anomalies

complete removal of Fallopian tubes is known as ______________.

salpingectomy

which method of female tubal ligation is associated with decreased incidence of epithelial ovarian cancer?

salpingectomy

TX/staging/prognosis for fallopian tube cancer?

same as ovarian cancer - likely that fallopian, ovarian and peritoneal cancer have a common pathogenesis

__________________ is the most common vulvar cyst.

sebaceous aka epidermoid cyst

When is syphilis most contagious?

secondary

Complications associated with gonorrheal infection in pregnancy?

septic abortion preterm delivery PROM chorioamnionitis postpartum infections

What does a sinusoidal FHR pattern imply?

severe fetal anemia

What is round ligament pain?

sharp u/l or b/l groin pain that typically occurs in T2 that worsens with sudden movement/change in position

outpatient pid tx

single dose ceftriaxone and 2 weeks doxy plus or minus 2 weeks metro

tx trich

single dose of Mentronidazole If treatment fails: 7 days of Metro. r/o reinfection

HSV transmission

skin to skin contact with partner with a lesion *Not prevented by condom use* Partner more contagious during outbreak IgM's develop during 2 weeks after exposure Virus lies dormant in between outbreaks

___________________ epithelium lines the ectocervix.

squamous

Most vulvar cancers are of what type?

squamous; 90%

SECONDARY AMENORRHEA Possible causes of cervical dysfunction?

stenosis 2/2 LEEP or cold-knife cone Tx: cervical dilation

effective, long term birth control with no effort and is the most common method for controlling fertility among men and women

sterilization - male is vasectomy - female is tubal ligation or occlusion 1 million procedures in US, most popular form of BC

____________ and ___________ types of fibroids present with heavy bleeding.

submucosal and intramural Other types: subserosal - just below the serosa/peritoneum, pedunculated stalk or broad cervical parasitic - fibroid steals blood supply from another organ) interligamentous - grows laterally into the broad ligament, likely started out as su

How to treat: submucousal fibroid intrauterine septum uterine didelphys Asherman syndrome

submucousal fibroid - hysteroscopy --> resection/myomectomy intrauterine septum - hysteroscopy uterine didelphys - metroplasty unifies the two cavities Asherman syndrome - hysteroscopy

How does the uterus displace the appendix during pregnancy?

superiolaterally (may not feel pain at McBurney point)

_______ cancers are staged surgically, while ______ are staged clinically.

surgical staging: - ovarian - fallopian - endometrial - vulvar clinical staging: - cervical - vaginal

Endometrial cancer is staged how? Stage is determined by?

surgically 1. the spread of tumor in the uterus 2. the degree of myometrial invasion 3. the presence of extrauterine tumor spread

Staging of vulvar cancer is _______ Staging of vaginal cancer is _______

surgically staged = vulvar clinically staged = vaginally

intimate partner violence (IPV)

suspect IPV with any injury during pregnancy esp those to the abdomen or breasts - pregnant woman are at an increased risk for IPV - always ask if they feel safe at home/in their relationship - sexual abuse occurs in ~ 2/3 of relationships involving physical abuse -assess safety of pt and children - connect her to resources and develop an exit plan for when she is ready to leave or should she feel unsafe - you are required to report, but the patent may not press charges

Breast Abscess: Presentation Dx Tx

suspect if mastitis w/ fever that does not resolve in 2-3 d after Abx tx - fluctuant, tender, palpable mass - U/S to see fluid collection TX: I&D or US guided needle aspiration plus broad spectrum antibiotics - may cont BF and pumping

Meconium staining is more common in _______ than ___________

term and postterm pregnancies than in preterm pregnancies

Define pelvic organ prolapse (POP). Types?

the failure of the pelvic floor musculature to maintain the pelvic organs in their normal position Anterior compartment herniation: cystocele (bladder) anterior vaginal wall Posterior compartment: - rectocele - posterior vag wall Enterocele: herniation of intestines through the vaginal wall Apical: - uterine prolapse - vaginal vault Uterine procidentia: herniation of all three compartments, including uterus through the introitus

Define the puerperium period.

the period of about six weeks after childbirth during which the mother's reproductive organs return to their original non-pregnant condition

Why is parovirus associated with hydrops fetalis?

the virus suppresses fetal erythropoietin

30 YOF presents with amenorrhea. She has been taking OCPs for over a year. She is worried if something is wrong. What do you tell her?

this is common with long term use of OCPs. If you want reassurance we can do a pregnancy test.

Treatment for pregnant woman with vaginal yeast infection?

topical antifungal preferred but okay to give oral fluconazole

Most common protozoan infection in pregnancy is obtained from raw meat or excrement of infected animals - what is it?

toxoplasmosis

What is Chorionic Villus Sampling (CVS)? When is it performed? Risks?

transcervical or transabdominal retrieval of the chorionic villi between 9-12 weeks allows for fetal karyotype 0.5-1% complication rate: preterm delivery PROM fetal injury (esp limb abnormalities if done before 9 weeks)

Where do the majority of cervical cancers arise?

transformation zone (90% arise here) - this is the area of metaplasia from columnar (endocervical) to squamous (ectocervix)

This method of sterilization prevents egg release into uterus.

tubal ligation (2x as many vs vasectomy)

Grading is determined by

tumor histology G1 is well differentiated < 5% solid pattern G2 5-50% solid G3 is poorly differentiated with > 50% solid

HIV opportunistic infections

typically later on Esophageal/oral candidiasis CMV PJP pneumonia

Si/Sx of DVT

unilateral calf/leg swelling calf pain palpable cord in calf

Etiology of vulvar cancer?

unknown, HPV implicated (esp 16, 18, 31, 33)

The majority of breast cancers are detected in what quadrant?

upper outer quadrant

Sebaceous Cysts Sx Appearance Tx

usually Asx occur beneath labia majora when pilosebaceous ducts get blocked palpable, smooth, non-tender mass if expressed --> yellow cheesy thick material Tx none unless gets infected then I&D

What is normal blood loss for vaginal delivery? For C section?

vag ~ 500 cc CS ~ 1000 cc

This method of sterilization prevents sperm release.

vasectomy

What are the three main indications for HRT?

vasomotor sx vag atrophy (although vag estrogen preferred) mood lability (alone or in combo with antidepressant)

HPV transmission Maternal Presentation Management

vertical transmission mostly condyloma acuminata (genital warts) increase in number and size while pregnant, lesions often regress spontaneously after delivery CS not routinely recommended unless obstructing tri or bichloroacetic acid applied weekly for external warts cryotherapy or laser NOT other treatments used in non-pregos like podophyllin resin, 5-FU etc.

Absence of vaginal bleeding after progesterone withdrawal challenge signifies...?

very low Estrogen level

dx of initial hsv outbreak

viral culture IgG will be neg sx only 40% sensitive

At what viral load do you recommend CS before labor or ROM?

viral load > 1000 copies Note: with antiviral therapy and CS, vertical transmission declines from ~25% to 2%

What vitamin is not found in human breast milk?

vitamin K - given shortly after birth to prevent hemorrhagic disease of the newborn

Discriminatory zone for detecting an IUP via TVUS?

was 1500-2000, now ACOG says be conservative: "value should be conservatively high (eg, as high as 3,500 mIU/mL) to avoid the potential for misdiagnosis and possible interruption of an intrauterine pregnancy that a woman hopes to continue" Consider twins (higher HCG at earlier gestational age)..thus be conservative

A patient desires to use the monophasic transdermal contraceptive patch. What instructions do you give her?

wear 1 patch each week for a total of 3 weeks, then no patch for one week (bleed)

Instructions for using the vaginal ring?

wear for 3 weeks, remove for one week, replace same day of the week you removed it Can remove for short periods - do not remove for longer than 3 hours.

trichomonas dx

wet mount of cervical discharge sample- looking for movement (flagellated protozoan) *NAAT most specific*

expedited partner therapy

when patients with STDs are given sufficient medication to treat their partner(s) can write rx for partner without examining them not permissible in all states (OK in NC) increases cure rates *retest 3m to ensure resolution* abstinence until both partners complete tx

is bone mineral density loss reversible in women who use Depo?

yes

In what age group are the non-epithelial cancers more common?

young girls (20s)

How is *CIN* treated? What are complications?

· Large loop excision of the transformation zone (LLETZ) · Laser · Cryosurgery · Electrocoagulation · Cervical excision: *CKC or LEEP* *Complications* · Bleeding, infection · Cervical stenosis · Cervical incompetence

What is performed if a *colposcopy* is *unsatisfactory*?

· Loop electrosurgical excision procedure (*LEEP*) · Cold knife cone (*CKC*)


Kaugnay na mga set ng pag-aaral

CHAPTER FOUR: TAXES, RETIREMENT, AND OTHER INSURANCE CONCEPTS

View Set

Lab 1.2: Networking Topologies and Characteristics

View Set

Character and Culture in Literature

View Set